2001--2012考研英语一真题及答案

发布时间:2020-05-02 22:18:57   来源:文档文库   
字号:

2012考研英语(一)真题参考答案

Section Use of English

  Directions:

  Read the following text. Choose the best word(s) for each numbered blank and mark [A], [B], [C] or [D]on ANSWER SHEET 1. ( 10 points)

  The ethical judgments of the Supreme Court justices became an important issue recently. The court cannot_____ its legitimacy as guardian of the rule of law______ justices behave like politicians. Yet, in several instances, justices acted in ways that_____ the courts reputation for being independent and impartial

  Justices Antonin Scalia and Samuel Alito Jr., for example, appeared at political events. That kind of activity makes it less likely that the courts decisions will be____ as impartial judgments. Part of the problem is that the justices are not _____ by an ethics code. At the very least, the court should make itself_______ to the code of conduct that ______to the rest of the federal judiciary

  This and other cases ______the question of whether there is still a _____ between the court and politics

  The framers of the Constitution envisioned law____ having authority apart from politics. They gave justices permanent positions ____ they would be free to ____those in power and have no need to_____ political support. Our legal system was designed to set law apart from politics precisely because they are so closely _____

  Constitutional law is political because it results from choices rooted in fundamental social ______like liberty and property. When the court deals with social policy decisions, the law it _____is inescapably political which is why decisions split along ideological lines are so easily _____ as unjust

  The justices must _____doubts about the courts legitimacy by making themselves _____to the code of conduct. That would make their rulings more likely to be seen as separate from politics and, _____, convincing as law

  1 A emphasizeB maintainC modifyD recognize 2 A whenB bestC beforeD unles

3 A renderedB weakenedC establishedD eliminated

  4 A challengedB compromisedC suspectedD accepted 5. A advancedB caught C boundD founded 6. A resistantB subjectC immuneD prone 7. A resortsB sticksC leadsD applies 8. A evadeB raiseC denyD settle 9. A lineB barrier C similarity D conflict 10. A byB asC throughD towards 11. A soB sinceC providedD though 12. A serveB satisfyC upsetD replace 13. A confirm B express C cultivate D offer 14 A guardedB followedC studiedD tied

  15. A concepts B theories C divisions D convenience16. A excludes B questions C shapes D controls17. A dismissed B released C ranked D distorted18. A suppress B exploitC addressD ignore 19. A accessibleB. amiableC agreeable D accountable20. A by all meansB at all costsC in a wordD as a result

  Section Ⅱ Reading Comprehension

  Part A Directions:

  Read the following four texts. Answer the questions below each text by choosing [A], [B], [C] or [D]. Mark your answers on ANSWER SHEET1. (40 points)

  Text 2  Pretty in pink: adult women do not remember being so obsessed with the colour, yet it is pervasive in our young girls’ lives. It is not that pink intrinsically bad, but it is a tiny slice of the rainbow and, though it may celebrate girlhood in one way, it also repeatedly and firmly fused girls’ identity to appearance. Then it presents that connection, even among two-year-olds, between girls as not only innocent but as evidence of innocence. Looking around, despaired at the singular lack of imagination about girls’ lives and interests。

  Girls' attraction to pink may seem unavoidable, somehow encoded in their DNA, but according to Jo Paoletti, an associate professor of American Studies, it's not. Children were not colour-coded at all until the early 20th century: in the era before domestic washing machines all babies wore white as a practical matter, since the only way of getting clothes clean was to boil them. What's more, both boys and girls wore what were thought of as gender-neutral dresses. When nursery colours were introduced, pink was actually considered the more masculine colour, a pastel version of red, which was associated with strength. Blue, with its intimations of the Virgin Mary, constancy and faithfulness, symbolised femininity. It was not until the mid-1980s, when amplifying age and sex differences became a dominant children's marketing strategy, that pink fully came into its own, when it began to seem innately attractive to girls, part of what defined them as female, at least for the first few critical years。

  I had not realised how profoundly marketing trends dictated our perception of what is natural to kids, including our core beliefs about their psychological development. Take the toddler. I assumed that phase was something experts developed after years of research into children's behaviour: wrong. Turns out, according to Daniel Cook, a historian of childhood consumerism, it was popularised as a marketing gimmick by clothing manufacturers in the 1930s。

  Trade publications counselled department stores that, in order to increase sales, they should create a "third stepping stone" between infant wear and older kids' clothes. It was only after "toddler" became common shoppers' term that it evolved into a broadly accepted developmental stage. Splitting kids, or adults, into ever-tinier categories has proved a sure-fire way to boost profits. And one of the easiest ways to segment a market is to magnify gender differences – or invent them where they did not previously exist。

  26 By saying "it is ... The rainbow"(line 3, Para 1), the author means pink _______。

  A should not be the sole representation of girlhood

  B should not be associated with girls' innocence

  C cannot explain girls' lack of imagination

  D cannot influence girls' lives and interests

  27 According to Paragraph 2, which of the following is true of colours?

  A Colors are encoded in girls' DNA B Blue used to be regarded as the color for girls

  C Pink used to be a neutral color in symbolizing genders

  D White is preferred by babies

  28 The author suggests that our perception of children's psychological devotement was much influenced by ________。

  [A] the marketing of products for children[B] the observation of children's nature

  [C] researches into children's behavior[D] studies of childhood consumption

  29. We may learn from Paragraph 4 that department stores were advised ________。

  A focuses on infant wear and older kids' clothes

  B attach equal importance to different genders

  C classify consumers into smaller groups D create some common shoppers' terms

  30. it can be concluded that girl's attraction to pink seems to be _____。

  A clearly explained by their inborn tendency B fully understood by clothing manufacturers

  C mainly imposed by profit-driven businessmenD well interpreted by psychological experts

  Part B Directions:

  For questions 41-45, choose the most suitable paragraphs from the list A-G and fill them into the numbered boxes to form a coherent text. Paragraph E has been correctly placed. There is one paragraph which does not fit in with the text. Mark your answers on ANSWER SHEET 1. ( 10 points)

  Part C Directions:

  Read the following text carefully and then translate the underlined segments into Chinese. Your translation should be written clearly on ANSWER SHEET 2. ( 10 points)

  Section Ⅲ Writing  Part A  51. Directions:

  You should write about 100 words on ANSWER SHEET 2.

  Do not sign your own name at the end of the notice. Use "Postgraduates' Association" instead. ( 10 points)

  Part B

  52. Directions:

  Write an essay of 160-200 words based on the following drawing. In your essay, you should

  describe the picture briefly,explain its intended meaning, and give your comments。

  You should write neatly on answer sheet 2.

  2012年全国硕士 研究生入学考试英语试题National Entrance Test of English for MA/MSCandidates (NETEM) 跨考英语教研室—杨凤芝Section Ⅰ Use of English  Directions:

  Read the following text. Choose the best word(s) for each numbered blank

  and mark [A], [B], [C] or [D]on ANSWER SHEET 1. ( 10 points)

  The ethical judgments of the Supreme Court justices became an important issue

  recently. The court cannot_____ its legitimacy as guardian of the rule of

  law______ justices behave like politicians. Yet, in several instances,

  justices acted in ways that_____ the court’s reputation for being independent

  and impartial。

  Justices Antonin Scalia and Samuel Alito Jr., for example, appeared at

  political events. That kind of activity makes it less likely that the court’s

  decisions will be____ as impartial judgments. Part of the problem is that

  the justices are not _____ by an ethics code. At the very least, the court

  should make itself_______ to the code of conduct that ______to the rest of the

  federal judiciary。

  This and other cases ______the question of whether there is still a _____

  between the court and politics。

  The framers of the Constitution envisioned law____ having authority apart

  from politics. They gave justices permanent positions ____ they would be free

  to ____those in power and have no need to_____ political support. Our legal

  system was designed to set law apart from politics precisely because they are

  so closely _____。

  Constitutional law is political because it results from choices rooted in

  fundamental social ______like liberty and property. When the court deals with

  social policy decisions, the law it ____is inescapably political — which

  is why decisions split along ideological lines are so easily _____ as unjust。

  The justices must _____doubts about the court’s legitimacy by making

  themselves _____to the code of conduct. That would make their rulings more

  likely to be seen as separate from politics and, _____, convincing as law。

  1 A emphasize B maintain C modify D recognize 2 A when B best C before D unless

  3 A rendered B weakened C established D eliminated

  4 A challenged B compromised C suspected D accepted 5. A advanced B caught C bound D founded 6. A resistant B subject C immune D prone 7. A resorts B sticks C leads D applies 8. A evade B raise C deny D settle 9. A line B barrier C similarity D conflict 10. A by B as C through D towards 11. A so B since C provided D though 12. A serve B satisfy C upset D replace 13. A confirm B express C cultivate D offer 14 A guarded B followed C studied D tied

  15. A concepts B theories C divisions D convenience16. A excludes B questions C shapes D controls17. A dismissed B released C ranked D distorted18. A suppress B exploit C address D ignore 19. A accessible B. amiable C agreeable D accountable20. A by all means B at all costs C in a word D as a result

  Section Ⅱ Reading ComprehensionPart A

  Directions:

  Read the following four texts. Answer the questions below each text by

  choosing [A], [B], [C] or [D]. Mark your answers on ANSWER SHEET1. (40 points)

  Text 2

  Pretty in pink: adult women do not remember being so obsessed with the

  colour, yet it is pervasive in our young girls’ lives. It is not that pink

  intrinsically bad, but it is a tiny slice of the rainbow and, though it may

  celebrate girlhood in one way, it also repeatedly and firmly fused girls’

  identity to appearance. Then it presents that connection, even among two-year-olds,

  between girls as not only innocent but as evidence of innocence。

  Looking around, despaired at the singular lack of imagination about girls’

  lives and interests。

  Girls' attraction to pink may seem unavoidable, somehow encoded in their DNA,

  but according to Jo Paoletti, an associate professor of American Studies,

  it's not. Children were not colour-coded at all until the early 20th

  century: in the era before domestic washing machines all babies wore white as a

  practical matter, since the only way of getting clothes clean was to boil

  them. What's more, both boys and girls wore what were thought of as gender-neutral dresses. When nursery colours were introduced, pink was actually

  considered the more masculine colour, a pastel version of red, which was

  associated with strength. Blue, with its intimations of the Virgin Mary,

  constancy and faithfulness, symbolised femininity. It was not until the

  mid-1980s, when amplifying age and sex differences became a dominant

  children's marketing strategy, that pink fully came into its own, when it

  began to seem innately attractive to girls, part of what defined them as female,

  at least for the first few critical years。

  I had not realised how profoundly marketing trends dictated our perception

  of what is natural to kids, including our core beliefs about their psychological

  development. Take the toddler. I assumed that phase was something experts

  developed after years of research into children's behaviour: wrong. Turns out,

  according to Daniel Cook, a historian of childhood consumerism, it was

  popularised as a marketing gimmick by clothing manufacturers in the 1930s。

  Trade publications counselled department stores that, in order to increase

  sales, they should create a "third stepping stone" between infant wear and

  older kids' clothes. It was only after "toddler" became common shoppers'

  term that it evolved into a broadly accepted developmental stage. Splitting

  kids, or adults, into ever-tinier categories has proved a sure-fire way to

  boost profits. And one of the easiest ways to segment a market is to

  magnify gender differences –or invent them where they did not previously exist。

  26 By saying "it is ... The rainbow"(line 3, Para 1), the author means pink _______。

  A should not be the sole representation of girlhood

  B should not be associated with girls' innocence

  C cannot explain girls' lack of imagination

  D cannot influence girls' lives and interests

  27 According to Paragraph 2, which of the following is true of colours?

  A Colors are encoded in girls' DNA B Blue used to be regarded as the color for girls

  C Pink used to be a neutral color in symbolizing genders D White is preferred by babies

  28 The author suggests that our perception of children's psychological

  devotement was much influenced by ________。

  [A] the marketing of products for children [B] the observation of children's nature

  [C] researches into children's behavior[D] studies of childhood consumption

  29. We may learn from Paragraph 4 that department stores were advised ________。

  A focuses on infant wear and older kids' clothes B attach equal importance to different genders

  C classify consumers into smaller groupsD create some common shoppers' terms

  30. it can be concluded that girl's attraction to pink seems to be _____。

  A clearly explained by their inborn tendencyB fully understood by clothing manufacturers

  C mainly imposed by profit-driven businessmenD well interpreted by psychological experts

  Part B

  Directions:

  For questions 41-45, choose the most suitable paragraphs from the list

  A-G and fill them into thenumbered boxes to form a coherent text. Paragraph

  E has been correctly placed. There is one paragraph which does not fit in with

  the text. Mark your answers on ANSWER SHEET 1. ( 10 points)

  Part C

  Directions:

  Read the following text carefully and then translate the underlined segments

  into Chinese. Your translation should be written clearly on ANSWER SHEET 2.

  ( 10 points)

  Section Ⅲ Writing  Part A

  51. Directions:

  You should write about 100 words on ANSWER SHEET 2.

  Do not sign your own name at the end of the notice. Use "Postgraduates'

  Association" instead. ( 10 points)

  Part B

  52. Directions:

  Write an essay of 160-200 words based on the following drawing. In your

  essay,you should

  1) describe the picture briefly,2) explain its intended meaning, and

  3) give your comments。You should write neatly on answer sheet 2.

1.B 2.A 3.B 4.D 5.C6.B 7.D 8.B 9.A 10.B

11.A 12.C 13.C 14.D 15.A16.C 17.A 18.C 19.D 20.D

21.D 22.D 23.A 24.C 25.D26.C 27.A 28.A 29.B 30.B

31.A 32.D 33.B 34.D 35.D36.C 37.D 38.B 39.A 40.A

41.C 42.D 43.A 44.F 45.G

46.在物理学上,一种方法是将这种冲动完美发挥到极点并且导找到一种万能的理论---一条我们都可以看的见,明白的普遍公式。

47.在这里,达尔文主义似乎提供了一个准则,如果所有的人类都有共同的起源,那么文化差异能够追寻到更早的可控的起源也是合理的。

48.从我们的共同特征中过滤独特性能够使我们明白文化行为的复杂性起源以及是什么在进化方面和认知方面指导我们人类。

49、其实,由约书亚格林伯说,将更多的经验主义用在了普遍性上,验证许多语言所共有的特点,这些特点被认为是代表了由认知限制造成的偏见。

50. 乔姆斯基的语法应该表现了语言更改的模式,是通过独立的家谱或由它所跟踪的路径,而通过性预测的特定类型间的合作关系。

Part A

47.Directions

Suppose you have found something wrong with the electronic dictionary that you bought from an online store the other day. Write and email to the customer service center to

1) make a complaint and

2) demand a prompt solution.

You should write about 100 words on ANSWER SHEET 2.

Do not sign your own name at the end of the letter. Use “Zhang Wei” instead.

参考范文:

Dear Mr./Miss

This letter is a complaint concerning the electronic dictionary I bought from your online store the other day.

Two weeks ago I mailed the money that ordered and soon received the electronic dictionary. I followed the instructions attached with it. Instead I cannot get it started anyway. After changed the battery inside several times in vain I totally gave it up.

I wrote to your company expecting a prompt solution to this problem. I would hope that you could do something to give me a satisfactory feedback. And I will appreciate it very much if you put a strict test on these electronic dictionaries thus stop the continuing complaints and suffers of other comtomers.

Sincerely Yours

Zhang Wei

 2012年考研英语真题:作文题目及范文

  作文一些外国留学生将要来你的大学,以学生会的名义给他们写一封email,

  1. 表示你的欢迎 2. 为他们在这里的校园生活提供一些建议

  请写100个字左右,请不要在信的末尾写你的名字,用笔名代替,不要写地址。

  范文

  亲爱的同学们,

  首先请允许我代表我校的各位领导老师及同学们对你们的到来表示热烈的欢迎,欢迎来到我校学习和生活。

不同国家的校园生活有所不同,为了使你们的生活更加舒适,下面我将介绍一些在我校生活的一些建议。

  首先,在中国不可以直呼老师的名字,因为中国是一个礼仪之邦,中国人用称呼表达对老师的尊重。

  其次,希望你们珍惜在中国学习的时间,主动增加与中国人交流的机会,这样既能提高你们的汉语水平,也能了解中国的文化。

  最后,如果你们在生活和学习上遇到困难,及时与我们沟通。祝你们在中国的留学生活愉快!

  学生会

  Dear students,

  First of all, allow me, on behalf of the leaders of our school teachers and students are warmly welcome to come to you, welcome to our school and life.

  Campus life is different in different countries, in order to make your life more comfortable, the following I will describe some of the proposals in my school life.

  First of all, cannot call the teacher's name in China, because China is a ritual of ceremony, Chinese used to call the expression of respect for teachers.

  Secondly, I hope you cherish the time studying in China, take the initiative to increase opportunities for interaction with Chinese people, so that both can improve your Chinese language level, can understand Chinese culture.

  Finally, if you encounter difficulty in living and learning, to communicate with us in a timely manner.Wishes you to study abroad in China live in interesting times!

  Student Union

 作文2

  这幅漫画象征性的描述了一个倒在地上的瓶子,一些牛奶洒了出来。在这个瓶子的旁边站着两个人,一个垂头丧气的说全完了!”,而另一个则说幸好还剩点儿!”。这幅画所表达的内容既意义深远又发人深省。

  这幅漫画的目的是告诉我们在生活、工作和学习中遇到挫折时,不同的人持有不同的态度。积极乐观的人总是能够发掘事情好的一面,而消极悲观的人总是为他失去的东西伤心抱怨。总之,一个人的态度能够决定他的成败。

  在我看来,我们应该向那个积极乐观的人学习。在生活中无论遇到什么样的困难,我们都应该用积极乐观的态度来面对。只有这样,我们才能取得成功。

  This cartoon token describes a bottle that fell to the ground, some milk spilled out. The bottle stands next to two people, a dejected saying "is all over! "While another said" Fortunately have left! ” The contents expressed in the painting is both meaningful and thought-provoking.

  This cartoon is designed to tell us to live, work and learning are down, you, different people hold different attitudes. Optimistic people can always discover what's good side, and negative and pessimistic people always losing things sad for him to complain. In short, a person's attitudes can make or break him.

  In my opinion, we should learn from the positive and optimistic man. No matter what difficulties she met in life, we should use a positive and optimistic attitude to face. Only in this way, we can be successful.

2011年考研英语一真题及答案

Section I Use of English

Directions:

Ancient Greek philosopher Aristotle viewed laughter as “a bodily exercise precious to health.” But ­­­_____some claims to the contrary, laughing probably has little influence on physical filness Laughter does _____short-term changes in the function of the heart and its blood vessels, ____ heart rate and oxygen consumption But because hard laughter is difficult to ____, a good laugh is unlikely to have _____ benefits the way, say, walking or jogging does.

____, instead of straining muscles to build them, as exercise does, laughter apparently accomplishes the ____, studies dating back to the 1930’s indicate that laughter. muscles,

Such bodily reaction might conceivably help____the effects of psychological stress.Anyway,the act of laughing probably does produce other types of ______feedback,that improve an individual’s emotional state. ______one classical theory of emotion,our feelings are partially rooted _______ physical reactions. It was argued at the end of the 19th century that humans do not cry ______they are sad but they become sad when te tears begin to flow.

Although sadness also _______ tears,evidence suggests that emotions can flow _____ muscular responses.In an experiment published in 1988,social psychologist Fritz.

1[A]among [B]except [C]despite [D]like

2[A]reflect [B]demand [C]indicate [D]produce

3[A]stabilizing [B]boosting [C]impairing [D]determining

4[A]transmit [B]sustain [C]evaluate [D]observe

5[A]measurable [B]manageable [C]affordable [D]renewable

6[A]In turn [B]In fact [C]In addition [D]In brief

7[A]opposite [B]impossible [C]average [D]expected

8[A]hardens [B]weakens [C]tightens [D]relaxes

9[A]aggravate [B]generate [C]moderate [D]enhance

10[A]physical [B]mentl [C]subconscious [D]internal

11[A]Except for [B]According to [C]Due to [D]As for

12[A]with [B]on [C]in [D]at

13[A]unless [B]until C]if [D]because

14[A]exhausts [B]follows [C]precedes [D]suppresses

15[A]into [B]from [C]towards [D]beyond

16[A]fetch [B]bite [C]pick [D]hold

17[A]disappointed [B]excited [C]joyful [D]indifferent

18[A]adapted [B]catered [C]turned [D]reacted

19[A]suggesting [B]requiring [C]mentioning [D]supposing

20[A]Eventually [B]Consequently [C]Similarly [D]Conversely

Section II Reading Comprehension

Part A

Directions:

Read the following four texts. Answer the questions below each text by choosing [A], [B], [C] or [D]. Mark your answers on ANSWER SHEET 1. (40 points)

Text 1

The decision of the New York Philharmonic to hire Alan Gilbert as its next music director has been the talk of the classical-music world ever since the sudden announcement of his appointment in 2009. For the most part, the response has been favorable, to say the least. “Hooray! At last!” wrote Anthony Tommasini, a sober-sided classical-music critic.

One of the reasons why the appointment came as such a surprise, however, is that Gilbert is comparatively little known. Even Tommasini, who had advocated Gilbert’s appointment in the Times, calls him “an unpretentious musician with no air of the formidable conductor about him.” As a description of the next music director of an orchestra that has hitherto been led by musicians like Gustav Mahler and Pierre Boulez, that seems likely to have struck at least some Times readers as faint praise.

For my part, I have no idea whether Gilbert is a great conductor or even a good one. To be sure, he performs an impressive variety of interesting compositions, but it is not necessary for me to visit Avery Fisher Hall, or anywhere else, to hear interesting orchestral music. All I have to do is to go to my CD shelf, or boot up my computer and download still more recorded music from iTunes.

Devoted concertgoers who reply that recordings are no substitute for live performance are missing the point. For the time, attention, and money of the art-loving public, classical instrumentalists must compete not only with opera houses, dance troupes, theater companies, and museums, but also with the recorded performances of the great classical musicians of the 20th century. There recordings are cheap, available everywhere, and very often much higher in artistic quality than today’s live performances; moreover, they can be “consumed” at a time and place of the listener’s choosing. The widespread availability of such recordings has thus brought about a crisis in the institution of the traditional classical concert.

One possible response is for classical performers to program attractive new music that is not yet available on record. Gilbert’s own interest in new music has been widely noted: Alex Ross, a classical-music critic, has described him as a man who is capable of turning the Philharmonic into “a markedly different, more vibrant organization.” But what will be the nature of that difference? Merely expanding the orchestra’s repertoire will not be enough. If Gilbert and the Philharmonic are to succeed, they must first change the relationship between America’s oldest orchestra and the new audience it hops to attract.

21. We learn from Para.1 that Gilbert’s appointment has

[A]incurred criticism. [B]raised suspicion. [C]received acclaim. [D]aroused curiosity.

22. Tommasini regards Gilbert as an artist who is

[A]influential. [B]modest. [C]respectable. [D]talented.

23. The author believes that the devoted concertgoers

[A]ignore the expenses of live performances. [B]reject most kinds of recorded performances.

[C]exaggerate the variety of live performances. [D]overestimate the value of live performances.

24. According to the text, which of the following is true of recordings?

[A]They are often inferior to live concerts in quality.

[B]They are easily accessible to the general public.

[C]They help improve the quality of music. [D]They have only covered masterpieces.

25. Regarding Gilbert’s role in revitalizing the Philharmonic, the author feels

[A]doubtful. [B]enthusiastic. [C]confident. [D]puzzled.

Text 2

When Liam McGee departed as president of Bank of America in August, his explanation was surprisingly straight up. Rather than cloaking his exit in the usual vague excuses, he came right out and said he was leaving “to pursue my goal of running a company.” Broadcasting his ambition was “very much my decision,” McGee says. Within two weeks, he was talking for the first time with the board of Hartford Financial Services Group, which named him CEO and chairman on September 29.

McGee says leaving without a position lined up gave him time to reflect on what kind of company he wanted to run. It also sent a clear message to the outside world about his aspirations. And McGee isn’t alone. In recent weeks the No.2 executives at Avon and American Express quit with the explanation that they were looking for a CEO post. As boards scrutinize succession plans in response to shareholder pressure, executives who don’t get the nod also may wish to move on. A turbulent business environment also has senior managers cautious of letting vague pronouncements cloud their reputations.

As the first signs of recovery begin to take hold, deputy chiefs may be more willing to make the jump without a net. In the third quarter, CEO turnover was down 23% from a year ago as nervous boards stuck with the leaders they had, according to Liberum Research. As the economy picks up, opportunities will abound for aspiring leaders.

The decision to quit a senior position to look for a better one is unconventional. For years executives and headhunters have adhered to the rule that the most attractive CEO candidates are the ones who must be poached. Says Korn/Ferry senior partner Dennis Carey:”I can’t think of a single search I’ve done where a board has not instructed me to look at sitting CEOs first.”

Those who jumped without a job haven’t always landed in top positions quickly. Ellen Marram quit as chief of Tropicana a decade age, saying she wanted to be a CEO. It was a year before she became head of a tiny Internet-based commodities exchange. Robert Willumstad left Citigroup in 2005 with ambitions to be a CEO. He finally took that post at a major financial institution three years later.

Many recruiters say the old disgrace is fading for top performers. The financial crisis has made it more acceptable to be between jobs or to leave a bad one. “The traditional rule was it’s safer to stay where you are, but that’s been fundamentally inverted,” says one headhunter. “The people who’ve been hurt the worst are those who’ve stayed too long.”

26. When McGee announced his departure, his manner can best be described as being

[A]arrogant. [B]frank. [C]self-centered. [D]impulsive.

27. According to Paragraph 2, senior executives’ quitting may be spurred by

[A]their expectation of better financial status. [B]their need to reflect on their private life.

[C]their strained relations with the boards. [D]their pursuit of new career goals.

28. The word “poached” (Line 3, Paragraph 4) most probably means

[A]approved of. [B]attended to. [C]hunted for. [D]guarded against.

29. It can be inferred from the last paragraph that

[A]top performers used to cling to their posts. [B]loyalty of top performers is getting out-dated.

[C]top performers care more about reputations. [D]it’s safer to stick to the traditional rules.

30. Which of the following is the best title for the text?

[A]CEOs: Where to Go? [B]CEOs: All the Way Up?

[C]Top Managers Jump without a Net [D]The Only Way Out for Top Performers

Text 3

The rough guide to marketing success used to be that you got what you paid for. No longer. While traditional “paid” media – such as television commercials and print advertisements – still play a major role, companies today can exploit many alternative forms of media. Consumers passionate about a product may create “owned” media by sending e-mail alerts about products and sales to customers registered with its Web site. The way consumers now approach the broad range of factors beyond conventional paid media.

Paid and owned media are controlled by marketers promoting their own products. For earned media , such marketers act as the initiator for users’ responses. But in some cases, one marketer’s owned media become another marketer’s paid media – for instance, when an e-commerce retailer sells ad space on its Web site. We define such sold media as owned media whose traffic is so strong that other organizations place their content or e-commerce engines within that environment. This trend ,which we believe is still in its infancy, effectively began with retailers and travel providers such as airlines and hotels and will no doubt go further. Johnson & Johnson, for example, has created BabyCenter, a stand-alone media property that promotes complementary and even competitive products. Besides generating income, the presence of other marketers makes the site seem objective, gives companies opportunities to learn valuable information about the appeal of other companies’ marketing, and may help expand user traffic for all companies concerned.

The same dramatic technological changes that have provided marketers with more (and more diverse) communications choices have also increased the risk that passionate consumers will voice their opinions in quicker, more visible, and much more damaging ways. Such hijacked media are the opposite of earned media: an asset or campaign becomes hostage to consumers, other stakeholders, or activists who make negative allegations about a brand or product. Members of social networks, for instance, are learning that they can hijack media to apply pressure on the businesses that originally created them.

If that happens, passionate consumers would try to persuade others to boycott products, putting the reputation of the target company at risk. In such a case, the company’s response may not be sufficiently quick or thoughtful, and the learning curve has been steep. Toyota Motor, for example, alleviated some of the damage from its recall crisis earlier this year with a relatively quick and well-orchestrated social-media response campaign, which included efforts to engage with consumers directly on sites such as Twitter and the social-news site Digg.

31.Consumers may create “earned” media when they are

[A] obscssed with online shopping at certain Web sites.

[B] inspired by product-promoting e-mails sent to them.

[C] eager to help their friends promote quality products.

[D] enthusiastic about recommending their favorite products.

32. According to Paragraph 2,sold media feature

[A] a safe business environment. [B] random competition.

[C] strong user traffic. [D] flexibility in organization.

33. The author indicates in Paragraph 3 that earned media

[A] invite constant conflicts with passionate consumers.

[B] can be used to produce negative effects in marketing.

[C] may be responsible for fiercer competition.

[D] deserve all the negative comments about them.

34. Toyota Motor’s experience is cited as an example of

[A] responding effectively to hijacked media.[B] persuading customers into boycotting products.

[C] cooperating with supportive consumers. [D] taking advantage of hijacked media.

35. Which of the following is the text mainly about ?

[A] Alternatives to conventional paid media.[B] Conflict between hijacked and earned media.

[C] Dominance of hijacked media. [D] Popularity of owned media.

Text 4

It’s no surprise that Jennifer Senior’s insightful, provocative magazine cover story, “I love My Children, I Hate My Life,” is arousing much chatter – nothing gets people talking like the suggestion that child rearing is anything less than a completely fulfilling, life-enriching experience. Rather than concluding that children make parents either happy or miserable, Senior suggests we need to redefine happiness: instead of thinking of it as something that can be measured by moment-to-moment joy, we should consider being happy as a past-tense condition. Even though the day-to-day experience of raising kids can be soul-crushingly hard, Senior writes that “the very things that in the moment dampen our moods can later be sources of intense gratification and delight.”

The magazine cover showing an attractive mother holding a cute baby is hardly the only Madonna-and-child image on newsstands this week. There are also stories about newly adoptive – and newly single – mom Sandra Bullock, as well as the usual “Jennifer Aniston is pregnant” news. Practically every week features at least one celebrity mom, or mom-to-be, smiling on the newsstands.

In a society that so persistently celebrates procreation, is it any wonder that admitting you regret having children is equivalent to admitting you support kitten-killing ? It doesn’t seem quite fair, then, to compare the regrets of parents to the regrets of the children. Unhappy parents rarely are provoked to wonder if they shouldn’t have had kids, but unhappy childless folks are bothered with the message that children are the single most important thing in the world: obviously their misery must be a direct result of the gaping baby-size holes in their lives.

Of course, the image of parenthood that celebrity magazines like Us Weekly and People present is hugely unrealistic, especially when the parents are single mothers like Bullock. According to several studies concluding that parents are less happy than childless couples, single parents are the least happy of all. No shock there, considering how much work it is to raise a kid without a partner to lean on; yet to hear Sandra and Britney tell it, raising a kid on their “own” (read: with round-the-clock help) is a piece of cake.

It’s hard to imagine that many people are dumb enough to want children just because Reese and Angelina make it look so glamorous: most adults understand that a baby is not a haircut. But it’s interesting to wonder if the images we see every week of stress-free, happiness-enhancing parenthood aren’t in some small, subconscious way contributing to our own dissatisfactions with the actual experience, in the same way that a small part of us hoped getting “ the Rachel” might make us look just a little bit like Jennifer Aniston.

36.Jennifer Senior suggests in her article that raising a child can bring

[A]temporary delight [B]enjoyment in progress

[C]happiness in retrospect [D]lasting reward

37.We learn from Paragraph 2 that

[A]celebrity moms are a permanent source for gossip.

[B]single mothers with babies deserve greater attention.

[C]news about pregnant celebrities is entertaining.

[D]having children is highly valued by the public.

38.It is suggested in Paragraph 3 that childless folks

[A]are constantly exposed to criticism. [B]are largely ignored by the media.

[C]fail to fulfill their social responsibilities. [D]are less likely to be satisfied with their life.

39.According to Paragraph 4, the message conveyed by celebrity magazines is

[A]soothing .[B]ambiguous. [C]compensatory. [D]misleading.

40.Which of the following can be inferred from the last paragraph?

[A]Having children contributes little to the glamour of celebrity moms.

[B]Celebrity moms have influenced our attitude towards child rearing.

[C]Having children intensifies our dissatisfaction with life.

[D]We sometimes neglect the happiness from child rearing.

Part B

Directions:

The following paragraph are given in a wrong order. For Questions 41-45, you are required to reorganize these paragraphs into a coherent text by choosing from the list A-G to filling them into the numbered boxes. Paragraphs E and G have been correctly placed. Mark your answers on ANSWER SHEET 1. (10 points)

[A] No disciplines have seized on professionalism with as much enthusiasm as the humanities. You can, Mr Menand points out, became a lawyer in three years and a medical doctor in four. But the regular time it takes to get a doctoral degree in the humanities is nine years. Not surprisingly, up to half of all doctoral students in English drop out before getting their degrees.

[B] His concern is mainly with the humanities: Literature, languages, philosophy and so on. These are disciplines that are going out of style: 22% of American college graduates now major in business compared with only 2% in history and 4% in English. However, many leading American universities want their undergraduates to have a grounding in the basic canon of ideas that every educated person should posses. But most find it difficult to agree on what a “general education” should look like. At Harvard, Mr Menand notes, “the great books are read because they have been read”-they form a sort of social glue.

[C] Equally unsurprisingly, only about half end up with professorships for which they entered graduate school. There are simply too few posts. This is partly because universities continue to produce ever more PhDs. But fewer students want to study humanities subjects: English departments awarded more bachelor’s degrees in 1970-71 than they did 20 years later. Fewer students requires fewer teachers. So, at the end of a decade of theses-writing, many humanities students leave the profession to do something for which they have not been trained.

[D] One reason why it is hard to design and teach such courses is that they can cut across the insistence by top American universities that liberal-arts educations and professional education should be kept separate, taught in different schools. Many students experience both varieties. Although more than half of Harvard undergraduates end up in law, medicine or business, future doctors and lawyers must study a non-specialist liberal-arts degree before embarking on a professional qualification.

[E] Besides professionalizing the professions by this separation, top American universities have professionalised the professor. The growth in public money for academic research has speeded the process: federal research grants rose fourfold between 1960and 1990, but faculty teaching hours fell by half as research took its toll. Professionalism has turned the acquisition of a doctoral degree into a prerequisite for a successful academic career: as late as 1969a third of American professors did not possess one. But the key idea behind professionalisation, argues Mr Menand, is that “the knowledge and skills needed for a particular specialization are transmissible but not transferable.”So disciplines acquire a monopoly not just over the production of knowledge, but also over the production of the producers of knowledge.

[F] The key to reforming higher education, concludes Mr Menand, is to alter the way in which “the producers of knowledge are produced.”Otherwise, academics will continue to think dangerously alike, increasingly detached from the societies which they study, investigate and criticize.”Academic inquiry, at least in some fields, may need to become less exclusionary and more holistic.”Yet quite how that happens, Mr Menand dose not say.

[G] The subtle and intelligent little book The Marketplace of Ideas: Reform and Resistance in the American University should be read by every student thinking of applying to take a doctoral degree. They may then decide to go elsewhere. For something curious has been happening in American Universities, and Louis Menand, a professor of English at Harvard University, captured it skillfully.

G 41. 42. E 43. 44. 45.

Part C

Directions:

Read the following text carefully and then translate the underlined segments into Chinese. Your translation should be written carefully on ANSWER SHEET 2. (10 points)

With its theme that “Mind is the master weaver,” creating our inner character and outer circumstances, the book As a Man Thinking by James Allen is an in-depth exploration of the central idea of self-help writing.

(46) Allen’s contribution was to take an assumption we all share-that because we are not robots we therefore control our thoughts-and reveal its erroneous nature. Because most of us believe that mind is separate from matter, we think that thoughts can be hidden and made powerless; this allows us to think one way and act another. However, Allen believed that the unconscious mind generates as much action as the conscious mind, and (47) while we may be able to sustain the illusion of control through the conscious mind alone, in reality we are continually faced with a question: “Why cannot I make myself do this or achieve that? ”

Since desire and will are damaged by the presence of thoughts that do not accord with desire, Allen concluded : “ We do not attract what we want, but what we are.” Achievement happens because you as a person embody the external achievement; you don’t “ get success but become it. There is no gap between mind and matter.

Part of the fame of Allen’s book is its contention that “Circumstances do not make a person, they reveal him.” (48) This seems a justification for neglect of those in need, and a rationalization of exploitation, of the superiority of those at the top and the inferiority of those at the bottom.

This ,however, would be a knee-jerk reaction to a subtle argument. Each set of circumstances, however bad, offers a unique opportunity for growth. If circumstances always determined the life and prospects of people, then humanity would never have progressed. In fat, (49)circumstances seem to be designed to bring out the best in us and if we feel that we have been “wronged” then we are unlikely to begin a conscious effort to escape from our situation .Nevertheless, as any biographer knows, a person’s early life and its conditions are often the greatest gift to an individual.

The sobering aspect of Allen’s book is that we have no one else to blame for our present condition except ourselves. (50) The upside is the possibilities contained in knowing that everything is up to us; where before we were experts in the array of limitations, now we become authorities of what is possible.

Section Writing

Part A

51. Directions:

Write a letter to a friend of yours to

1) recommend one of your favorite movies and

2) give reasons for your recommendation

Your should write about 100 words on ANSWER SHEET 2

Do not sign your own name at the end of the leter. User “LI MING” instead.

Do not writer the address.(10 points)

Part B

52. Directions:

Write an essay of 160---200 words based on the following drawing. In your essay, you should

1) describe the drawing briefly,

2) explain it’s intended meaning, and

3) give your comments.

Your should write neatly on ANSWER SHEET 2. (20 points)



附阅读Part A翻译:

译文 1

2009年纽约交响乐团突然宣布聘用艾伦•吉尔伯特为下一位乐曲指挥,从那时起一直到现在,这次任命都成为古典音乐界的话题。退一步说,从总体上看,反应还是不错的。如冷静的古典音乐评论家安东尼•托姆西尼就这样写:从长时间来看,这次委命是英明的。

然而,这次任命还是令人意外。原因之一在于吉乐伯特名声相对较小。就连那时主张雇用吉尔伯特的托姆西尼,也称吉尔伯特其貌不扬,缺乏一位令人敬仰的指挥大师的气质。作为对这个很牛的管弦乐队(牛的表现:到目前为止一直被牛人领导着)下一任指挥家的描述,这种描述跟虚浮的赞扬一样,确实会令至少一部分泰晤士报的读者觉得愕然不解(让他们觉得不可思议)。

就我的观点而言,我不知道吉尔伯特是不是一位伟大的指挥家,甚至连他是不是算好的指挥家也不敢确定。可以确信的是,虽然他演出了很多令人印象深刻的有趣的乐曲。然而,我不需要访问Avery Fisher Hall(可能是纽约交响乐团所在地,即吉尔伯特表演之所),或者其他地方才能听到有趣的管弦乐。(作者意思是,不需要听吉尔伯特,到处可以听到有趣的管弦乐。)我所做的,只需要到我的CD棚里去,随便打开我的电脑,从ITUNES上就可下载比那(当指吉尔伯特表演的)多得多的类似的音乐。

对于唱片,那些专门参加音乐会的人会说,现场表演是不可替代的。他们显然忽视了一个要点。为了替音乐爱好者节省时间、精力、金钱考虑,古典乐曲表演表不仅要在各种表演场所进行竞争,还要在记录这些行为的媒介上竞争。记在唱片上的表演比现场表演更便宜,更易得,甚至质量更好。而且它们的消费时间地点可以任由听者选择。因此,这种唱片的广泛应用,给传统音乐会带来了生存危机。

一个可能的应对方式(解决办法)是古典音乐表演者发明有吸引力的从唱片上听不到的曲子。吉尔伯特在新音乐方面投入了自己的兴趣,这已广被人知:如古典音乐评论家罗斯就把吉尔伯特描述成一个可以扭转交响乐方向的人,认为他把交响乐带进了一个明显不同的更有活力的天地。但是,这种“不同”的实质是什么呢?仅仅扩展交响乐的节目是不够的。吉尔伯特和交响乐要想取得成功,必须首先改变美国旧的管弦乐和它们想吸引的新的听众之间的关系。

译文 2

当列姆•麦克杰八月份从美国银行任上离职时,他的解释确实令人意外。与通常会用的模糊理由不同的是,他直率地说,他离开是为了找一家公司当管理者,而那是他一向就有的追求。他说,作出这一选择纯属个人原因。两周之内,他与哈佛财务服务集团的董事会实现了首次会谈,这一集团在929日聘他担任CEO

麦克杰说,他离开时并没有确定的目标,这使他得以思考自己究竟想管理什么样的公司。这同时也可以让他向外界展示自己的魄力。无独有偶(并不只有他才有这种雄心)。最近几周,雅芳公司和美国联邦快递公司的第二执行官都离开自己的公司,他们的解释都是想当CEO。由于股东施压,董事会需要审查继承人方案,还没有被准许离开的这几位执行官肯定希望事情早点出来结果。商业环境复杂异常,这使得高级经理人员不愿用模糊的声明来损害自己的名声。

随着经济开始出现复苏的迹象,这些希望离任者可能在还没有找到下家时就跳槽。根据“登记册”研究机构的报告,在第三季度,CEO营业额从一年前开始下降了23%,把那些紧跟在这些领导人身后的董事会也弄得神经兮兮。由于经济复苏,那些有抱负的领导人将大有机会。

放弃高级职位去寻找更好的职位,这种决定是非同寻常的,过去可不常见。多年来,执行官和猎头们都坚持认为,最好的CEO候选人需要去挖别人的墙角才能得到(而不是那些主动离开原岗位的人)。某某猎头说,当董事会还没有委托我先去找一个还在任上的CEO时,我不能去考虑那些我在网上一搜就有的人。

那些没有工作去向就跳槽的人不会总是很快找到理想岗位。十年前爱伦•马拉姆从T公司领导人的位子上退下,也是为了当一个CEO。一年前她才成为一家小型电子交易所的领导人。罗伯特在2005年为了当CEO而离开,他最终在一家重要的财务机构找到这种工作是在三年之后。

许多招聘人表示,对于那些最好的演员来说,旧的耻辱正在淡忘。财务危机使得在两个工作机会之间进行选择或者离开更坏的工作这样的行为变得可以接受。“传统规则是,最好呆在你原来的地方,但现在这种规则被从根本上颠覆了。” 一个猎头说,“在一个地方呆得越久,就越容易受损。”

译文3

在过去,销售成功的基本法则是:种瓜得瓜,一分耕耘一分收获。现在不同了。传统的付出方式(媒介,指企业付钱给电视台做广告或者报社做报刊广告)——电视购物和印刷广告——虽然仍占主要地位,但是现在的企业可以开发出更多的替代这些媒介的形式。对产品有热情的用户可能通过给在自己网站上注册的顾客发关于产品和商品的电邮的提醒,来建立自己的媒介。这样用户现在接近了广阔的媒介因素,这些因素超越了传统的付费媒介。

付费并占有的媒介,是被想促销自己产品的商人控制的。而对于白捡的媒介(免费的媒介报道)而言,这种商人的角色仅是作为响应用户需求的第一环(直接面对用户的不是他们)。但是在一些案例中,一个商人拥有的媒介成为另一个商人的付费媒介(但有时候,促销产品的商人也直接面对用户,即把别人占有的媒介暂时变成自己占有的媒介)。例如,当一个电子商务零售商在自己的网站上出售广告空间时,就是如此。我们把这种出售的媒介定义为拥有的媒介。这种(出售空间式的)拥有的媒介是如此强大普遍,以致于其他团体把他们的希望(满意;内容;电子商务发动机)寄托在这种环境中。这种(寄托)趋势虽然依然在婴儿期,但我们相信这种从零售商和旅行提供商(如航空公司、旅馆)有效起步的趋势会越来越强劲。例如强生建立了一个婴儿中心,这是一种杰出的媒介资产,可用于推销提升配套产品,包括那些有竞争力的产品。除了带来利润,除了由于其他商人的到场可以使这个地方显得客观可信,以及给各个公司有机会了解有关其他公司需求的有价值的信息,还能有利于拓展所有公司都关心的用户交易。

这类戏剧性的技术革新给商人带来数量越来越多(种类也越来越多)的通信机会的同时,也同样会提高风险。因为热情的用户会更快、更形象、更有破坏力地表达自己的反对,这种被绑架的媒介,与上述的免费利用的媒介背道而驰(不是商人所希望出现的)。此时,媒介就像人质一样,成为敌人可用的财产或者发起的一次行动(敌人有用户、其他竞争对手、对某商标或产品向来没有好话的社会活动家)。例如,社会网络中的成员正在意识到他们可以绑架媒介,来对建立媒介的那些商人施加压力。

如果这种事情发生了,热情的用户就会努力劝说其他人抵制产品,使得目标公司声名处于危险之中。此时,公司的反应往往不会足够快,也不会足够理性,学习曲线将会变得很陡(学习曲线是表示单位产品生产时间与所生产的产品总数量之间的关系的一条曲线。一般情况下,产品总量越大,单个产品生产时间越短。也可以表示工人一定时间所犯错误数量与练习时间的关系,一般练习时间越长,单位时间内错误越少。这里的曲线陡时大约表示相同产量规模下,现在比原来单个产品所消耗的资源更多,或者说相同练习程度下,一定时间错误更多。总之,是比原来更糟糕了)。例如丰田汽车,今年早些时候通过相对来说较快和精心策划的行动从车辆召回危机中把损尽量降下来,丰田的行动包括努力请用户进土威特这样的地方,挖掘社会新闻的利用等等。

译文 4

毫无疑问,作为有煸动性的杂志封面故事,詹尼弗西尼尔的深刻见解——“我爱我的孩子们,我讨厌我目前的生活状况”——可以唤起人们的谈兴。可是,人们不会想到,养孩子可不是一件完全令人愉悦、生活充实的事情。西尼尔并没有简单地说,孩子使得父母既快乐又痛苦。她建议,我们需要重新定义幸福:幸福不应该像过去那样被定义为由一个个瞬间的快乐组合而成的东西;我们应该把幸福视为一种过去的状态。尽管抚养孩子的日子漫长难熬,令人筋疲力尽,但是西尼尔认为,正是那些心绪沉重的时刻,日后却给我们带来由衷的欣喜。

杂志封面上一位有魅力的母亲抱着一个可爱的婴儿,这种圣母与圣子的图画这周在报摊上可不止西尼尔这一起。例如杂志上讲到最近刚收养孩子的母亲——有时是刚变成单身母亲的人——桑德拉布鲁克,以及那种很常见的“詹尼弗阿尼斯顿怀孕了”的新闻。实际上,每周都有至少一位名流母亲、或者准母亲在杂志上笑迎读者。

在一个坚持不懈地倡导生育的社会中,承认自己后悔生育孩子就相当于承认自己赞同谋杀宠物猫,这难道不值得反思吗?把父母亲的后悔与孩子的后悔相提并论(可能指把作为孩子家长的那种辛苦产生的悔恨理解为根源出在孩子身上,从而产生关于生下孩子的后悔),这显然并不合理。(因此)不情愿养孩子的父母很少会反思自己是否应该养育孩子。但是那不幸福的无孩子的人却为类似“孩子是世上唯一最可珍惜的东西”这样的信息所烦恼。显然,他们的不幸必须通过生儿育女才能得以消除。

当然,在美国周刊与人这样的杂志上所提供的“社会名流父母亲”现象是不切实际的。特别是当“父母亲”是布鲁克这样的单身母亲时更是如此。多项研究表明,有孩子的父母很少比没有孩子的夫妇更快乐,而单亲家庭中的家长烦愁尤甚。这并不奇怪,因为一个人养一个孩子实在太麻烦了。然而,你看看桑德拉和布列尼说的话:自己“一个人”养孩子,其实非常简单。(她们当然觉得简单了,因为她们是在周围人全天候的帮助下养着孩子的。)

当然,要说很多人傻头傻脑地生育孩子,只是因为里斯和安格丽娜这种名流使这种行为看上去显得诱人,这也是不可能的——多数成年人其实理解:养孩子可不是像做个发型那么简单。但是这确实是一件很有趣的值得反思的事情:我们每周看的“轻松快乐做父母”的杂志封面,并不是通过潜意识的方式里让我们对(没有孩子的)现实经历不满,而是这些图片在潜意识中让我们有那种想成为雷切尔的心理,但实际上却使得我们看上去有点像詹尼弗亚尼斯顿。(大约指雷切尔养孩子显得潇洒,而詹尼弗生养孩子显得狼狈。)

2010年全国硕士研究生入学统一考试英语试题

Section I Use of English

Directions:

In 1924 America's National Research Council sent two engineers to supervise a series of industrial experiments at a large telephone-parts factory called the Hawthorne Plant near Chicago. It hoped they would learn how stop-floor lighting workers' productivity. Instead, the studies ended giving their name to the "Hawthorne effect", the extremely influential idea that the very to being experimented upon changed subjects' behavior.

The idea arose because of the behavior of the women in the Hawthorne plant. According to of the experiments, their hourly output rose when lighting was increased, but also when it was dimmed. It did not what was done in the experiment; something was changed, productivity rose. A(n) that they were being experimented upon seemed to be to alter workers' behavior itself.

After several decades, the same data were to econometric the analysis. Hawthorne experiments has another surprise store the descriptions on record, no systematic was found that levels of productivity were related to changes in lighting.

It turns out that peculiar way of conducting the experiments may be have let to interpretation of what happed. , lighting was always changed on a Sunday. When work started again on Monday, output rose compared with the previous Saturday and 17 to rise for the next couple of days. , a comparison with data for weeks when there was no experimentation showed that output always went up on Monday, workers to be diligent for the first few days of the week in any case, before a plateau and then slackening off. This suggests that the alleged "Hawthorne effect" is hard to pin down.

1. [A] affected [B] achieved [C] extracted [D] restored

2. [A] at [B] up [C] with [D] off

3. [A] truth [B] sight [C] act [D] proof

4. [A] controversial [B] perplexing [C] mischievous [D] ambiguous

5. [A] requirements [B] explanations [C] accounts [D] assessments

6. [A] conclude [B] matter [C] indicate [D] work

7. [A] as far as [B] for fear that [C] in case that [D] so long as

8. [A] awareness [B] expectation [C] sentiment [D] illusion

9. [A] suitable [B] excessive [C] enough [D] abundant

10. [A] about [B] for [C] on [D] by

11. [A] compared [B] shown [C] subjected [D] conveyed

12. [A] contrary to [B] consistent with [C] parallel with [D] peculiar to

13. [A] evidence [B] guidance [C] implication [D] source

14. [A] disputable [B] enlightening [C] reliable [D] misleading

15. [A] In contrast [B] For example [C] In consequence [D] As usual

16. [A] duly [B] accidentally [C] unpredictably [D] suddenly

17. [A] failed [B] ceased [C] started [D] continued

20. [A] breaking [B] climbing [C] surpassing [D] hitting

 Section II Reading Comprehension

Part A

Directions:

Read the following four texts. Answer the questions below each text by choosing [A], [B], [C] or [D]. Mark your answers on ANSWER SHEET 1. (40 points)

Text 1

Of all the changes that have taken place in English-language newspapers during the past quarter-century, perhaps the most far-reaching has been the inexorable decline in the scope and seriousness of their arts coverage.

It is difficult to the point of impossibility for the average reader under the age of forty to imagine a time when high-quality arts criticism could be found in most big-city newspapers. Yet a considerable number of the most significant collections of criticism published in the 20th century consisted in large part of newspaper reviews. To read such books today is to marvel at the fact that their learned contents were once deemed suitable for publication in general-circulation dailies.

We are even farther removed from the unfocused newspaper reviews published in England between the turn of the 20th century and the eve of World War II, at a time when newsprint was dirt-cheap and stylish arts criticism was considered an ornament to the publications in which it appeared. In those far-off days, it was taken for granted that the critics of major papers would write in detail and at length about the events they covered. Theirs was a serious business, and even those reviewers who wore their learning lightly, like George Bernard Shaw and Ernest Newman, could be trusted to know what they were about. These men believed in journalism as a calling, and were proud to be published in the daily press. “So few authors have brains enough or literary gift enough to keep their own end up in journalism,” Newman wrote, “that I am tempted to define ‘journalism’ as ‘a term of contempt applied by writers who are not read to writers who are.’”

Unfortunately, these critics are virtually forgotten. Neville Cardus, who wrote for the Manchester Guardian from 1917 until shortly before his death in 1975, is now known solely as a writer of essays on the game of cricket. During his lifetime, though, he was also one of England’s foremost classical-music critics, a stylist so widely admired that his Autobiography (1947) became a best-seller. He was knighted in 1967, the first music critic to be so honored. Yet only one of his books is now in print, and his vast body of writings on music is unknown save to specialists.

Is there any chance that Cardus’s criticism will enjoy a revival? The prospect seems remote. Journalistic tastes had changed long before his death, and postmodern readers have little use for the richly upholstered Vicwardian prose in which he specialized. Moreover, the amateur tradition in music criticism has been in headlong retreat.

21. It is indicated in Paragraphs 1 and 2 that

[A] arts criticism has disappeared from big-city newspapers.

[B] English-language newspapers used to carry more arts reviews.

[C] high-quality newspapers retain a large body of readers.

[D] young readers doubt the suitability of criticism on dailies.

22. Newspaper reviews in England before World War II were characterized by

[A] free themes. [B] casual style. [C] elaborate layout. [D] radical viewpoints.

23. Which of the following would Shaw and Newman most probably agree on?

[A] It is writers' duty to fulfill journalistic goals.[B] It is contemptible for writers to be journalists.

[C] Writers are likely to be tempted into journalism.

[D] Not all writers are capable of journalistic writing.

24. What can be learned about Cardus according to the last two paragraphs?

[A] His music criticism may not appeal to readers today.

[B] His reputation as a music critic has long been in dispute.

[C] His style caters largely to modern specialists.[D] His writings fail to follow the amateur tradition.

25. What would be the best title for the text?

[A] Newspapers of the Good Old Days [B] The Lost Horizon in Newspapers

[C] Mournful Decline of Journalism [D] Prominent Critics in Memory

Text 2

Over the past decade, thousands of patents have been granted for what are called business methods. Amazon.com received one for its "one-click" online payment system. Merrill Lynch got legal protection for an asset allocation strategy. One inventor patented a technique for lifting a box.

Now the nation's top patent court appears completely ready to scale back on business-method patents, which have been controversial ever since they were first authorized 10 years ago. In a move that has intellectual-property lawyers abuzz the U.S. court of Appeals for the federal circuit said it would use a particular case to conduct a broad review of business-method patents. In re Bilski, as the case is known , is "a very big deal", says Dennis D. Crouch of the University of Missouri School of law. It "has the potential to eliminate an entire class of patents."

Curbs on business-method claims would be a dramatic about-face, because it was the federal circuit itself that introduced such patents with is 1998 decision in the so-called state Street Bank case, approving a patent on a way of pooling mutual-fund assets. That ruling produced an explosion in business-method patent filings, initially by emerging internet companies trying to stake out exclusive rights to specific types of online transactions. Later, move established companies raced to add such patents to their files, if only as a defensive move against rivals that might beat them to the punch. In 2005, IBM noted in a court filing that it had been issued more than 300 business-method patents despite the fact that it questioned the legal basis for granting them. Similarly, some Wall Street investment films armed themselves with patents for financial products, even as they took positions in court cases opposing the practice.

The Bilski case involves a claimed patent on a method for hedging risk in the energy market. The Federal circuit issued an unusual order stating that the case would be heard by all 12 of the court's judges, rather than a typical panel of three, and that one issue it wants to evaluate is whether it should "reconsider" its state street Bank ruling.

The Federal Circuit's action comes in the wake of a series of recent decisions by the supreme Court that has narrowed the scope of protections for patent holders. Last April, for example the justices signaled that too many patents were being upheld for "inventions" that are obvious. The judges on the Federal circuit are "reacting to the anti-patent trend at the Supreme Court", says Harold C. Wegner, a patent attorney and professor at George Washington University Law School.

26. Business-method patents have recently aroused concern because of

[A] their limited value to business [B] their connection with asset allocation

[C] the possible restriction on their granting [D] the controversy over authorization

27. Which of the following is true of the Bilski case?

[A] Its ruling complies with the court decisions [B] It involves a very big business transaction

[C] It has been dismissed by the Federal Circuit [D] It may change the legal practices in the U.S.

28. The word "about-face" (Line 1, Para 3) most probably means

[A] loss of good will [B] increase of hostility

[C] change of attitude [D] enhancement of dignity

29. We learn from the last two paragraphs that business-method patents

[A] are immune to legal challenges [B] are often unnecessarily issued

[C] lower the esteem for patent holders [D] increase the incidence of risks

30. Which of the following would be the subject of the text?

[A] A looming threat to business-method patents[B] Protection for business-method patent holders

[C] A legal case regarding business-method patents

[D] A prevailing trend against business-method patents

Text 3

In his book The Tipping Point, Malcolm Gladwell argues that social epidemics are driven in large part by the acting of a tiny minority of special individuals, often called influentials, who are unusually informed, persuasive, or well-connected. The idea is intuitively compelling, but it doesn't explain how ideas actually spread.

The supposed importance of influentials derives from a plausible sounding but largely untested theory called the "two step flow of communication": Information flows from the media to the influentials and from them to everyone else. Marketers have embraced the two-step flow because it suggests that if they can just find and influence the influentials, those selected people will do most of the work for them. The theory also seems to explain the sudden and unexpected popularity of certain looks, brands, or neighborhoods. In many such cases, a cursory search for causes finds that some small group of people was wearing, promoting, or developing whatever it is before anyone else paid attention. Anecdotal evidence of this kind fits nicely with the idea that only certain special people can drive trends

In their recent work, however, some researchers have come up with the finding that influentials have far less impact on social epidemics than is generally supposed. In fact, they don't seem to be required of all.

The researchers' argument stems from a simple observing about social influence, with the exception of a few celebrities like Oprah Winfrey—whose outsize presence is primarily a function of media, not interpersonal, influence—even the most influential members of a population simply don't interact with that many others. Yet it is precisely these non-celebrity influentials who, according to the two-step-flow theory, are supposed to drive social epidemics by influencing their friends and colleagues directly. For a social epidemic to occur, however, each person so affected, must then influence his or her own acquaintances, who must in turn influence theirs, and so on; and just how many others pay attention to each of these people has little to do with the initial influential. If people in the network just two degrees removed from the initial influential prove resistant, for example from the initial influential prove resistant, for example the cascade of change won't propagate very far or affect many people.

Building on the basic truth about interpersonal influence, the researchers studied the dynamics of populations manipulating a number of variables relating of populations, manipulating a number of variables relating to people's ability to influence others and their tendency to be influenced. Our work shows that the principal requirement for what we call "global cascades"– the widespread propagation of influence through networks – is the presence not of a few influentials but, rather, of a critical mass of easily influenced people, each of whom adopts, say, a look or a brand after being exposed to a single adopting neighbor. Regardless of how influential an individual is locally, he or she can exert global influence only if this critical mass is available to propagate a chain reaction.

31. By citing the book The Tipping Point, the author intends to

[A] analyze the consequences of social epidemics[B] discuss influentials' function in spreading ideas

[C] exemplify people's intuitive response to social epidemics

[D] describe the essential characteristics of influentials.

32. The author suggests that the "two-step-flow theory"

[A] serves as a solution to marketing problems[B] has helped explain certain prevalent trends

[C] has won support from influentials[D] requires solid evidence for its validity

33. What the researchers have observed recently shows that

[A] the power of influence goes with social interactions

[B] interpersonal links can be enhanced through the media

[C] influentials have more channels to reach the public

[D] most celebrities enjoy wide media attention

34. The underlined phrase "these people" in paragraph 4 refers to the ones who

[A] stay outside the network of social influence[B] have little contact with the source of influence

[C] are influenced and then influence others [D] are influenced by the initial influential

35. what is the essential element in the dynamics of social influence?

[A] The eagerness to be accepted [B] The impulse to influence others

[C] The readiness to be influenced [D] The inclination to rely on others

Text 4

Bankers have been blaming themselves for their troubles in public. Behind the scenes, they have been taking aim at someone else: the accounting standard-setters. Their rules, moan the banks, have forced them to report enormous losses, and it's just not fair. These rules say they must value some assets at the price a third party would pay, not the price managers and regulators would like them to fetch.

Unfortunately, banks' lobbying now seems to be working. The details may be unknowable, but the independence of standard-setters, essential to the proper functioning of capital markets, is being compromised. And, unless banks carry toxic assets at prices that attract buyers, reviving the banking system will be difficult.

After a bruising encounter with Congress, America's Financial Accounting Standards Board (FASB) rushed through rule changes. These gave banks more freedom to use models to value illiquid assets and more flexibility in recognizing losses on long-term assets in their income statement. Bob Herz, the FASB's chairman, cried out against those who "question our motives." Yet bank shares rose and the changes enhance what one lobby group politely calls "the use of judgment by management."

European ministers instantly demanded that the International Accounting Standards Board (IASB) do likewise. The IASB says it does not want to act without overall planning, but the pressure to fold when it completes it reconstruction of rules later this year is strong. Charlie McCreevy, a European commissioner, warned the IASB that it did "not live in a political vacuum" but "in the real word" and that Europe could yet develop different rules.

It was banks that were on the wrong planet, with accounts that vastly overvalued assets. Today they argue that market prices overstate losses, because they largely reflect the temporary illiquidity of markets, not the likely extent of bad debts. The truth will not be known for years. But bank's shares trade below their book value, suggesting that investors are skeptical. And dead markets partly reflect the paralysis of banks which will not sell assets for fear of booking losses, yet are reluctant to buy all those supposed bargains.

To get the system working again, losses must be recognized and dealt with. America's new plan to buy up toxic assets will not work unless banks mark assets to levels which buyers find attractive. Successful markets require independent and even combative standard-setters. The FASB and IASB have been exactly that, cleaning up rules on stock options and pensions, for example, against hostility from special interests. But by giving in to critics now they are inviting pressure to make more concessions.

36. Bankers complained that they were forced to

[A] follow unfavorable asset evaluation rules [B] collect payments from third parties

[C] cooperate with the price managers [D] reevaluate some of their assets.

37. According to the author , the rule changes of the FASB may result in

[A] the diminishing role of management [B] the revival of the banking system

[C] the banks' long-term asset losses [D] the weakening of its independence

38. According to Paragraph 4, McCreevy objects to the IASB's attempt to

[A] keep away from political influences. [B] evade the pressure from their peers.

[C] act on their own in rule-setting. [D] take gradual measures in reform.

39. The author thinks the banks were "on the wrong planet" in that they

[A] misinterpreted market price indicators [B] exaggerated the real value of their assets

[C] neglected the likely existence of bad debts. [D] denied booking losses in their sale of assets.

40. The author's attitude towards standard-setters is one of

[A] satisfaction. [B] skepticism. [C] objectiveness [D] sympathy

Part B

Directions:

For Questions 41-45, choose the most suitable paragraphs from the list A-G and fill them into the numbered boxes to form a coherent text. Paragraph E has been correctly placed. There is one paragraph which does not fit in with the text. Mark your answers on ANSWER SHEET1. (10 points)

[A] The first and more important is the consumer's growing preference for eating out; the consumption of food and drink in places other than homes has risen from about 32 percent of total consumption in 1995 to 35 percent in 2000 and is expected to approach 38 percent by 2005. This development is boosting wholesale demand from the food service segment by 4 to 5 percent a year across Europe, compared with growth in retail demand of 1 to 2 percent. Meanwhile, as the recession is looming large, people are getting anxious. They tend to keep a tighter hold on their purse and consider eating at home a realistic alternative.

[B] Retail sales of food and drink in Europe's largest markets are at a standstill, leaving European grocery retailers hungry for opportunities to grow. Most leading retailers have already tried e-commerce, with limited success, and expansion abroad. But almost all have ignored the big, profitable opportunity in their own backyard: the wholesale food and drink trade, which appears to be just the kind of market retailers need.

[C] Will such variations bring about a change in the overall structure of the food and drink market? Definitely not. The functioning of the market is based on flexible trends dominated by potential buyers. In other words, it is up to the buyer, rather than the seller, to decide what to buy .At any rate, this change will ultimately be acclaimed by an ever-growing number of both domestic and international consumers, regardless of how long the current consumer pattern will take hold.

[D] All in all, this clearly seems to be a market in which big retailers could profitably apply their scale, existing infrastructure and proven skills in the management of product ranges, logistics, and marketing intelligence. Retailers that master the intricacies of wholesaling in Europe may well expect to rake in substantial profits thereby. At least, that is how it looks as a whole. Closer inspection reveals important differences among the biggest national markets, especially in their customer segments and wholesale structures, as well as the competitive dynamics of individual food and drink categories. Big retailers must understand these differences before they can identify the segments of European wholesaling in which their particular abilities might unseat smaller but entrenched competitors. New skills and unfamiliar business models are needed too.

[E] Despite variations in detail, wholesale markets in the countries that have been closely examined—France, Germany, Italy, and Spain—are made out of the same building blocks. Demand comes mainly from two sources: independent mom-and-pop grocery stores which, unlike large retail chains, are two small to buy straight from producers, and food service operators that cater to consumers when they don't eat at home. Such food service operators range from snack machines to large institutional catering ventures, but most of these businesses are known in the trade as "horeca": hotels, restaurants, and cafes. Overall, Europe's wholesale market for food and drink is growing at the same sluggish pace as the retail market, but the figures, when added together, mask two opposing trends.

[F] For example, wholesale food and drink sales come to $268 billion in France, Germany, Italy, Spain, and the United Kingdom in 2000—more than 40 percent of retail sales. Moreover, average overall margins are higher in wholesale than in retail; wholesale demand from the food service sector is growing quickly as more Europeans eat out more often; and changes in the competitive dynamics of this fragmented industry are at last making it feasible for wholesalers to consolidate.

[G] However, none of these requirements should deter large retailers (and even some large good producers and existing wholesalers) from trying their hand, for those that master the intricacies of wholesaling in Europe stand to reap considerable gains.

41 42 43 44 E 45

Part C

Directions:

Read the following text carefully and then translate the underlined segments into Chinese. Your translation should be written carefully on ANSWER SHEET 2. (10 points)

One basic weakness in a conservation system based wholly on economic motives is that most members of the land community have no economic value. Yet these creatures are members of the biotic community and, if its stability depends on its integrity, they are entitled to continuance.

When one of these noneconomic categories is threatened and, if we happen to love it .We invert excuses to give it economic importance. At the beginning of century songbirds were supposed to be disappearing. (46) Scientists jumped to the rescue with some distinctly shaky evidence to the effect that insects would eat us up if birds failed to control them. the evidence had to be economic in order to be valid.

It is painful to read these round about accounts today. We have no land ethic yet, (47) but we have at least drawn near the point of admitting that birds should continue as a matter of intrinsic right, regardless of the presence or absence of economic advantage to us.

A parallel situation exists in respect of predatory mammals and fish-eating birds. (48) Time was when biologists somewhat over worded the evidence that these creatures preserve the health of game by killing the physically weak, or that they prey only on "worthless" species.

Some species of tree have been read out of the party by economics-minded foresters because they grow too slowly, or have too low a sale vale to pay as timber crops. (49) In Europe, where forestry is ecologically more advanced, the non-commercial tree species are recognized as members of native forest community, to be preserved as such, within reason.

To sum up: a system of conservation based solely on economic self-interest is hopelessly lopsided. (50) It tends to ignore, and thus eventually to eliminate, many elements in the land community that lack commercial value, but that are essential to its healthy functioning. It assumes, falsely, I think, that the economic parts of the biotic clock will function without the uneconomic parts.

Section Writing

Part A

51. Directions:

You are supposed to write for the postgraduate association a notice to recruit volunteers for an international conference on globalization, you should conclude the basic qualification of applicant and the other information you think relative.

You should write about 100 words. Do not sign your own name at the end of the letter. Use "postgraduate association" instead.

Part B

52. Directions:

Write an essay of 160-200 words based on the following drawing. In your essay, you should

1) describe the drawing briefly,

2) explain its intended meaning, and then

3) give your comments.

You should write neatly on ANSHWER SHEET 2. (20 points)

2010年全国硕士研究生入学统一考试英语试题答案

Section I: Use of English (10 points)

1. A

2. B

3. C

4. B

5. C

6. B

7. D

8. A

9. C

10. D

11. C

12. A

13. A

14. D

15. B

16. A

17. D

18. C

19. B

20. D

Section II: Reading Comprehension (60 points)

Part A (40 points)

21. B

22. C

23. D

24. A

25. A

26. D

27. D

28. C

29. B

30. A

31. B

32. D

33. A

34. C

35. C

36. A

37. D

38. C

39. B

40. A

Part B (10 points)

41. B

42. F

43. D

44. G

45. A

Part C (10 points)   

46.科学家们提出一些明显站不住脚的证据迅速来拯救,其大意是:如果鸟类无法控制害虫,那么这些害虫就会吃光我们人类。

47. 但我们至少几乎也承认这样一点:不管鸟类是否给我们带来经济上的好处,但鸟类作为生物其固有的权利应该继续存在。

48. 曾几何时,生物学家们有点过度使用这个证据,即这些物种通过杀死体质弱者来保持猎物的正常繁衍或 者这些生物捕杀的仅仅是毫无价值的物种。

49. 在欧洲,林业在生态方面更加发达,无商业价值的树种被看作是原生森林群落的一部分,而得到合理的保护。  

50. 这一系统易于忽视,因而最终会消除掉这个土地共同体里的许多要素(成员),虽然这些要素(成员)缺乏商业价值,但这些要素(成员)对这个共同体的健康运行来说是必要的。

Section III: Writing (30 points)

Part A (10 points)

51. 参考范文

  Volunteers Wanted/Needed

  An international conference on globalization will be held in the coming winter vacation. This conference will be organized by the Postgraduates Association

  At present we will recruit 10 volunteers to work as assistants for this conference. The applicants are required to speak English fluently. Those who can speak another foreign language such as French or Japanese are preferable. In addition to the language skills, those volunteers are expected to be patient, helpful, open-minded with a loving heart. The volunteers will be provided free three meals a day as well as transportation from and back to the conference site

  Those who are interested in working as volunteers, please send your resume and application letter to the e-mail address "> . The Postgraduates Association will contact those chosen candidates for an interview before Febulary, 1th

  We are sincerely waiting for you to join us!

  Postgraduates Association

Part B (20 points)

52. 参考范文

  As is vividly depicted in the drawing, in the middle of the cartoon stands a hot pot, containing many Chinese cultural symbols, such as Beijing operas, Daoism, and some foreigh cultural symbols, etc. How impressive the drawing is in describing the common phenomenon that Chinese culture is becoming increasingly integrated into the world. The drawers intention seems to be highly self-evident and the meaning causes us to be thought-provoking

  It holds to be apparent that the cartoon is indicative of a pervasive phenomenon with regard to culture. When it comes to(一谈到) cultureits great impacts and benefits cant be too estimated. As China opens to the outside world, our traditional culture is embracing the foreign culture, thus making our Chinese culture more diversified, colorful and internationalized just like a melting pot. What the picture conveys goes far beyond this. The fact that people from different countries are attracted to each other, indicating that to some extent different cultures can be acceptedrespectedappreciated and shared internationally. Or put it in another way, Chinese unique culture can become international through worldwide cultural exchanges. Since the trend of globalization become irresistible, cultural integration can effectively improve mutual understanding and friendship among different countries.

  In my personal sense, Chinese national culture as priceless spiritual treasure should be preserved and cherished. Meanwhilethere are good reasons to embrace foreign cultures on the ground that those ideas from other cultures can provide different perspectives for us to observe the world in the long run. Howeverconfronted with a different culturewe should be sensible enough to absorb its essence and to resist its dark side. Only in this way can we promote cultural integration positively, thus making our motherland dimensional, colorful and vigorous. (298 words)

2009年全国硕士研究生入学统一考试

英语试题

Section I Use of English

Directions:

Research on animal intelligence always makes me wonder just how smart humans are. 1 the fruit-fly experiments described in Carl Zimmer’s piece in the Science Times on Tuesday. Fruit flies who were taught to be smarter than the average fruit fly 2 to live shorter lives. This suggests that 3 bulbs burn longer, that there is an 4 in not being too terrifically bright.

Intelligence, it 5 out, is a high-priced option. It takes more upkeep, burns more fuel and is slow 6 the starting line because it depends on learning — a gradual 7 — instead of instinct. Plenty of other species are able to learn, and one of the things they’ve apparently learned is when to 8 .

Is there an adaptive value to 9 intelligence? That’s the question behind this new research. I like it. Instead of casting a wistful glance 10 at all the species we’ve left in the dust I.Q.-wise, it implicitly asks what the real 11 of our own intelligence might be. This is 12 the mind of every animal I’ve ever met.

Research on animal intelligence also makes me wonder what experiments animals would 13 on humans if they had the chance. Every cat with an owner, 14 , is running a small-scale study in operant conditioning. we believe that 15 animals ran the labs, they would test us to 16 the limits of our patience, our faithfulness, our memory for terrain. They would try to decide what intelligence in humans is really 17 , not merely how much of it there is. 18 , they would hope to study a 19 question: Are humans actually aware of the world they live in? 20 the results are inconclusive.

1. [A] Suppose [B] Consider [C] Observe [D] Imagine

2. [A] tended [B] feared [C] happened [D] threatened

3. [A] thinner [B] stabler [C] lighter [D] dimmer

4. [A] tendency [B] advantage [C] inclination [D] priority

5. [A] insists on [B] sums up [C] turns out [D] puts forward

6. [A] off [B] behind [C] over [D] along

7. [A] incredible [B] spontaneous [C]inevitable [D] gradual

8. [A] fight [B] doubt [C] stop [D] think

9. [A] invisible [B] limited [C] indefinite [D] different

10.[A] upward [B] forward [C] afterward [D] backward

11. [A] features [B] influences [C] results [D] costs

12. [A] outside [B] on [C] by [D] across

13. [A] deliver [B] carry [C] perform [D] apply

14. [A] by chance [B] in contrast [C] as usual [D] for instance

15. [A] if [B] unless [C] as [D] lest

16. [A] moderate [B] overcome [C] determine [D] reach

17. [A] at [B] for [C] after [D] with

18. [A] Above all [B] After all [C] However [D] Otherwise

19. [A] fundamental [B] comprehensive [C] equivalent [D] hostile

20. [A] By accident [B] In time [C] So far [D] Better still

Section II Reading Comprehension

Part A

Directions:

Read the following four texts. Answer the questions below each text by choosing A, B, C or D. Mark your answers on ANSWER SHEET 1. (40 points)

Text1

Habits are a funny thing. We reach for them mindlessly, setting our brains on auto-pilot and relaxing into the unconscious comfort of familiar routine. “Not choice, but habit rules the unreflecting herd,” William Wordsworth said in the 19th century. In the ever-changing 21st century, even the word “habit” carries a negative connotation.

So it seems antithetical to talk about habits in the same context as creativity and innovation. But brain researchers have discovered that when we consciously develop new habits, we create parallel synaptic paths, and even entirely new brain cells, that can jump our trains of thought onto new, innovative tracks.

But don’t bother trying to kill off old habits; once those ruts of procedure are worn into the hippocampus, they’re there to stay. Instead, the new habits we deliberately ingrain into ourselves create parallel pathways that can bypass those old roads.

“The first thing needed for innovation is a fascination with wonder,” says Dawna Markova, author of “The Open Mind” and an executive change consultant for Professional Thinking Partners. “But we are taught instead to ‘decide,’ just as our president calls himself ‘the Decider.’ ” She adds, however, that “to decide is to kill off all possibilities but one. A good innovational thinker is always exploring the many other possibilities.”

All of us work through problems in ways of which we’re unaware, she says. Researchers in the late 1960 covered that humans are born with the capacity to approach challenges in four primary ways: analytically, procedurally, relationally (or collaboratively) and innovatively. At puberty, however, the brain shuts down half of that capacity, preserving only those modes of thought that have seemed most valuable during the first decade or so of life.

The current emphasis on standardized testing highlights analysis and procedure, meaning that few of us inherently use our innovative and collaborative modes of thought. “This breaks the major rule in the American belief system — that anyone can do anything,” explains M. J. Ryan, author of the 2006 book “This Year I Will...” and Ms. Markova’s business partner. “That’s a lie that we have perpetuated, and it fosters commonness. Knowing what you’re good at and doing even more of it creates excellence.” This is where developing new habits comes in.

21. The view of Wordsworth habit is claimed by being

A. casual B. familiar C. mechanical D. changeable.

22. The researchers have discovered that the formation of habit can be

A. predicted B. regulated C. traced D. guided

23.ruts(in line one, paragraph 3) has closest meaning to

A. tracks B. series C. characteristics D. connections

24. Ms. Markova’s comments suggest that the practice of standard testing ?

A, prevents new habits form being formed B, no longer emphasizes commonness

C, maintains the inherent American thinking model D, complies with the American belief system

25. Ryan most probably agree that

A. ideas are born of a relaxing mind B. innovativeness could be taught

C. decisiveness derives from fantastic ideas D. curiosity activates creative minds

Text 2

It is a wise father that knows his own child, but today a man can boost his paternal (fatherly) wisdom or at least confirm that hes the kids dad. All he needs to do is shell our $30 for paternity testing kit (PTK) at his local drugstore and another $120 to get the results.

More than 60,000 people have purchased the PTKs since they first become available without prescriptions last years, according to Doug Fog, chief operating officer of Identigene, which makes the over-the-counter kits. More than two dozen companies sell DNA tests Directly to the public , ranging in price from a few hundred dollars to more than $2500.

Among the most popular : paternity and kinship testing , which adopted children can use to find their biological relatives and latest rage a many passionate genealogists-and supports businesses that offer to search for a familys geographic roots .

Most tests require collecting cells by webbing saliva in the mouth and sending it to the company for testing. All tests require a potential candidate with whom to compare DNA.

But some observers are skeptical,There is a kind of false precision being hawked by people claiming they are doing ancestry testing, says Trey Duster, a New York University sociologist. He notes that each individual has many ancestors-numbering in the hundreds just a few centuries back. Yet most ancestry testing only considers a single lineage, either the Y chromosome inherited through men in a fathers line or mitochondrial DNA, which a passed down only from mothers. This DNA can reveal genetic information about only one or two ancestors, even though, for example, just three generations back people also have six other great-grandparents or, four generations back, 14 other great-great-grandparents.

Critics also argue that commercial genetic testing is only as good as the reference collections to which a sample is compared. Databases used by some companies dont rely on data collected systematically but rather lump together information from different research projects. This means that a DNA database may differ depending on the company that processes the results. In addition, the computer programs a company uses to estimate relationships may be patented and not subject to peer review or outside evaluation.

26.In paragraphs 1 and 2 , the text shows PTKs ___________.

[A]easy availability [B]flexibility in pricing [C] successful promotion[D] popularity with households

27. PTK is used to __________.

[A] locate ones birth place [B] promote genetic research

[C] identify parent-child kinship [D] choose children for adoption

28. Skeptical observers believe that ancestry testing fails to__________.

[A]trace distant ancestors [B]rebuild reliable bloodlines

[C]fully use genetic information [D]achieve the claimed accuracy

29. In the last paragraph ,a problem commercial genetic testing faces is __________.

[A]disorganized data collection [B]overlapping database building

30. An appropriate title for the text is most likely to be__________.

[A] Fors and Againsts of DNA testing [B] DNA testing and Its problems

[C] DNA testing outside the lab [D] lies behind DNA testing

Text 3

The relationship between formal education and economic growth in poor countries is widely misunderstood by economists and politicians alike progress in both area is undoubtedly necessary for the social, political and intellectual development of these and all other societies; however, the conventional view that education should be one of the very highest priorities for promoting rapid economic development in poor countries is wrong. We are fortunate that is it, because new educational systems there and putting enough people through them to improve economic performance would require two or three generations. The findings of a research institution have consistently shown that workers in all countries can be trained on the job to achieve radical higher productivity and, as a result, radically higher standards of living.

Ironically, the first evidence for this idea appeared in the United States. Not long ago, with the country entering a recessing and Japan at its pre-bubble peak. The U.S. workforce was derided as poorly educated and one of primary cause of the poor U.S. economic performance. Japan was, and remains, the global leader in automotive-assembly productivity. Yet the research revealed that the U.S. factories of Honda Nissan, and Toyota achieved about 95 percent of the productivity of their Japanese countere pants a result of the training that U.S. workers received on the job.

More recently, while examing housing construction, the researchers discovered that illiterate, non-English- speaking Mexican workers in Houston, Texas, consistently met best-practice labor productivity standards despite the complexity of the building industry’s work.

What is the real relationship between education and economic development? We have to suspect that continuing economic growth promotes the development of education even when governments don’t force it. After all, that’s how education got started. When our ancestors were hunters and gatherers 10,000 years ago, they didn’t have time to wonder much about anything besides finding food. Only when humanity began to get its food in a more productive way was there time for other things.

As education improved, humanity’s productivity potential, they could in turn afford more education. This increasingly high level of education is probably a necessary, but not a sufficient, condition for the complex political systems required by advanced economic performance. Thus poor countries might not be able to escape their poverty traps without political changes that may be possible only with broader formal education. A lack of formal education, however, doesn’t constrain the ability of the developing world’s workforce to substantially improve productivity for the forested future. On the contrary, constraints on improving productivity explain why education isn’t developing more quickly there than it is.

31. The author holds in paragraph 1 that the important of education in poor countries ___________.

[A] is subject groundless doubts [B] has fallen victim of bias

[C] is conventional downgraded [D] has been overestimated

32. It is stated in paragraph 1 that construction of a new education system __________.

[A]challenges economists and politicians [B]takes efforts of generations

[C] demands priority from the government [D] requires sufficient labor force

33.A major difference between the Japanese and U.S workforces is that __________.

[A] the Japanese workforce is better disciplined [B] the Japanese workforce is more productive

[C]the U.S workforce has a better education [D] ]the U.S workforce is more organize

34. The author quotes the example of our ancestors to show that education emerged __________.

[A] when people had enough time [B] prior to better ways of finding food

[C] when people on longer went hung [D] as a result of pressure on government

35. According to the last paragraph , development of education __________.

[A] results directly from competitive environments [B] does not depend on economic performance

[C] follows improved productivity [D] cannot afford political changes

Text 4

The most thoroughly studied in the history of the new world are the ministers and political leaders of seventeenth-century New England. According to the standard history of American philosophy, nowhere else in colonial America was So much important attached to intellectual pursuits According to many books and articles, New Englands leaders established the basic themes and preoccupations of an unfolding, dominant Puritan tradition in American intellectual life.

To take this approach to the New Englanders normally mean to start with the Puritans’ theological innovations and their distinctive ideas about the church-important subjects that we may not neglect. But in keeping with our examination of southern intellectual life, we may consider the original Puritans as carriers of European culture adjusting to New world circumstances. The New England colonies were the scenes of important episodes in the pursuit of widely understood ideals of civility and virtuosity.

The early settlers of Massachusetts Bay included men of impressive education and influence in England. `Besides the ninety or so learned ministers who came to Massachusetts church in the decade after 1629,There were political leaders like John Winthrop, an educated gentleman, lawyer, and official of the Crown before he journeyed to Boston. There men wrote and published extensively, reaching both New World and Old World audiences, and giving New England an atmosphere of intellectual earnestness.

We should not forget , however, that most New Englanders were less well educated. While few crafts men or farmers, let alone dependents and servants, left literary compositions to be analyzed, The in thinking often had a traditional superstitions quality. A tailor named John Dane, who emigrated in the late 1630s, left an account of his reasons for leaving England that is filled with signs. sexual confusion, economic frustrations , and religious hope-all name together in a decisive moment when he opened the Bible, told his father the first line he saw would settle his fate, and read the magical words:come out from among them, touch no unclean thing , and I will be your God and you shall be my people. One wonders what Dane thought of the careful sermons explaining the Bible that he heard in puritan churched.

Mean while , many settles had slighter religious commitments than Danes, as one clergyman learned in confronting folk along the coast who mocked that they had not come to the New world for religion . Our main end was to catch fish.

36. The author notes that in the seventeenth-century New England___________.

[A] Puritan tradition dominated political life. [B] intellectual interests were encouraged.

[C] Politics benefited much from intellectual endeavors.

[D] intellectual pursuits enjoyed a liberal environment.

37. It is suggested in paragraph 2 that New Englanders__________.

[A] experienced a comparatively peaceful early history.

[B] brought with them the culture of the Old World

[C] paid little attention to southern intellectual life[D] were obsessed with religious innovations

38. The early ministers and political leaders in Massachusetts Bay__________.

[A] were famous in the New World for their writings

[B] gained increasing importance in religious affairs

[C] abandoned high positions before coming to the New World

[D] created a new intellectual atmosphere in New England

39. The story of John Dane shows that less well-educated New Englanders were often __________.

[A] influenced by superstitions [B] troubled with religious beliefs

[C] puzzled by church sermons [D] frustrated with family earnings

40. The text suggests that early settlers in New England__________.

[A] were mostly engaged in political activities[B] were motivated by an illusory prospect

[C] came from different backgrounds. [D] left few formal records for later reference

Part B

Directions:

Directions: In the following text, some sentences have been removed. For Questions (41-45), choose the most suitable one from the list A-G to fit into each of the numbered blank. There are two extra choices, which do not fit in any of the gaps. Mark your answers on ANSWER SHEET 1. (10 points)

Coinciding with the groundbreaking theory of biological evolution proposed by British naturalist Charles Darwin in the 1860s, British social philosopher Herbert Spencer put forward his own theory of biological and cultural evolution. Spencer argued that all worldly phenomena, including human societies, changed over time, advancing toward perfection. 41.____________.

American social scientist Lewis Henry Morgan introduced another theory of cultural evolution in the late 1800s. Morgan, along with Tylor, was one of the founders of modern anthropology. In his work, he attempted to show how all aspects of culture changed together in the evolution of societies.42._____________.

In the early 1900s in North America, German-born American anthropologist Franz Boas developed a new theory of culture known as historical particularism. Historical particularism, which emphasized the uniqueness of all cultures, gave new direction to anthropology. 43._____________ .

Boas felt that the culture of any society must be understood as the result of a unique history and not as one of many cultures belonging to a broader evolutionary stage or type of culture. 44._______________.

Historical particularism became a dominant approach to the study of culture in American anthropology, largely through the influence of many students of Boas. But a number of anthropologists in the early 1900s also rejected the particularist theory of culture in favor of diffusionism. Some attributed virtually every important cultural achievement to the inventions of a few, especially gifted peoples that, according to diffusionists, then spread to other cultures. 45.________________.

Also in the early 1900s, French sociologist Émile Durkheim developed a theory of culture that would greatly influence anthropology. Durkheim proposed that religious beliefs functioned to reinforce social solidarity. An interest in the relationship between the function of society and culture—known as functionalism—became a major theme in European, and especially British, anthropology.

[A] Other anthropologists believed that cultural innovations, such as inventions, had a single origin and passed from society to society. This theory was known as diffusionism.

[B] In order to study particular cultures as completely as possible, Boas became skilled in linguistics, the study of languages, and in physical anthropology, the study of human biology and anatomy.

[C] He argued that human evolution was characterized by a struggle he called the “survival of the fittest,” in which weaker races and societies must eventually be replaced by stronger, more advanced races and societies.

[D] They also focused on important rituals that appeared to preserve a people’s social structure, such as initiation ceremonies that formally signify children’s entrance into adulthood.

[E] Thus, in his view, diverse aspects of culture, such as the structure of families, forms of marriage, categories of kinship, ownership of property, forms of government, technology, and systems of food production, all changed as societies evolved.

[F]Supporters of the theory viewed as a collection of integrated parts that work together to keep a society functioning.

[G] For example, British anthropologists Grafton Elliot Smith and W. J. Perry incorrectly suggested, on the basis of inadequate information, that farming, pottery making, and metallurgy all originated in ancient Egypt and diffused throughout the world. In fact, all of these cultural developments occurred separately at different times in many parts of the world.

Part C

Directions:

Read the following text carefully and then translate the underlined segments into Chinese. Your translation should be written carefully on ANSWER SHEET 2. (10 points)

There is a marked difference between the education which every one gets from living with others, and the deliberate educating of the young. In the former case the education is incidental; it is natural and important, but it is not the express reason of the association.46 It may be said that the measure of the worth of any social institution is its effect in enlarging and improving experience; but this effect is not a part of its original motive. Religious associations began, for example, in the desire to secure the favor of overruling powers and to ward off evil influences; family life in the desire to gratify appetites and secure family perpetuity; systematic labor, for the most part, because of enslavement to others, etc. 47Only gradually was the by-product of the institution noted, and only more gradually still was this effect considered as a directive factor in the conduct of the institution. Even today, in our industrial life, apart from certain values of industriousness and thrift, the intellectual and emotional reaction of the forms of human association under which the world's work is carried on receives little attention as compared with physical output.

But in dealing with the young, the fact of association itself as an immediate human fact, gains in importance.48 While it is easy to ignore in our contact with them the effect of our acts upon their disposition, it is not so easy as in dealing with adults. The need of training is too evident; the pressure to accomplish a change in their attitude and habits is too urgent to leave these consequences wholly out of account. 49Since our chief business with them is to enable them to share in a common life we cannot help considering whether or no we are forming the powers which will secure this ability. If humanity has made some headway in realizing that the ultimate value of every institution is its distinctively human effect we may well believe that this lesson has been learned largely through dealings with the young.

50 We are thus led to distinguish, within the broad educational process which we have been so far considering, a more formal kind of education -- that of direct tuition or schooling. In undeveloped social groups, we find very little formal teaching and training. These groups mainly rely for instilling needed dispositions into the young upon the same sort of association which keeps the adults loyal to their group.

Section Writing

Part A

51. Directions: Restrictions on the use of plastic bags have not been so successful in some regions. White pollution is still going on. Write a letter to the editor(s) of your local newspaper to

give your opinions briefly and

make two or three suggestions

You should write about 100 words. Do not sign your own name at the end of the letter. Use "Li Ming" instead. You do not need to write the address.

Part B

52. Directions:

In your essay, you should

1) describe the drawing briefly,

2) explain its intended meaning, and then

3) give your comments.

You should write neatly on ANSHWER SHEET 2. (20 points)

51:在某些地区塑料袋的禁止使用并不是很成功,因此白色污染仍然继续

  给当地报纸的编辑写一封信,信内包括

  1 给出自己简短的看法和观点

  2 给出2-3条建议

  【范文】

  Dear Editors,

  I, as your faithful reader, am writing this letter to suggest that plastic bags should be restricted in our daily life. However, to my surprise and sadness, I have found that disposable plastic bags are still widely used in some areas just because people are unwilling to change their old habits, thus causing their surroundings to be even worse.

  To solve this serious problem, I would like to put forward a couple of practical suggestions. Above all, our country should establish a strong agency to limit the production of those bags and monitor the use of them. Furthermore, some other choices should be adovcated to replace plastic bags with paper or bamboo ones. Finally, consumers should pay for the use of plastic bags so as to enhance their consciousness of environmental protection.

  I really hope my suggestions would attract your due attention and receive an early reply.

  Yours Sincerely,

  Li Ming

  52:用网络使我们看不见的东西能听见,远隔千山万水但能联系,意思就是网络的远与近

  大作文是:网络的近与远

  一 客观描述图 说明意思 给出观点

  【范文】

  As is vividly depicted in the drawing, in front of computers and in narrow spaces are sitting many people, exchanging their views with each other by surfing the Internet. How impressive the drawing is in describing the people’s addiction to the Internet. The drawer’s intention seems to be highly self-evident and the meaning causes us to be thought-provoking.

  It holds apparent that the cartoon is indicative of a pervasive problem with regard to Internet. When it comes to(一谈到) Internetits great impacts and benefits can’t be too estimated. It is the Internet that makes our big world become a global village. However, as a growing number of individuals are addicted to (沉溺于) the Internet, they forget doing other important work and are gradually indulged in(沉溺在) the virtual world, unwilling to go into the real society. As a consequence(因此), they close their doors and never go out as soon as they return home from work, reluctant to have any face-to-face connection even with other folks except on line. What’s worse, they become indifferent(漠不关心)to their friends, neighbours, as well as relatives.

  This phenomenon is harmful to us and our community too. As the pressure in life and work increases, we should learn to use proper ways to relieve it. It’s time that we took some measures to improve the situation. People can be organized to hold some activities together to develop some good hobbies. Consequently, we must make full use of Internet to do everything beneficial to make our daily life both joyful and meaningful. 272 words

2008年全国硕士研究生入学统一考试英语试题

Section I Use of English

Directions:

The idea that some groups of people may be more intelligent than others is one of those hypotheses that dare not speak its name. But Gregory Cochran is to say it anyway. He is that bird, a scientist who works independently any institution. He helped popularize the idea that some diseases not thought to have a bacterial cause were actually infections, which aroused much controversy when it was first suggested.

he, however, might tremble at the of what he is about to do. Together with another two scientists, he is publishing a paper which not only that one group of humanity is more intelligent than the others, but explains the process that has brought this about. The group in are a particular people originated from central Europe. The process is natural selection.

This group generally do well in IQ test, 12-15 points above the value of 100, and have contributed to the intellectual and cultural life of the West, as the of their elites, including several world-renowned scientists, . They also suffer more often than most people from a number of nasty genetic diseases, such as breast cancer. These facts, , have previously been thought unrelated. The former has been to social effects, such as a strong tradition of education. The latter was seen as a (an) of genetic isolation. Dr. Cochran suggests that the intelligence and diseases are intimately . His argument is that the unusual history of these people has them to unique evolutionary pressures that have resulted in this state of affairs.

1. [A] selected [B] prepared [C] obliged [D] pleased

2. [A] unique [B] particular [C] special [D] rare

3. [A] of [B] with [C] in [D] against

4. [A] subsequently[B] presently [C] previously [D] lately

5. [A] Only [B] So [C] Even [D] Hence

6. [A] thought [B] sight [C] cost [D] risk

7. [A] advises [B] suggests [C] protests [D] objects

8. [A] progress [B] fact [C] need [D] question

9. [A] attaining [B] scoring [C] reaching [D] calculating

10. [A] normal [B] common [C] mean [D] total

11. [A] unconsciously [B] disproportionately

[C] indefinitely [D] unaccountably

12. [A] missions [B] fortunes [C] interests [D] careers

13. [A] affirm [B] witness [C] observe [D] approve

14. [A] moreover [B] therefore [C] however [D] meanwhile

15. [A] given up [B] got over [C] carried on [D] put down

16. [A] assessing [B] supervising [C] administering [D] valuing

17. [A] development [B] origin [C] consequence [D] instrument

18. [A] linked [B] integrated [C] woven [D] combined

19. [A] limited [B] subjected [C] converted [D] directed

20. [A] paradoxical [B] incompatible [C] inevitable [D] continuous

Section II Reading Comprehension

Part A

Directions:

Read the following four texts. Answer the questions below each text by choosing A, B, C or D. Mark your answers on ANSWER SHEET 1. (40 points)

Text 1

While still catching-up to men in some spheres of modern life, women appear to be way ahead in at least one undesirable category. Women are particularly susceptible to developing depression and anxiety disorders in response to stress compared to men, according to Dr. Yehuda, chief psychiatrist at New Yorks Veterans Administration Hospital.

Studies of both animals and humans have shown that sex hormones somehow affect the stress response, causing females under stress to produce more of the trigger chemicals than do males under the same conditions. In several of the studies, when stressed-out female rats had their ovaries (the female reproductive organs) removed, their chemical responses became equal to those of the males.

Adding to a womans increased dose of stress chemicals, are her increased opportunities for stress. Its not necessarily that women dont cope as well. Its just that they have so much more to cope with, says Dr. Yehuda. Their capacity for tolerating stress may even be greater than mens, she observes, its just that theyre dealing with so many more things that they become worn out from it more visibly and sooner.

Dr. Yehuda notes another difference between the sexes. I think that the kinds of things that women are exposed to tend to be in more of a chronic or repeated nature. Men go to war and are exposed to combat stress. Men are exposed to more acts of random physical violence. The kinds of interpersonal violence that women are exposed to tend to be in domestic situations, by, unfortunately, parents or other family members, and they tend not to be one-shot deals. The wear-and-tear that comes from these longer relationships can be quite devastating.

Adeline Alvarez married at 18 and gave birth to a son, but was determined to finish college. I struggled a lot to get the college degree. I was living in so much frustration that that was my escape, to go to school, and get ahead and do better. Later, her marriage ended and she became a single mother. Its the hardest thing to take care of a teenager, have a job, pay the rent, pay the car payment, and pay the debt. I lived from paycheck to paycheck.

Not everyone experiences the kinds of severe chronic stresses Alvarez describes. But most women today are coping with a lot of obligations, with few breaks, and feeling the strain. Alvarezs experience demonstrates the importance of finding ways to diffuse stress before it threatens your health and your ability to function.

21. Which of the following is true according to the first two paragraphs?

[A] Women are biologically more vulnerable to stress.

[B] Women are still suffering much stress caused by men.

[C] Women are more experienced than men in coping with stress.

[D] Men and women show different inclinations when faced with stress.

22. Dr. Yehudas research suggests that women

[A] need extra doses of chemicals to handle stress.

[B] have limited capacity for tolerating stress.

[C] are more capable of avoiding stress. [D] are exposed to more stress.

23. According to Paragraph 4, the stress women confront tends to be

[A] domestic and temporary. [B] irregular and violent.

[C] durable and frequent. [D] trivial and random.

24. The sentence I lived from paycheck to paycheck. (Line 6, Para. 5) shows that

[A] Alvarez cared about nothing but making money.

[B] Alvarezs salary barely covered her household expenses.

[C] Alvarez got paychecks from different jobs.

[D] Alvarez paid practically everything by check.

25. Which of the following would be the best title for the text?

[A] Strain of Stress: No Way Out? [B] Responses to Stress: Gender Difference

[C] Stress Analysis: What Chemicals Say [D] Gender Inequality: Women Under Stress

Text 2

It used to be so straightforward. A team of researchers working together in the laboratory would submit the results of their research to a journal. A journal editor would then remove the authors names and affiliations from the paper and send it to their peers for review. Depending on the comments received, the editor would accept the paper for publication or decline it. Copyright rested with the journal publisher, and researchers seeking knowledge of the results would have to subscribe to the journal.

No longer. The Internet and pressure from funding agencies, who are questioning why commercial publishers are making money from government-funded research by restricting access to it is making access to scientific results a reality. The Organization for Economic Co-operation and Development (OECD) has just issued a report describing the far-reaching consequences of this. The report, by John Houghton of Victoria University in Australia and Graham Vickery of the OECD, makes heavy reading for publishers who have, so far, made handsome profits. But it goes further than that. It signals a change in what has, until now, been a key element of scientific endeavor.

The value of knowledge and the return on the public investment in research depends, in part, upon wide distribution and ready access. It is big business. In America, the core scientific publishing market is estimated at between $7 billion and $11 billion. The International Association of Scientific, Technical and Medical Publishers says that there are more than 2,000 publishers worldwide specializing in these subjects. They publish more than 1.2 million articles each year in some 16,000 journals.

This is now changing. According to the OECD report, some 75% of scholarly journals are now online. Entirely new business models are emerging; three main ones were identified by the reports authors. There is the so-called big deal, where institutional subscribers pay for access to a collection of online journal titles through site-licensing agreements. There is open-access publishing, typically supported by asking the author (or his employer) to pay for the paper to be published. Finally, there are open-access archives, where organizations such as universities or international laboratories support institutional repositories. Other models exist that are hybrids of these three, such as delayed open-access, where journals allow only subscribers to read a paper for the first six months, before making it freely available to everyone who wishes to see it. All this could change the traditional form of the peer-review process, at least for the publication of papers.

26. In the first paragraph, the author discusses

[A] the background information of journal editing.[B] the publication routine of laboratory reports.

[C] the relations of authors with journal publishers.[D] the traditional process of journal publication.

27. Which of the following is true of the OECD report?

[A] It criticizes government-funded research.[B] It introduces an effective means of publication.

[C] It upsets profit-making journal publishers.[D] It benefits scientific research considerably.

28. According to the text, online publication is significant in that

[A] it provides an easier access to scientific results.[B] it brings huge profits to scientific researchers.

[C] it emphasizes the crucial role of scientific knowledge.

[D] it facilitates public investment in scientific research.

29. With the open-access publishing model, the author of a paper is required to

[A] cover the cost of its publication.[B] subscribe to the journal publishing it.

[C] allow other online journals to use it freely.[D] complete the peer-review before submission.

30. Which of the following best summarizes the main idea of the text?

[A] The Internet is posing a threat to publishers.[B] A new mode of publication is emerging.

[C] Authors welcome the new channel for publication.

[D] Publication is rendered easier by online service.

Text 3

In the early 1960s Wilt Chamberlain was one of only three players in the National Basketball Association (NBA) listed at over seven feet. If he had played last season, however, he would have been one of 42. The bodies playing major professional sports have changed dramatically over the years, and managers have been more than willing to adjust team uniforms to fit the growing numbers of bigger, longer frames.

The trend in sports, though, may be obscuring an unrecognized reality: Americans have generally stopped growing. Though typically about two inches taller now than 140 years ago, todays people especially those born to families who have lived in the U.S. for many generations apparently reached their limit in the early 1960s. And they arent likely to get any taller. In the general population today, at this genetic, environmental level, weve pretty much gone as far as we can go, says anthropologist William Cameron Chumlea of Wright State University. In the case of NBA players, their increase in height appears to result from the increasingly common practice of recruiting players from all over the world.

Growth, which rarely continues beyond the age of 20, demands calories and nutrients – notably, protein to feed expanding tissues. At the start of the 20th century, under-nutrition and childhood infections got in the way. But as diet and health improved, children and adolescents have, on average, increased in height by about an inch and a half every 20 years, a pattern known as the secular trend in height. Yet according to the Centers for Disease Control and Prevention, average height 59 for men, 54 for women hasnt really changed since 1960.

Genetically speaking, there are advantages to avoiding substantial height. During childbirth, larger babies have more difficulty passing through the birth canal. Moreover, even though humans have been upright for millions of years, our feet and back continue to struggle with bipedal posture and cannot easily withstand repeated strain imposed by oversize limbs. There are some real constraints that are set by the genetic architecture of the individual organism, says anthropologist William Leonard of Northwestern University.

Genetic maximums can change, but dont expect this to happen soon. Claire C. Gordon, senior anthropologist at the Army Research Center in Natick, Mass., ensures that 90 percent of the uniforms and workstations fit recruits without alteration. She says that, unlike those for basketball, the length of military uniforms has not changed for some time. And if you need to predict human height in the near future to design a piece of equipment, Gordon says that by and large, you could use todays data and feel fairly confident.

31. Wilt Chamberlain is cited as an example to

[A] illustrate the change of height of NBA players.

[B] show the popularity of NBA players in the U.S..

[C] compare different generations of NBA players.

[D] assess the achievements of famous NBA players.

32. Which of the following plays a key role in body growth according to the text?

[A] Genetic modification. [B] Natural environment.

[C] Living standards. [D] Daily exercise.

33. On which of the following statements would the author most probably agree?

[A] Non-Americans add to the average height of the nation.

[B] Human height is conditioned by the upright posture.

[C] Americans are the tallest on average in the world.

[D] Larger babies tend to become taller in adulthood.

34. We learn from the last paragraph that in the near future

[A] the garment industry will reconsider the uniform size.

[B] the design of military uniforms will remain unchanged.

[C] genetic testing will be employed in selecting sportsmen.

[D] the existing data of human height will still be applicable.

35. The text intends to tell us that

[A] the change of human height follows a cyclic pattern.

[B] human height is becoming even more predictable.

[C] Americans have reached their genetic growth limit.

[D] the genetic pattern of Americans has altered.

Text 4

In 1784, five years before he became president of the United States, George Washington, 52, was nearly toothless. So he hired a dentist to transplant nine teeth into his jaw having extracted them from the mouths of his slaves.

Thats a far different image from the cherry-tree-chopping George most people remember from their history books. But recently, many historians have begun to focus on the roles slavery played in the lives of the founding generation. They have been spurred in part by DNA evidence made available in 1998, which almost certainly proved Thomas Jefferson had fathered at least one child with his slave Sally Hemings. And only over the past 30 years have scholars examined history from the bottom up. Works of several historians reveal the moral compromises made by the nations early leaders and the fragile nature of the countrys infancy. More significantly, they argue that many of the Founding Fathers knew slavery was wrong and yet most did little to fight it.

More than anything, the historians say, the founders were hampered by the culture of their time. While Washington and Jefferson privately expressed distaste for slavery, they also understood that it was part of the political and economic bedrock of the country they helped to create.

For one thing, the South could not afford to part with its slaves. Owning slaves was “like having a large bank account,” says Wiencek, author of An Imperfect God: George Washington, His Slaves, and the Creation of America. The southern states would not have signed the Constitution without protections for the “peculiar institution, including a clause that counted a slave as three fifths of a man for purposes of congressional representation.

And the statesmens political lives depended on slavery. The three-fifths formula handed Jefferson his narrow victory in the presidential election of 1800 by inflating the votes of the southern states in the Electoral College. Once in office, Jefferson extended slavery with the Louisiana Purchase in 1803; the new land was carved into 13 states, including three slave states.

Still, Jefferson freed Hemingss children though not Hemings herself or his approximately 150 other slaves. Washington, who had begun to believe that all men were created equal after observing the bravery of the black soldiers during the Revolutionary War, overcame the strong opposition of his relatives to grant his slaves their freedom in his will. Only a decade earlier, such an act would have required legislative approval in Virginia.

36. George Washingtons dental surgery is mentioned to

[A] show the primitive medical practice in the past.

[B] demonstrate the cruelty of slavery in his days.

[C] stress the role of slaves in the U.S. history.[D] reveal some unknown aspect of his life.

37. We may infer from the second paragraph that

[A] DNA technology has been widely applied to history research.

[B] in its early days the U.S. was confronted with delicate situations.

[C] historians deliberately made up some stories of Jeffersons life.

[D] political compromises are easily found throughout the U.S. history.

38. What do we learn about Thomas Jefferson?

[A] His political view changed his attitude towards slavery.

[B] His status as a father made him free the child slaves.

[C] His attitude towards slavery was complex.[D] His affair with a slave stained his prestige.

39. Which of the following is true according to the text?

[A] Some Founding Fathers benefit politically from slavery.

[B] Slaves in the old days did not have the right to vote.

[C] Slave owners usually had large savings accounts.

[D] Slavery was regarded as a peculiar institution.

40. Washingtons decision to free slaves originated from his

[A] moral considerations. [B] military experience.[C] financial conditions. [D] political stand.

Part B

Directions:

In the following article, some sentences have been removed. For Questions 41—45, choose the most suitable one from the list A-G to fit into each of the numbered blanks. There are two extra choices, which do not fit in any of the blanks. Mark your answers on ANSWER SHEET 1. (10 points)

The time for sharpening pencils, arranging your desk, and doing almost anything else instead of writing has ended. The first draft will appear on the page only if you stop avoiding the inevitable and sit, stand up, or lie down to write. (41)是大家网原创出品

Be flexible. Your outline should smoothly conduct you from one point to the next, but do not permit it to railroad you. If a relevant and important idea occurs to you now, work it into the draft. (42) 是大家网原创出品 Grammar, punctuation, and spelling can wait until you revise. Concentrate on what you are saying. Good writing most often occurs when you are in hot pursuit of an idea rather than in a nervous search for errors.

(43) 是大家网原创出品 Your pages will be easier to keep track of that way, and, if you have to clip a paragraph to place it elsewhere, you will not lose any writing on the other side.

If you are working on a word processor, you can take advantage of its capacity to make additions and deletions as well as move entire paragraphs by making just a few simple keyboard commands. Some software programs can also check spelling and certain grammatical elements in your writing. (44) 是大家网原创出品 These printouts are also easier to read than the screen when you work on revisions.

Once you have a first draft on paper, you can delete material that is unrelated to your thesis and add material necessary to illustrate your points and make your paper convincing. The student who wrote The A & P as a State of Mind wisely dropped a paragraph that questioned whether Sammy displays chauvinistic attitudes toward women. (45) 是大家网原创出品

Remember that your initial draft is only that. You should go through the paper many times and then again working to substantiate and clarify your ideas. You may even end up with several entire versions of the paper. Rewrite. The sentences within each paragraph should be related to a single topic. Transitions should connect one paragraph to the next so that there are no abrupt or confusing shifts. Awkward or wordy phrasing or unclear sentences and paragraphs should be mercilessly poked and prodded into shape.

[A] To make revising easier, leave wide margins and extra space between lines so that you can easily add words, sentences, and corrections. Write on only one side of the paper.

[B] After you have clearly and adequately developed the body of your paper, pay particular attention to the introductory and concluding paragraphs. Its probably best to write the introduction last, after you know precisely what you are introducing. Concluding paragraphs demand equal attention because they leave the reader with a final impression.

[C] Its worth remembering, however, that though a clean copy fresh off a printer may look terrific, it will read only as well as the thinking and writing that have gone into it. Many writers prudently store their data on disks and print their pages each time they finish a draft to avoid losing any material because of power failures or other problems.

[D] It makes no difference how you write, just so you do. Now that you have developed a topic into a tentative thesis, you can assemble your notes and begin to flesh out whatever outline you have made.

[E] Although this is an interesting issue, it has nothing to do with the thesis, which explains how the setting influences Sammys decision to quit his job. Instead of including that paragraph, she added one that described Lengels crabbed response to the girls so that she could lead up to the A & P policy he enforces.

[F] In the final paragraph about the significance of the setting in A & P, the student brings together the reasons Sammy quit his job by referring to his refusal to accept Lengels store policies.

[G] By using the first draft as a means of thinking about what you want to say, you will very likely discover more than your notes originally suggested. Plenty of good writers dont use outlines at all but discover ordering principles as they write. Do not attempt to compose a perfectly correct draft the first time around.

Part C

Directions:

Read the following text carefully and then translate the underlined segments into Chinese. Your translation should be written clearly on ANSWER SHEET 2. (10 points)

In his autobiography, Darwin himself speaks of his intellectual powers with extraordinary modesty. He points out that he always experienced much difficulty in expressing himself clearly and concisely, but (46) he believes that this very difficulty may have had the compensating advantage of forcing him to think long and intently about every sentence, and thus enabling him to detect errors in reasoning and in his own observations. He disclaimed the possession of any great quickness of apprehension or wit, such as distinguished Huxley. (47) He asserted, also, that his power to follow a long and purely abstract train of thought was very limited, for which reason he felt certain that he never could have succeeded with mathematics. His memory, too, he described as extensive, but hazy. So poor in one sense was it that he never could remember for more than a few days a single date or a line of poetry. (48) On the other hand, he did not accept as well founded the charge made by some of his critics that, while he was a good observer, he had no power of reasoning. This, he thought, could not be true, because the Origin of Species is one long argument from the beginning to the end, and has convinced many able men. No one, he submits, could have written it without possessing some power of reasoning. He was willing to assert that I have a fair share of invention, and of common sense or judgment, such as every fairly successful lawyer or doctor must have, but not, I believe, in any higher degree. (49) He adds humbly that perhaps he was superior to the common run of men in noticing things which easily escape attention, and in observing them carefully.

Writing in the last year of his life, he expressed the opinion that in two or three respects his mind had changed during the preceding twenty or thirty years. Up to the age of thirty or beyond it poetry of many kinds gave him great pleasure. Formerly, too, pictures had given him considerable, and music very great, delight. In 1881, however, he said: Now for many years I cannot endure to read a line of poetry. I have also almost lost my taste for pictures or music. (50) Darwin was convinced that the loss of these tastes was not only a loss of happiness, but might possibly be injurious to the intellect, and more probably to the moral character.

Section III Writing

Part A

51. Directions:

You have just come back from Canada and found a music CD in your luggage that you forgot to return to Bob, your landlord there. Write him a letter to

1) make an apology, and

2) suggest a solution.

You should write about 100 words on ANSWER SHEET 2.

Do not sign your own name at the end of the letter. Use Li Ming instead.

Do not write the address. (10 points)

Part B

52. Directions:

Write an essay of 160-200 words based on the following drawing. In your essay, you should

1) describe the drawing briefly,

2) explain its intended meaning, and then

3) give your comments.

You should write neatly on ANSWER SHEET 2. (20 points)

应用文范文

Dear Bob,

I am writing to express my apology to you.

Several days ago, I borrowed your music CD when I lived in your house. Unfortunately, after I came back from Canada, I found it in my luggage. I was in such a hurry that I forgot to return it to you. I will send it to you by post or express as soon as possible. If necessary, I will compensate for any troubles it may cause.

Once again, I feel so sorry for any inconvenience caused. Please accept my apologies.

Sincerely yours,

Li Ming

大作文范文

As is illustrated in the picture, the two disabled persons whose crippled legs are bound together do a lot of traveling. Accordingly, this far-reaching picture reflects a common phenomenon in today’s society: the people who are in the dark want to turn the corner but they can not make it respectively and in turn they have to choose to pull together in times of trouble.

There are several reasons accounting for this. Since we have to live in an on-the-move lifestyle, we may encounter various plights, where we would be at a loss rather than to seek for others’ assistance. Further more, if we do not offer help to each other when we confront dilemma, we would not realize our dream. And no issue in China is as basic to build up the society in harmony as to conduct coordination in face of disasters.

Judging from what have been argued above, people have come to realize the value of mutual aid. It is, therefore, necessary that some effective steps be made to advocate spirit of supporting each other. To begin with, the government should make laws to encourage people to unite. In addition, people should enhance the awareness of caring each other especially when they are in trouble. Only in those ways, can we make people, even not being acquaintance, help each other.

2007年全国硕士研究生入学统一考试英语试题

Section I Use of English

Directions:

Read the following text. Choose the best word(s) for each numbered blank and mark [A], [B], [C] or [D] on ANSWER SHEET 1. (10 points)

By 1830 the former Spanish and Portuguese colonies had become independent nations. The roughly 20 million of these nations looked to the future. Born in the crisis of the old regime and Iberian Colonialism, many of the leaders of independence the ideals of representative government, careers to talent, freedom of commerce and trade, the to private property, and a belief in the individual as the basis of society. there was a belief that the new nations should be sovereign and independent states, large enough to be economically viable and integrated by a set of laws.

On the issue of of religion and the position of the church, , there was less agreement the leadership. Roman Catholicism had been the state religion and the only one by the Spanish crown. most leaders sought to maintain Catholicism the official religion of the new states, some sought to end the of other faiths. The defense of the Church became a rallying for the conservative forces.

The ideals of the early leaders of independence were often egalitarian, valuing equality of everything. Bolivar had received aid from Haiti and had in return to abolish slavery in the areas he liberated. By 1854 slavery had been abolished everywhere except Spains colonies. Early promises to end Indian tribute and taxes on people of mixed origin came much because the new nations still needed the revenue such policies . Egalitarian sentiments were often tempered by fears that the mass of the population was self-rule and democracy.

1. [A] natives [B] inhabitants [C] peoplesB [D] individuals

2. [A] confusedly [B] cheerfully [C] worriedlyD [D] hopefully

3. [A] shared [B] forgot [C] attainedA [D] rejected

4. [A] related [B] close [C] openC [D] devoted

5. [A] access [B] succession [C] rightC [D] return

6. [A] Presumably [B] Incidentally [C] ObviouslyD [D] Generally

7. [A] unique [B] common [C] particularB [D] typical

8. [A] freedom [B] origin [C] impactA [D] reform

9. [A] therefore [B] however [C] indeedB [D] moreover

10. [A] with [B] about [C] amongC [D] by

11. [A] allowed [B] preached [C] grantedA [D] funded

12. [A] Since [B] If [C] UnlessD [D] While

13. [A] as [B] for [C] underA [D] against

14. [A] spread [B] interference [C] exclusionC [D] influence

15. [A] support [B] cry [C] pleaB [D] wish

16. [A] urged [B] intended [C] expectedD [D] promised

17. [A] controlling [B] former [C] remainingC [D] original

18. [A] slower [B] faster [C] easierA [D] tougher

19. [A] created [B] produced [C] contributedB [D] preferred

20. [A] puzzled by[B] hostile to[C] pessimistic aboutD[D] unprepared for

Section II Reading Comprehension

Part A

Directions:

Read the following four texts. Answer the questions below each text by choosing [A], [B], [C], or [D]. Mark your answers on ANSWER SHEET 1. (40 points)

Text 1

If you were to examine the birth certificates of every soccer player in 2006s World Cup tournament, you would most likely find a noteworthy quirk: elite soccer players are more likely to have been born in the earlier months of the year than in the later months. If you then examined the European national youth teams that feed the World Cup and professional ranks, you would find this strange phenomenon to be even more pronounced.

What might account for this strange phenomenon? Here are a few guesses: a) certain astrological signs confer superior soccer skills; b) winter-born babies tend to have higher oxygen capacity, which increases soccer stamina; c) soccer-mad parents are more likely to conceive children in springtime, at the annual peak of soccer mania; d) none of the above.

Anders Ericsson, a 58-year-old psychology professor at Florida State University, says he believes strongly in none of the above. Ericsson grew up in Sweden, and studied nuclear engineering until he realized he would have more opportunity to conduct his own research if he switched to psychology. His first experiment, nearly 30 years ago, involved memory: training a person to hear and then repeat a random series of numbers. With the first subject, after about 20 hours of training, his digit span had risen from 7 to 20, Ericsson recalls. He kept improving, and after about 200 hours of training he had risen to over 80 numbers.

This success, coupled with later research showing that memory itself is not genetically determined, led Ericsson to conclude that the act of memorizing is more of a cognitive exercise than an intuitive one. In other words, whatever inborn differences two people may exhibit in their abilities to memorize, those differences are swamped by how well each person encodes the information. And the best way to learn how to encode information meaningfully, Ericsson determined, was a process known as deliberate practice. Deliberate practice entails more than simply repeating a task. Rather, it involves setting specific goals, obtaining immediate feedback and concentrating as much on technique as on outcome.

Ericsson and his colleagues have thus taken to studying expert performers in a wide range of pursuits, including soccer. They gather all the data they can, not just performance statistics and biographical details but also the results of their own laboratory experiments with high achievers. Their work makes a rather startling assertion: the trait we commonly call talent is highly overrated. Or, put another way, expert performers whether in memory or surgery, ballet or computer programming are nearly always made, not born.

21. The birthday phenomenon found among soccer players is mentioned to

[A] stress the importance of professional training.

[B] spotlight the soccer superstars in the World Cup.

[C] introduce the topic of what makes expert performance.C

[D] explain why some soccer teams play better than others.

22. The word mania (Line 4, Paragraph 2) most probably means

[A] fun. [B] craze. [C] hysteria.B [D] excitement.

23. According to Ericsson, good memory

[A] depends on meaningful processing of information.

[B] results from intuitive rather than cognitive exercises.

[C] is determined by genetic rather than psychological factors.A

[D] requires immediate feedback and a high degree of concentration.

24. Ericsson and his colleagues believe that

[A] talent is a dominating factor for professional success.

[B] biographical data provide the key to excellent performance.

[C] the role of talent tends to be overlooked.D

[D] high achievers owe their success mostly to nurture.

25. Which of the following proverbs is closest to the message the text tries to convey?

[A] Faith will move mountains. [B] One reaps what one sows.

[C] Practice makes perfect.C [D] Like father, like son.

Text 2

For the past several years, the Sunday newspaper supplement Parade has featured a column called Ask Marilyn. People are invited to query Marilyn vos Savant, who at age 10 had tested at a mental level of someone about 23 years old; that gave her an IQ of 228 the highest score ever recorded. IQ tests ask you to complete verbal and visual analogies, to envision paper after it has been folded and cut, and to deduce numerical sequences, among other similar tasks. So it is a bit confusing when vos Savant fields such queries from the average Joe (whose IQ is 100) as, Whats the difference between love and fondness? Or what is the nature of luck and coincidence? Its not obvious how the capacity to visualize objects and to figure out numerical patterns suits one to answer questions that have eluded some of the best poets and philosophers.

Clearly, intelligence encompasses more than a score on a test. Just what does it mean to be smart? How much of intelligence can be specified, and how much can we learn about it from neurology, genetics, computer science and other fields?

The defining term of intelligence in humans still seems to be the IQ score, even though IQ tests are not given as often as they used to be. The test comes primarily in two forms: the Stanford-Binet Intelligence Scale and the Wechsler Intelligence Scales (both come in adult and childrens version). Generally costing several hundred dollars, they are usually given only by psychologists, although variations of them populate bookstores and the World Wide Web. Superhigh scores like vos Savants are no longer possible, because scoring is now based on a statistical population distribution among age peers, rather than simply dividing the mental age by the chronological age and multiplying by 100. Other standardized tests, such as the Scholastic Assessment Test (SAT) and the Graduate Record Exam (GRE), capture the main aspects of IQ tests.

Such standardized tests may not assess all the important elements necessary to succeed in school and in life, argues Robert J. Sternberg. In his article How Intelligent Is Intelligence Testing?, Sternberg notes that traditional test best assess analytical and verbal skills but fail to measure creativity and practical knowledge, components also critical to problem solving and life success. Moreover, IQ tests do not necessarily predict so well once populations or situations change. Research has found that IQ predicted leadership skills when the tests were given under low-stress conditions, but under high-stress conditions, IQ was negatively correlated with leadership that is, it predicted the opposite. Anyone who has toiled through SAT will testify that test-taking skill also matters, whether its knowing when to guess or what questions to skip.

26. Which of the following may be required in an intelligence test?

[A] Answering philosophical questions.

[B] Folding or cutting paper into different shapes.

[C] Telling the differences between certain concepts.D

[D] Choosing words or graphs similar to the given ones.

27. What can be inferred about intelligence testing from Paragraph 3?

[A] People no longer use IQ scores as an indicator of intelligence.

[B] More versions of IQ tests are now available on the Internet.

[C] The test contents and formats for adults and children may be different.C

[D] Scientists have defined the important elements of human intelligence.

28. People nowadays can no longer achieve IQ scores as high as vos Savants because

[A] the scores are obtained through different computational procedures.

[B] creativity rather than analytical skills is emphasized now.

[C] vos Savants case is an extreme one that will not repeat.A

[D] the defining characteristic of IQ tests has changed.

29. We can conclude from the last paragraph that

[A] test scores may not be reliable indicators of ones ability.

[B] IQ scores and SAT results are highly correlated.

[C] testing involves a lot of guesswork.A

[D] traditional test are out of date.

30. What is the authors attitude towards IQ tests?

[A] Supportive. [B] Skeptical. [C] Impartial.B [D] Biased.

Text 3

During the past generation, the American middle-class family that once could count on hard work and fair play to keep itself financially secure had been transformed by economic risk and new realities. Now a pink slip, a bad diagnosis, or a disappearing spouse can reduce a family from solidly middle class to newly poor in a few months.

In just one generation, millions of mothers have gone to work, transforming basic family economics. Scholars, policymakers, and critics of all stripes have debated the social implications of these changes, but few have looked at the side effect: family risk has risen as well. Todays families have budgeted to the limits of their new two-paycheck status. As a result, they have lost the parachute they once had in times of financial setback a back-up earner (usually Mom) who could go into the workforce if the primary earner got laid off or fell sick. This added-worker effect could support the safety net offered by unemployment insurance or disability insurance to help families weather bad times. But today, a disruption to family fortunes can no longer be made up with extra income from an otherwise-stay-at-home partner.

During the same period, families have been asked to absorb much more risk in their retirement income. Steelworkers, airline employees, and now those in the auto industry are joining millions of families who must worry about interest rates, stock market fluctuation, and the harsh reality that they may outlive their retirement money. For much of the past year, President Bush campaigned to move Social Security to a saving-account model, with retirees trading much or all of their guaranteed payments for payments depending on investment returns. For younger families, the picture is not any better. Both the absolute cost of healthcare and the share of it borne by families have risen and newly fashionable health-savings plans are spreading from legislative halls to Wal-Mart workers, with much higher deductibles and a large new dose of investment risk for families future healthcare. Even demographics are working against the middle class family, as the odds of having a weak elderly parent and all the attendant need for physical and financial assistance have jumped eightfold in just one generation.

From the middle-class family perspective, much of this, understandably, looks far less like an opportunity to exercise more financial responsibility, and a good deal more like a frightening acceleration of the wholesale shift of financial risk onto their already overburdened shoulders. The financial fallout has begun, and the political fallout may not be far behind.

31. Todays double-income families are at greater financial risk in that

[A] the safety net they used to enjoy has disappeared.

[B] their chances of being laid off have greatly increased.

[C] they are more vulnerable to changes in family economics.C

[D] they are deprived of unemployment or disability insurance.

32. As a result of President Bushs reform, retired people may have

[A] a higher sense of security. [B] less secured payments.

[C] less chance to invest.B [D] a guaranteed future.

33. According to the author, health-savings plans will

[A] help reduce the cost of healthcare. [B] popularize among the middle class.

[C] compensate for the reduced pensions.D [D] increase the families investment risk.

34. It can be inferred from the last paragraph that

[A] financial risks tend to outweigh political risks.

[B] the middle class may face greater political challenges.

[C] financial problems may bring about political problems.C

[D] financial responsibility is an indicator of political status.

35. Which of the following is the best title for this text?

[A] The Middle Class on the Alert [B] The Middle Class on the Cliff

[C] The Middle Class in ConflictB [D] The Middle Class in Ruins

Text 4

It never rains but it pours. Just as bosses and boards have finally sorted out their worst accounting and compliance troubles, and improved their feeble corporation governance, a new problem threatens to earn them especially in America the sort of nasty headlines that inevitably lead to heads rolling in the executive suite: data insecurity. Left, until now, to odd, low-level IT staff to put right, and seen as a concern only of data-rich industries such as banking, telecoms and air travel, information protection is now high on the bosss agenda in businesses of every variety.

Several massive leakages of customer and employee data this year from organizations as diverse as Time Warner, the American defense contractor Science Applications International Corp and even the University of California, Berkeley have left managers hurriedly peering into their intricate IT systems and business processes in search of potential vulnerabilities.

Data is becoming an asset which needs to be guarded as much as any other asset, says Haim Mendelson of Stanford Universitys business school. The ability to guard customer data is the key to market value, which the board is responsible for on behalf of shareholders. Indeed, just as there is the concept of Generally Accepted Accounting Principles (GAAP), perhaps it is time for GASP, Generally Accepted Security Practices, suggested Eli Noam of New Yorks Columbia Business School. Setting the proper investment level for security, redundancy, and recovery is a management issue, not a technical one, he says.

The mystery is that this should come as a surprise to any boss. Surely it should be obvious to the dimmest executive that trust, that most valuable of economic assets, is easily destroyed and hugely expensive to restore and that few things are more likely to destroy trust than a company letting sensitive personal data get into the wrong hands.

The current state of affairs may have been encouraged though not justified by the lack of legal penalty (in America, but not Europe) for data leakage. Until California recently passed a law, American firms did not have to tell anyone, even the victim, when data went astray. That may change fast: lots of proposed data-security legislation is now doing the rounds in Washington, D.C. Meanwhile, the theft of information about some 40 million credit-card accounts in America, disclosed on June 17th, overshadowed a hugely important decision a day earlier by Americas Federal Trade Commission (FTC) that puts corporate America on notice that regulators will act if firms fail to provide adequate data security.

36. The statement It never rains but it pours is used to introduce

[A] the fierce business competition. [B] the feeble boss-board relations.

[C] the threat from news reports.D [D] the severity of data leakage.

37. According to Paragraph 2, some organizations check their systems to find out

[A] whether there is any weak point.[B] what sort of data has been stolen.

[C] who is responsible for the leakage.A [D] how the potential spies can be located.

38. In bringing up the concept of GASP the author is making the point that

[A] shareholders’ interests should be properly attended to.

[B] information protection should be given due attention.

[C] businesses should enhance their level of accounting security.B

[D] the market value of customer data should be emphasized.

39. According to Paragraph 4, what puzzles the author is that some bosses fail to

[A] see the link between trust and data protection.[B] perceive the sensitivity of personal data.

[C] realize the high cost of data restoration.A[D] appreciate the economic value of trust.

40. It can be inferred from Paragraph 5 that

[A] data leakage is more severe in Europe.[B] FTCs decision is essential to data security.

[C] California takes the lead in security legislation.D

[D] legal penalty is a major solution to data leakage.

Part B

Directions:

You are going to read a list of headings and a text about what parents are supposed to do to guide their children into adulthood. Choose a heading from the list AG that best fits the meaning of each numbered part of the text (41-45). The first and last paragraphs of the text are not numbered. There are two extra headings that you do not need to use. Mark your answers on ANSWER SHEET 1. (10 points)

A. Set a Good Example for Your KidsB. Build Your Kids Work Skills

C. Place Time Limits on Leisure ActivitiesD. Talk about the Future on a Regular Basis

E. Help Kids Develop Coping StrategiesF. Help Your Kids Figure Out Who They Are

G. Build Your Kids Sense of Responsibility

How Can a Parent Help?

Mothers and fathers can do a lot to ensure a safe landing in early adulthood for their kids. Even if a jobs starting salary seems too small to satisfy an emerging adults need for rapid content, the transition from school to work can be less of a setback if the start-up adult is ready for the move. Here are a few measures, drawn from my book Ready or Not, Here Life Comes, that parents can take to prevent what I call work-life unreadiness.

You can start this process when they are 11 or 12. Periodically review their emerging strengths and weaknesses with them and work together on any shortcomings, like difficulty in communicating well or collaborating. Also, identify the kinds of interests they keep coming back to, as these offer clues to the careers that will fit them best.

Kids need a range of authentic role models as opposed to members of their clique, pop stars and vaunted athletes. Have regular dinner-table discussions about people the family knows and how they got where they are. Discuss the joys and downsides of your own career and encourage your kids to form some ideas about their own future. When asked what they want to do, they should be discouraged from saying “I have no idea. They can change their minds 200 times, but having only a foggy view of the future is of little good.

Teachers are responsible for teaching kids how to learn; parents should be responsible for teaching them how to work. Assign responsibilities around the house and make sure homework deadlines are met. Encourage teenagers to take a part-time job. Kids need plenty of practice delaying gratification and deploying effective organizational skills, such as managing time and setting priorities.

Playing video games encourages immediate content. And hours of watching TV shows with canned laughter only teaches kids to process information in a passive way. At the same time, listening through earphones to the same monotonous beats for long stretches encourages kids to stay inside their bubble instead of pursuing other endeavors. All these activities can prevent the growth of important communication and thinking skills and make it difficult for kids to develop the kind of sustained concentration they will need for most jobs.

They should know how to deal with setbacks, stresses and feelings of inadequacy. They should also learn how to solve problems and resolve conflicts, ways to brainstorm and think critically. Discussions at home can help kids practice doing these things and help them apply these skills to everyday life situations.

What about the son or daughter who is grown but seems to be struggling and wandering aimlessly through early adulthood? Parents still have a major role to play, but now it is more delicate. They have to be careful not to come across as disappointed in their child. They should exhibit strong interest and respect for whatever currently interests their fledging adult (as naive or ill conceived as it may seem) while becoming a partner in exploring options for the future. Most of all, these new adults must feel that they are respected and supported by a family that appreciates them.

Part C

Directions:

Read the following text carefully and then translate the underlined segments into Chinese. Your translation should be written clearly on ANSWER SHEET 2. (10 points)

The study of law has been recognized for centuries as a basic intellectual discipline in European universities. However, only in recent years has it become a feature of undergraduate programs in Canadian universities. (46) Traditionally, legal learning has been viewed in such institutions as the special preserve of lawyers, rather than a necessary part of the intellectual equipment of an educated person. Happily, the older and more continental view of legal education is establishing itself in a number of Canadian universities and some have even begun to offer undergraduate degrees in law.

If the study of law is beginning to establish itself as part and parcel of a general education, its aims and methods should appeal directly to journalism educators. Law is a discipline which encourages responsible judgment. On the one hand, it provides opportunities to analyze such ideas as justice, democracy and freedom. (47) On the other, it links these concepts to everyday realities in a manner which is parallel to the links journalists forge on a daily basis as they cover and comment on the news. For example, notions of evidence and fact, of basic rights and public interest are at work in the process of journalistic judgment and production just as in courts of law. Sharpening judgment by absorbing and reflecting on law is a desirable component of a journalists intellectual preparation for his or her career.

(48) But the idea that the journalist must understand the law more profoundly than an ordinary citizen rests on an understanding of the established conventions and special responsibilities of the news media. Politics or, more broadly, the functioning of the state, is a major subject for journalists. The better informed they are about the way the state works, the better their reporting will be. (49) In fact, it is difficult to see how journalists who do not have a clear grasp of the basic features of the Canadian Constitution can do a competent job on political stories.

Furthermore, the legal system and the events which occur within it are primary subjects for journalists. While the quality of legal journalism varies greatly, there is an undue reliance amongst many journalists on interpretations supplied to them by lawyers. (50) While comment and reaction from lawyers may enhance stories, it is preferable for journalists to rely on their own notions of significance and make their own judgments. These can only come from a well-grounded understanding of the legal system.

Section III Writing

Part A

51. Directions:

Write a letter to you university library, making suggestions for improving its service.

You should write about 100 words on ANSWER SHEET 2.

Do not sign your own name at the end of the letter. Use Li Ming instead.

Do not write the address. (10 points)

Part B

52. Directions:

Write an essay of 160-200 words based on the following drawing. In your essay, you should

1) describe the drawing briefly,

2) explain its intended meaning, and then

3) support your view with an example/examples.

You should write neatly on ANSWER SHEET 2. (20 points)

51. 参考范文

January 20th, 2007

Dear Sir or Madam,

Im a student in the university and a loyal reader of this library. Im writing to tell some of my ideas, which I hope to be helpful for you.

I notice that many magazines in our library are out of date. It would be beneficial to us students if they could be updated in time. And I suggest introducing some new journals so as to bring new fresh air to the library. Furthermore, since we have a huge number of books, it is not easy to find the right one easily. However, if we can introduce some new searching means, such as implementing new information management system that would be useful.

Thank you for taking time reading this letter and Im looking forward to seeing some new changes soon.

Sincerely Yours,

Li Ming

Part B (20 points)

52. 参考范文

As can be seen from the cartoon, different ideas may come from the same thing. In the picture, while trying to catch the upcoming soccer, the goal-keeper says to himself why it is so big. And, the striker simply thinks in a different way, that is why it is so small?!

What makes such a big contrary on the same tournament at the same moment? It is no doubt that they are facing the very same goal and experiencing the very same moment. However, the subjective views result in different impression on the same object. Many of us may still remember the story of a pony crossing the river, which we learned from the textbook in primary school. The squirrel tells him, the river is deep; and the cow tells him, the river is not deep at all. However, in the end, he tells himself a third answer. Therefore, it is not exaggerating to say that most of us are looking into the world with personal ideas. Subjective mental status may result in a really big difference in personal views, just like the goal-keeper and the striker in the drawing.

A possible solution might be to face any situation as objectively as possible. If we realize this in an objective way, it would be good for us to deal with what we encounter in life, especially when we are in setbacks or facing difficulties.

2006年全国硕士研究生入学统一考试英语试题

Section I Use of English

Directions:

Read the following text. Choose the best word(s) for each numbered blank and mark [A], [B], [C] or [D] on ANSWER SHEET 1. (10 points)

The homeless make up a growing percentage of Americas population. homelessness has reached such proportions that local governments cant possibly . To help homeless people independence, the federal government must support job training programs, the minimum wage, and fund more low-cost housing.

everyone agrees on the number of Americans who are homeless. Estimates anywhere from 600,000 to 3 million. the figure may vary, analysts do agree on another matter: that the number of the homeless is . One of the federal government’s studies that the number of the homeless will reach nearly 19 million by the end of this decade.

Finding ways to this growing homeless population has become increasingly difficult. when homeless individuals manage to find a that will give them three meals a day and a place to sleep at night, a good number still spend the bulk of each day the street. Part of the problem is that many homeless adults are addicted to alcohol or drugs. And a significant number of the homeless have serious mental disorders. Many others, not addicted or mentally ill, simply lack the everyday skills needed to turn their lives . Boston Globe reporter Chris Reidy notes that the situation will improve only when there are programs that address the many needs of the homeless. Edward Zlotkowski, director of community service at Bentley College in Massachusetts, it, “There has to be of programs. What’s needed is a package deal.”

1. [A] Indeed [B] Likewise [C] ThereforeA [D] Furthermore

2. [A] stand [B] cope [C] approveB [D] retain

3. [A] in [B] for [C] withD [D] toward

4. [A] raise [B] add [C] takeA [D] keep

5. [A] generally [B] almost [C] hardlyD [D] not

6. [A] cover [B] change [C] rangeC [D] differ

7. [A] Now that [B] Although [C] ProvidedB [D] Except that

8. [A] inflating [B] expanding [C] increasingC [D] extending

9. [A] predicts [B] displays [C] provesA [D] discovers

10. [A] assist [B] track [C] sustainA [D] dismiss

11. [A] Hence [B] But [C] EvenC [D] Only

12. [A] lodging [B] shelter [C] dwellingB [D] house

13. [A] searching [B] strolling [C] crowdingD [D] wandering

14. [A] when [B] once [C] whileC [D] whereas

15. [A] life [B] existence [C] survivalC [D] maintenance

16. [A] around [B] over [C] onA [D] up

17. [A] complex [B] comprehensive [C] complementaryB [D] compensating

18. [A] So [B] Since [C] AsC [D] Thus

19. [A] puts [B] interprets [C] assumesA [D] makes

20. [A] supervision [B] manipulation [C] regulationD [D] coordination

Section II Reading Comprehension

Part A

Directions:

Read the following four texts. Answer the questions below each text by choosing [A], [B], [C], or [D]. Mark your answers on ANSWER SHEET 1. (40 points)

Text 1

In spite of endless talk of difference, American society is an amazing machine for homogenizing people. There is the democratizing uniformity of dress and discourse, and the casualness and absence of deference characteristic of popular culture. People are absorbed into a culture of consumption launched by the 19th-century department stores that offered “vast arrays of goods in an elegant atmosphere. Instead of intimate shops catering to a knowledgeable elite,” these were stores “anyone could enter, regardless of class or background. This turned shopping into a public and democratic act.” The mass media, advertising and sports are other forces for homogenization.

Immigrants are quickly fitting into this common culture, which may not be altogether elevating but is hardly poisonous. Writing for the National Immigration Forum, Gregory Rodriguez reports that todays immigration is neither at unprecedented levels nor resistant to assimilation. In 1998 immigrants were 9.8 percent of population; in 1900, 13.6 percent. In the 10 years prior to 1990, 3.1 immigrants arrived for every 1,000 residents; in the 10 years prior to 1890, 9.2 for every 1,000. Now, consider three indices of assimilation -- language, home ownership and intermarriage.

The 1990 Census revealed that a majority of immigrants from each of the fifteen most common countries of origin spoke English well or very well after ten years of residence. The children of immigrants tend to be bilingual and proficient in English. “By the third generation, the original language is lost in the majority of immigrant families.” Hence the description of America as a “graveyard” for languages. By 1996 foreign-born immigrants who had arrived before 1970 had a home ownership rate of 75.6 percent, higher than the 69.8 percent rate among native-born Americans.

Foreign-born Asians and Hispanics have higher rates of intermarriage than do U.S.-born whites and blacks. By the third generation, one third of Hispanic women are married to non-Hispanics, and 41 percent of Asian-American women are married to non-Asians.

Rodriguez notes that children in remote villages around the world are fans of superstars like Arnold Schwarzenegger and Garth Brooks, yet some Americans fear that immigrants living within the United States remain somehow immune to the nations assimilative power.”

Are there divisive issues and pockets of seething anger in America? Indeed. It is big enough to have a bit of everything. But particularly when viewed against Americas turbulent past, todays social indices hardly suggest a dark and deteriorating social environment.

21. The word homogenizing (Line 2, Paragraph 1) most probably means ________.

[A] identifying[B] associating[C] assimilatingC[D] monopolizing

22. According to the author, the department stores of the 19th century ________.

[A] played a role in the spread of popular culture[B] became intimate shops for common consumers

[C] satisfied the needs of a knowledgeable eliteA[D] owed its emergence tothe culture of consumption

23. The text suggests that immigrants now in the U.S. ________.

[A] are resistant to homogenization[B] exert a great influence on American culture

[C] are hardly a threat to the common cultureC[D] constitute the majority of the population

24. Why are Arnold Schwarzenegger and Garth Brooks mentioned in Paragraph 5?

[A] To prove their popularity around the world.[B] To reveal the publics fear of immigrants.

[C]To give examples of successful immigrants.D[D]To show the powerful influence of American culture.

25. In the authors opinion, the absorption of immigrants into American society is ________.

[A] rewarding[B] successful[C] fruitlessB[D] harmful

Text 2

Stratford-on-Avon, as we all know, has only one industry -- William Shakespeare -- but there are two distinctly separate and increasingly hostile branches. There is the Royal Shakespeare Company (RSC), which presents superb productions of the plays at the Shakespeare Memorial Theatre on the Avon. And there are the townsfolk who largely live off the tourists who come, not to see the plays, but to look at Anne Hathaway’s Cottage, Shakespeare’s birthplace and the other sights.

The worthy residents of Stratford doubt that the theatre adds a penny to their revenue. They frankly dislike the RSCs actors, them with their long hair and beards and sandals and noisiness. Its all deliciously ironic when you consider that Shakespeare, who earns their living, was himself an actor (with a beard) and did his share of noise-making.

The tourist streams are not entirely separate. The sightseers who come by bus -- and often take in Warwick Castle and Blenheim Palace on the side -- dont usually see the plays, and some of them are even surprised to find a theatre in Stratford. However, the playgoers do manage a little sight-seeing along with their playgoing. It is the playgoers, the RSC contends, who bring in much of the town’s revenue because they spend the night (some of them four or five nights) pouring cash into the hotels and restaurants. The sightseers can take in everything and get out of town by nightfall.

The townsfolk dont see it this way and local council does not contribute directly to the subsidy of the Royal Shakespeare Company. Stratford cries poor traditionally. Nevertheless every hotel in town seems to be adding a new wing or cocktail lounge. Hilton is building its own hotel there, which you may be sure will be decorated with Hamlet Hamburger Bars, the Lear Lounge, the Banquo Banqueting Room, and so forth, and will be very expensive.

Anyway, the townsfolk cant understand why the Royal Shakespeare Company needs a subsidy. (The theatre has broken attendance records for three years in a row. Last year its 1,431 seats were 94 percent occupied all year long and this year theyll do better.) The reason, of course, is that costs have rocketed and ticket prices have stayed low.

It would be a shame to raise prices too much because it would drive away the young people who are Stratfords most attractive clientele. They come entirely for the plays, not the sights. They all seem to look alike (though they come from all over) -- lean, pointed, dedicated faces, wearing jeans and sandals, eating their buns and bedding down for the night on the flagstones outside the theatre to buy the 20 seats and 80 standing-room tickets held for the sleepers and sold to them when the box office opens at 10:30 a.m.

26. From the first two paragraphs, we learn that ________.

[A] the townsfolk deny the RSC’s contribution to the town’s revenue

[B] the actors of the RSC imitate Shakespeare on and off stage

[C] the two branches of the RSC are not on good termsA

[D] the townsfolk earn little from tourism

27. It can be inferred from Paragraph 3 that ________.

[A] the sightseers cannot visit the Castle and the Palace separately

[B] the playgoers spend more money than the sightseers

[C] the sightseers do more shopping than the playgoersB

[D] the playgoers go to no other places in town than the theater

28. By saying “Stratford cries poor traditionally” (Line 2-3, Paragraph 4), the author implies that ________.

[A] Stratford cannot afford the expansion projects

[B] Stratford has long been in financial difficulties

[C] the town is not really short of moneyC

[D] the townsfolk used to be poorly paid

29. According to the townsfolk, the RSC deserves no subsidy because ________.

[A] ticket prices can be raised to cover the spending[B] the company is financially ill-managed

[C] the behavior of the actors is not socially acceptableD[D] the theatre attendance is on the rise

30. From the text we can conclude that the author ________.

[A] is supportive of both sides[B] favors the townsfolk’s view

[C] takes a detached attitudeD[D] is sympathetic to the RSC

Text 3

When prehistoric man arrived in new parts of the world, something strange happened to the large animals. They suddenly became extinct. Smaller species survived. The large, slow-growing animals were easy game, and were quickly hunted to extinction. Now something similar could be happening in the oceans.

That the seas are being overfished has been known for years. What researchers such as Ransom Myers and Boris Worm have shown is just how fast things are changing. They have looked at half a century of data from fisheries around the world. Their methods do not attempt to estimate the actual biomass (the amount of living biological matter) of fish species in particular parts of the ocean, but rather changes in that biomass over time. According to their latest paper published in Nature, the biomass of large predators (animals that kill and eat other animals) in a new fishery is reduced on average by 80% within 15 years of the start of exploitation. In some long-fished areas, it has halved again since then.

Dr. Worm acknowledges that these figures are conservative. One reason for this is that fishing technology has improved. Todays vessels can find their prey using satellites and sonar, which were not available 50 years ago. That means a higher proportion of what is in the sea is being caught, so the real difference between present and past is likely to be worse than the one recorded by changes in catch sizes. In the early days, too, longlines would have been more saturated with fish. Some individuals would therefore not have been caught, since no baited hooks would have been available to trap them, leading to an underestimate of fish stocks in the past. Furthermore, in the early days of longline fishing, a lot of fish were lost to sharks after they had been hooked. That is no longer a problem, because there are fewer sharks around now.

Dr. Myers and Dr. Worm argue that their work gives a correct baseline, which future management efforts must take into account. They believe the data support an idea current among marine biologists, that of the shifting baseline.” The notion is that people have failed to detect the massive changes which have happened in the ocean because they have been looking back only a relatively short time into the past. That matters because theory suggests that the maximum sustainable yield that can be cropped from a fishery comes when the biomass of a target species is about 50% of its original levels. Most fisheries are well below that, which is a bad way to do business.

31. The extinction of large prehistoric animals is noted to suggest that ________.

[A] large animal were vulnerable to the changing environment

[B] small species survived as large animals disappeared

[C] large sea animals may face the same threat todayC

[D] slow-growing fish outlive fast-growing ones

32. We can infer from Dr. Myers and Dr. Worms paper that ________.

[A] the stock of large predators in some old fisheries has reduced by 90%

[B] there are only half as many fisheries as there were 15 years ago

[C] the catch sizes in new fisheries are only 20% of the original amountA

[D] the number of larger predators dropped faster in new fisheries than in the old

33. By saying "these figures are conservative" (Line 1, paragraph 3), Dr. Worm means that ________.

[A] fishing technology has improved rapidly

[B] the catch-sizes are actually smaller than recorded

[C] the marine biomass has suffered a greater lossC

[D] the data collected so far are out of date

34. Dr. Myers and other researchers hold that ________.

[A] people should look for a baseline that can work for a longer time

[B] fisheries should keep their yields below 50% of the biomass

[C] the ocean biomass should be restored to its original levelD

[D] people should adjust the fishing baseline to the changing situation

35. The author seems to be mainly concerned with most fisheries ________.

[A] management efficiency[B] biomass level[C] catch-size limitsB[D] technological application

Text 4

Many things make people think artists are weird. But the weirdest may be this: artists only job is to explore emotions, and yet they choose to focus on the ones that feel bad.

This wasnt always so. The earliest forms of art, like painting and music, are those best suited for expressing joy. But somewhere from the 19th century onward, more artists began seeing happiness as meaningless, phony or, worst of all, boring, as we went from Wordsworth’s daffodils to Baudelaire’s flowers of evil.

You could argue that art became more skeptical of happiness because modern times have seen so much misery. But its not as if earlier times didnt know perpetual war, disaster and the massacre of innocents. The reason, in fact, may be just the opposite: there is too much damn happiness in the world today.

After all, what is the one modern form of expression almost completely dedicated to depicting happiness? Advertising. The rise of anti-happy art almost exactly tracks the emergence of mass media, and with it, a commercial culture in which happiness is not just an ideal but an ideology.

People in earlier eras were surrounded by reminders of misery. They worked until exhausted, lived with few protections and died young. In the West, before mass communication and literacy, the most powerful mass medium was the church, which reminded worshippers that their souls were in danger and that they would someday be meat for worms. Given all this, they did not exactly need their art to be a bummer too.

Today the messages the average Westerner is surrounded with are not religious but commercial, and forever happy. Fast-food eaters, news anchors, text messengers, all smiling, smiling, smiling. Our magazines feature beaming celebrities and happy families in perfect homes. And since these messages have an agenda -- to lure us to open our wallets -- they make the very idea of happiness seem unreliable. “Celebrate!” commanded the ads for the arthritis drug Celebrex, before we found out it could increase the risk of heart attacks.

But what we forget -- what our economy depends on us forgetting -- is that happiness is more than pleasure without pain. The things that bring the greatest joy carry the greatest potential for loss and disappointment. Today, surrounded by promises of easy happiness, we need art to tell us, as religion once did, Memento mori: remember that you will die, that everything ends, and that happiness comes not in denying this but in living with it. It’s a message even more bitter than a clove cigarette, yet, somehow, a breath of fresh air.

36. By citing the examples of poets Wordsworth and Baudelaire, the author intends to show that ________.

[A] poetry is not as expressive of joy as painting or music

[B] art grows out of both positive and negative feelings

[C] poets today are less skeptical of happinessD

[D] artists have changed their focus of interest

37. The word bummer (Line 5, paragraph 5) most probably means something ________.

[A] religious[B] unpleasant[C] entertainingB[D] commercial

38. In the authors opinion, advertising ________.

[A] emerges in the wake of the anti-happy art

[B] is a cause of disappointment for the general public

[C] replaces the church as a major source of informationD

[D] creates an illusion of happiness rather than happiness itself

39. We can learn from the last paragraph that the author believes ________.

[A] happiness more often than not ends in sadness[B] the anti-happy art is distasteful but refreshing

[C] misery should be enjoyed rather than deniedB

[D] the anti-happy art flourishes when economy booms

40. Which of the following is true of the text?

[A] Religion once functioned as a reminder of misery.

[B] Art provides a balance between expectation and reality.

[C] People feel disappointed at the realities of modern society.A

[D] Mass media are inclined to cover disasters and deaths.

Part B

Directions:

In the following article, some sentences have been removed. For Questions 41-45, choose the most suitable one from the list A-G to fit into each of the numbered gaps. There are two extra choices, which you do not need to use in any of the blanks. Mark your answers on ANSWER SHEET 1. (10 points)

On the north bank of the Ohio river sits Evansville, Ind., home of David Williams, 52, and of a riverboat casino (a place where gambling games are played). During several years of gambling in that casino, Williams, a state auditor earning $35,000 a year, lost approximately $175,000. He had never gambled before the casino sent him a coupon for $20 worth of gambling.

He visited the casino, lost the $20 and left. On his second visit he lost $800. The casino issued to him, as a good customer, a "Fun Card", which when used in the casino earns points for meals and drinks, and enables the casino to track the users gambling activities. For Williams, those activities become what he calls "electronic heroin".

(41) ________. In 1997 he lost $21,000 to one slot machine in two days. In March 1997 he lost $72,186. He sometimes played two slot machines at a time, all night, until the boat docked at 5 a.m., then went back aboard when the casino opened at 9 a.m. Now he is suing the casino, charging that it should have refused his patronage because it knew he was addicted. It did know he had a problem.

In March 1998 a friend of Williamss got him involuntarily confined to a treatment center for addictions, and wrote to inform the casino of Williamss gambling problem. The casino included a photo of Williams among those of banned gamblers, and wrote to him a “cease admissions” letter. Noting the medical/psychological nature of problem gambling behavior, the letter said that before being readmitted to the casino he would have to present medical/psychological information demonstrating that patronizing the casino would pose no threat to his safety or well-being.

(42) ________.

The Wall Street Journal reports that the casino has 24 signs warning: Enjoy the fun... and always bet with your head, not over it.” Every entrance ticket lists a toll-free number for counseling from the Indiana Department of Mental Health. Nevertheless, Williams’s suit charges that the casino, knowing he was “helplessly addicted to gambling,” intentionally worked to “lure” him to “engage in conduct against his will.Well.

(43) ________.

The fourth edition of the Diagnostic and Statistical Manual of Mental Disorders says pathological gambling involves persistent, recurring and uncontrollable pursuit less of money than of thrill of taking risks in quest of a windfall.

(44) ________. Pushed by science, or what claims to be science, society is reclassifying what once were considered character flaws or moral failings as personality disorders akin to physical disabilities.

(45) ________.

Forty-four states have lotteries, 29 have casinos, and most of these states are to varying degrees dependent on -- you might say addicted to -- revenues from wagering. And since the first Internet gambling site was created in 1995, competition for gamblers’ dollars has become intense. The Oct. 28 issue of Newsweek reported that 2 million gamblers patronize 1,800 virtual casinos every week. With $3.5 billion being lost on Internet wagers this year, gambling has passed pornography as the Web’s most profitable business.

[A] Although no such evidence was presented, the casinos marketing department continued to pepper him with mailings. And he entered the casino and used his Fun Card without being detected.

[B] It is unclear what luring was required, given his compulsive behavior. And in what sense was his will operative?

[C] By the time he had lost $5,000 he said to himself that if he could get back to even, he would quit. One night he won $5,500, but he did not quit.

[D] Gambling has been a common feature of American life forever, but for a long time it was broadly considered a sin, or a social disease. Now it is a social policy: the most important and aggressive promoter of gambling in America is the government.

[E] David Williamss suit should trouble this gambling nation. But dont bet on it.

[F] It is worrisome that society is medicalizing more and more behavioral problems, often defining as addictions what earlier, sterner generations explained as weakness of will.

[G] The anonymous, lonely, undistracted nature of online gambling is especially conducive to compulsive behavior. But even if the government knew how to move against Internet gambling, what would be its grounds for doing so?

Part C

Directions:

Read the following text carefully and then translate the underlined segments into Chinese. Your translation should be written clearly on ANSWER SHEET 2. (10 points)

Is it true that the American intellectual is rejected and considered of no account in his society? I am going to suggest that it is not true. Father Bruckberger told part of the story when he observed that it is the intellectuals who have rejected America. But they have done more than that. They have grown dissatisfied with the role of intellectual. It is they, not America, who have become anti-intellectual.

First, the object of our study pleads for definition. What is an intellectual? 46) I shall define him as an individual who has elected as his primary duty and pleasure in life the activity of thinking in a Socratic (苏格拉底) way about moral problems. He explores such problems consciously, articulately, and frankly, first by asking factual questions, then by asking moral questions, finally by suggesting action which seems appropriate in the light of the factual and moral information which he has obtained. 47) His function is analogous to that of a judge, who must accept the obligation of revealing in as obvious a manner as possible the course of reasoning which led him to his decision.

This definition excludes many individuals usually referred to as intellectuals -- the average scientist, for one. 48) I have excluded him because, while his accomplishments may contribute to the solution of moral problems, he has not been charged with the task of approaching any but the factual aspects of those problems. Like other human beings, he encounters moral issues even in the everyday performance of his routine duties -- he is not supposed to cook his experiments, manufacture evidence, or doctor his reports. 49) But his primary task is not to think about the moral code which governs his activity, any more than a businessman is expected to dedicate his energies to an exploration of rules of conduct in business. During most of his waking life he will take his code for granted, as the businessman takes his ethics.

The definition also excludes the majority of teachers, despite the fact that teaching has traditionally been the method whereby many intellectuals earn their living. 50) They may teach very well and more than earn their salaries, but most of them make little or no independent reflections on human problems which involve moral judgment. This description even fits the majority of eminent scholars. Being learned in some branch of human knowledge is one thing, living in "public and illustrious thoughts,” as Emerson would say, is something else.

Section III Writing

Part A

51. Directions

You want to contribute to Project Hope by offering financial aid to a child in a remote area. Write a letter to the department concerned, asking them to help find a candidate. You should specify what kind of child you want to help and how you will carry out your plan.

Write your letter in no less than 100 words. Write it neatly on ANSWER SHEET 2.

Do not sign your own name at the end of the letter; use Li Ming instead.

Do not write the address. (10 points)

Part B

52. Directions:

Study the following photos carefully and write an essay in which you should

1. describe the photos briefly,

2. interpret the social phenomenon reflected by them, and

3. give your point of view.

You should write 160-200 words neatly on ANSWER SHEET 2. (20 points)

51. 参考范文

Dear Sir or Madam,

As a college student who is studying and living in a good environment, I wish to contribute to Project Hope by offering financial assistance to a child in a remote area. Having conceived such a plan for a long time, I write this letter to request your help to recommend a proper candidate.

I wonder if it is convenient for you if three things concerning the child are taken into consideration. First, the child should come from Gansu Province, for I intend to help a child from my hometown. Second, it will be better if the child is a primary school student. I hope I will help him/her from the very beginning. In addition, he/she must be willing to return to his hometown to help built it after graduation from university.

My plan will be carried out as follows. On one hand, I will remit at least 2,000 Yuan in cash every year until he/she finishes his/her education before entering college. On the other hand, I decide to teach the child math and English in person during my summer vacation, which will surely be more beneficial to the child.

Your prompt help would be highly appreciated. And I am looking forward to your reply very soon.

Yours sincerely,

Li Ming

Part B (20 points)

52. 参考范文

How ironic the two pictures are in describing one of the most widespread social phenomena concerning idol adoration! In the first picture, a young man writes the name of Beckham on his face. In the second picture, another young man spends 300 Yuan in dealing with his hair to make himself look like Beckham.

The meaning conveyed in the two pictures reveals that in current China some young people are losing themselves. I am greatly shocked by the enthusiasm for this British football superstar shown by these two young men. Frankly speaking, things of this kind really happen among us. Some people, especially college students, do nothing but concentrate on imitating superstars. This does great harm to their study and growth. If we can’t stop the worsening of this tendency, our own culture will be damaged, and we ourselves will be the ultimate victims.

From my point of view, a lot of measures should be taken to save our losing culture and re-find ourselves. In fact, some measures have already been taken. In my university, campaigns have been launched to educate people to pay more attention to our traditional culture and read more books instead of focusing on our appearances. As a result, we have witnessed some improvements but still there is a long way to go.

2005年全国硕士研究生入学统一考试英语试题

Section I Use of English

Directions:

Read the following text. Choose the best word(s) for each numbered blank and mark [A], [B], [C] or [D] on ANSWER SHEET 1 (10 points)

The human nose is an underrated tool. Humans are often thought to be insensitive smellers compared with animals, this is largely because, animals, we stand upright. This means that our noses are to perceiving those smells which float through the air, the majority of smells which stick to surfaces. In fact, , we are extremely sensitive to smells, we do not generally realize it. Our noses are capable of human smells even when these are to far below one part in one million.

Strangely, some people find that they can smell one type of flower but not another, others are sensitive to the smells of both flowers. This may be because some people do not have the genes necessary to generate smell receptors in the nose. These receptors are the cells which sense smells and send to the brain. However, it has been found that even people insensitive to a certain smell can suddenly become sensitive to it when to it often enough.

The explanation for insensitivity to smell seems to be that the brain finds it to keep all smell receptors working all the time but can new receptors if necessary. This may explain why we are not usually sensitive to our own smellswe simply do not need to be. We are not of the usual smell of our own house, but we new smells when we visit someone else’s. The brain finds it best to keep smell receptors for unfamiliar and emergency signals the smell of smoke, which might indicate the danger of fire.

1. [A] although[B] as[C] butC[D] while

2. [A] above[B] unlike[C] excludingB[D] besides

3. [A] limited[B] committed[C] dedicatedA[D] confined

4. [A] catching[B] ignoring[C] missingC[D] tracking

5. [A] anyway[B] though[C] insteadB[D] therefore

6. [A] even if[B] if only[C] only ifA[D] as if

7. [A] distinguishing[B] discovering[C] determiningD[D] detecting

8. [A] diluted[B] dissolved[C] dispersedA[D] diffused

9. [A] when[B] since[C] forD[D] whereas

10. [A] unusual[B] particular[C] uniqueB[D] typical

11. [A] signs[B] stimuli[C] messagesC[D] impulses

12. [A] at first[B] at all[C] at largeA[D] at times

13. [A] subjected[B] left[C] drawnD[D] exposed

14. [A] ineffective[B] incompetent[C] inefficientC[D] insufficient

15. [A] introduce[B] summon[C] triggerD[D] create

16. [A] still[B] also[C] otherwiseB[D] nevertheless

17. [A] sure[B] sick[C] awareC[D] tired

18. [A] tolerate[B] repel[C] neglectD[D] notice

19. [A] available[B] reliable[C] identifiableA[D] suitable

20. [A] similar to[B] such as[C] along withB[D] aside from

Section II Reading Comprehension

Part A

Directions:

Read the following four texts. Answer the questions below each text by choosing [A], [B], [C] or D. Mark your answers on ANSWER SHEET 1 (40 points)

Text 1

Everybody loves a fat pay rise. Yet pleasure at your own can vanish if you learn that a colleague has been given a bigger one. Indeed, if he has a reputation for slacking, you might even be outraged. Such behaviour is regarded as all too human,” with the underlying assumption that other animals would not be capable of this finely developed sense of grievance. But a study by Sarah Brosnan and Frans de Waal of Emory University in Atlanta, Georgia, which has just been published in Nature, suggests that it is all too monkey, as well.

The researchers studied the behaviour of female brown capuchin monkeys. They look cute. They are good-natured, co-operative creatures, and they share their food readily. Above all, like their female human counterparts, they tend to pay much closer attention to the value of “goods and services” than males.

Such characteristics make them perfect candidates for Dr. Brosnan’s and Dr. de Waal’s study. The researchers spent two years teaching their monkeys to exchange tokens for food. Normally, the monkeys were happy enough to exchange pieces of rock for slices of cucumber. However, when two monkeys were placed in separate but adjoining chambers, so that each could observe what the other was getting in return for its rock, their behaviour became markedly different.

In the world of capuchins, grapes are luxury goods (and much preferable to cucumbers). So when one monkey was handed a grape in exchange for her token, the second was reluctant to hand hers over for a mere piece of cucumber. And if one received a grape without having to provide her token in exchange at all, the other either tossed her own token at the researcher or out of the chamber, or refused to accept the slice of cucumber. Indeed, the mere presence of a grape in the other chamber (without an actual monkey to eat it) was enough to induce resentment in a female capuchin.

The researchers suggest that capuchin monkeys, like humans, are guided by social emotions. In the wild, they are a co-operative, group-living species. Such co-operation is likely to be stable only when each animal feels it is not being cheated. Feelings of righteous indignation, it seems, are not the preserve of people alone. Refusing a lesser reward completely makes these feelings abundantly clear to other members of the group. However, whether such a sense of fairness evolved independently in capuchins and humans, or whether it stems from the common ancestor that the species had 35 million years ago, is, as yet, an unanswered question.

21. In the opening paragraph, the author introduces his topic by ________.

[A] posing a contrast[B] justifying an assumption

[C] making a comparisonC[D] explaining a phenomenon

22. The statement it is all too monkey (Last line, Paragraph l) implies that ________.

[A] monkeys are also outraged by slack rivals[B] resenting unfairness is also monkeys nature

[C] monkeys, like humans, tend to be jealous of each otherB

[D] no animals other than monkeys can develop such emotions

23. Female capuchin monkeys were chosen for the research most probably because they are ________.

[A] more inclined to weigh what they get[B] attentive to researchers instructions

[C] nice in both appearance and temperamentA[D] more generous than their male companions

24. Dr. Brosnan and Dr. de Waal have eventually found in their study that the monkeys ________.

[A] prefer grapes to cucumbers[B] can be taught to exchange things

[C] will not be co-operative if feeling cheatedC[D] are unhappy when separated from others

25. What can we infer from the last paragraph?

[A] Monkeys can be trained to develop social emotions.

[B] Human indignation evolved from an uncertain source.

[C] Animals usually show their feelings openly as humans do.B

[D] Cooperation among monkeys remains stable only in the wild.

Text 2

Do you remember all those years when scientists argued that smoking would kill us but the doubters insisted that we didnt know for sure? That the evidence was inconclusive, the science uncertain? That the antismoking lobby was out to destroy our way of life and the government should stay out of the way? Lots of Americans bought that nonsense, and over three decades, some 10 million smokers went to early graves.

There are upsetting parallels today, as scientists in one wave after another try to awaken us to the growing threat of global warming. The latest was a panel from the National Academy of Sciences, enlisted by the White House, to tell us that the Earths atmosphere is definitely warming and that the problem is largely man-made. The clear message is that we should get moving to protect ourselves. The president of the National Academy, Bruce Alberts, added this key point in the preface to the panel’s report: “Science never has all the answers. But science does provide us with the best available guide to the future, and it is critical that our nation and the world base important policies on the best judgments that science can provide concerning the future consequences of present actions.”

Just as on smoking, voices now come from many quarters insisting that the science about global warming is incomplete, that its OK to keep pouring fumes into the air until we know for sure. This is a dangerous game: by the time 100 percent of the evidence is in, it may be too late. With the risks obvious and growing, a prudent people would take out an insurance policy now.

Fortunately, the White House is starting to pay attention. But its obvious that a majority of the presidents advisers still dont take global warming seriously. Instead of a plan of action, they continue to press for more research -- a classic case of paralysis by analysis.

To serve as responsible stewards of the planet, we must press forward on deeper atmospheric and oceanic research. But research alone is inadequate. If the Administration wont take the legislative initiative, Congress should help to begin fashioning conservation measures. A bill by Democratic Senator Robert Byrd of West Virginia, which would offer financial incentives for private industry, is a promising start. Many see that the country is getting ready to build lots of new power plants to meet our energy needs. If we are ever going to protect the atmosphere, it is crucial that those new plants be environmentally sound.

26. An argument made by supporters of smoking was that ________.

[A] there was no scientific evidence of the correlation between smoking and death

[B] the number of early deaths of smokers in the past decades was insignificant

[C] people had the freedom to choose their own way of lifeC

[D] antismoking people were usually talking nonsense

27. According to Bruce Alberts, science can serve as ________.

[A] a protector[B] a judge[C] a criticD[D] a guide

28. What does the author mean by paralysis by analysis (Last line, Paragraph 4)?

[A] Endless studies kill action.[B] Careful investigation reveals truth.

[C] Prudent planning hinders progress.A[D] Extensive research helps decision-making.

29. According to the author, what should the Administration do about global warming?

[A] Offer aid to build cleaner power plants.[B] Raise public awareness of conservation.

[C] Press for further scientific research.D[D] Take some legislative measures.

30. The author associates the issue of global warming with that of smoking because ________.

[A] they both suffered from the governments negligence

[B] a lesson from the latter is applicable to the former

[C] the outcome of the latter aggravates the formerB[D] both of them have turned from bad to worse

Text 3

Of all the components of a good nights sleep, dreams seem to be least within our control. In dreams, a window opens into a world where logic is suspended and dead people speak. A century ago, Freud formulated his revolutionary theory that dreams were the disguised shadows of our unconscious desires and fears; by the late 1970s, neurologists had switched to thinking of them as just “mental noise” -- the random byproducts of the neural-repair work that goes on during sleep. Now researchers suspect that dreams are part of the mind’s emotional thermostat, regulating moods while the brain is “off-line.” And one leading authority says that these intensely powerful mental events can be not only harnessed but actually brought under conscious control, to help us sleep and feel better, “It’s your dream,” says Rosalind Cartwright, chair of psychology at Chicago’s Medical Center. “If you don’t like it, change it.”

Evidence from brain imaging supports this view. The brain is as active during REM (rapid eye movement) sleep -- when most vivid dreams occur -- as it is when fully awake, says Dr, Eric Nofzinger at the University of Pittsburgh. But not all parts of the brain are equally involved; the limbic system (the “emotional brain”) is especially active, while the prefrontal cortex (the center of intellect and reasoning) is relatively quiet. “We wake up from dreams happy or depressed, and those feelings can stay with us all day.” says Stanford sleep researcher Dr. William Dement.

The link between dreams and emotions shows up among the patients in Cartwrights clinic. Most people seem to have more bad dreams early in the night, progressing toward happier ones before awakening, suggesting that they are working through negative feelings generated during the day. Because our conscious mind is occupied with daily life we dont always think about the emotional significance of the days events -- until, it appears, we begin to dream.

And this process need not be left to the unconscious. Cartwright believes one can exercise conscious control over recurring bad dreams. As soon as you awaken, identify what is upsetting about the dream. Visualize how you would like it to end instead; the next time it occurs, try to wake up just enough to control its course. With much practice people can learn to, literally, do it in their sleep.

At the end of the day, theres probably little reason to pay attention to our dreams at all unless they keep us from sleeping or we wake up in a panic, Cartwright says. Terrorism, economic uncertainties and general feelings of insecurity have increased people’s anxiety. Those suffering from persistent nightmares should seek help from a therapist. For the rest of us, the brain has its ways of working through bad feelings. Sleep -- or rather dream -- on it and you’ll feel better in the morning.

31. Researchers have come to believe that dreams ________.

[A] can be modified in their courses[B] are susceptible to emotional changes

[C] reflect our innermost desires and fearsA[D] are a random outcome of neural repairs

32. By referring to the limbic system, the author intends to show ________.

[A] its function in our dreams[B] the mechanism of REM sleep

[C] the relation of dreams to emotionsC[D] its difference from the prefrontal cortex

33. The negative feelings generated during the day tend to ________.

[A] aggravate in our unconscious mind[B] develop into happy dreams

[C] persist till the time we fall asleepD[D] show up in dreams early at night

34. Cartwright seems to suggest that ________.

[A] waking up in time is essential to the ridding of bad dreams[B] visualizing bad dreams helps bring them under control[C] dreams should be left to their natural progressionD

[D] dreaming may not entirely belong to the unconscious

35. What advice might Cartwright give to those who sometimes have bad dreams?

[A] Lead your life as usual.[B] Seek professional help.

[C] Exercise conscious control.A[D] Avoid anxiety in the daytime.

Text 4

Americans no longer expect public figures, whether in speech or in writing, to command the English language with skill and gift. Nor do they aspire to such command themselves. In his latest book, Doing Our Own Thing: The Degradation of Language and Music and Why We Should, Like, Care, John McWhorter, a linguist and controversialist of mixed liberal and conservative views, sees the triumph of 1960s counter-culture as responsible for the decline of formal English.

Blaming the permissive 1960s is nothing new, but this is not yet another criticism against the decline in education. Mr. McWhorters academic speciality is language history and change, and he sees the gradual disappearance of whom,” for example, to be natural and no more regrettable than the loss of the case-endings of Old English.

But the cult of the authentic and the personal, doing our own thing,” has spelt the death of formal speech, writing, poetry and music. While even the modestly educated sought an elevated tone when they put pen to paper before the 1960s, even the most well regarded writing since then has sought to capture spoken English on the page. Equally, in poetry, the highly personal, performative genre is the only form that could claim real liveliness. In both oral and written English, talking is triumphing over speaking, spontaneity over craft.

Illustrated with an entertaining array of examples from both high and low culture, the trend that Mr. McWhorter documents is unmistakable. But it is less clear, to take the question of his subtitle, why we should, like, care. As a linguist, he acknowledges that all varieties of human language, including non-standard ones like Black English, can be powerfully expressive -- there exists no language or dialect in the world that cannot convey complex ideas. He is not arguing, as many do, that we can no longer think straight because we do not talk proper.

Russians have a deep love for their own language and carry large chunks of memorized poetry in their heads, while Italian politicians tend to elaborate speech that would seem old-fashioned to most English-speakers. Mr. McWhorter acknowledges that formal language is not strictly necessary, and proposes no radical education reforms -- he is really grieving over the loss of something beautiful more than useful. We now take our English “on paper plates instead of china.” A shame, perhaps, but probably an inevitable one.

36. According to McWhorter, the decline of formal English ________.

[A] is inevitable in radical education reforms

[B] is but all too natural in language development

[C] has caused the controversy over the counter-cultureB

[D] brought about changes in public attitudes in the 1960s

37. The word talking (Line 6, Paragraph 3) denotes ________.

[A] modesty[B] personality[C] livelinessD[D] informality

38. To which of the following statements would McWhorter most likely agree?

[A] Logical thinking is not necessarily related to the way we talk.

[B] Black English can be more expressive than standard English.

[C] Non-standard varieties of human language are just as entertaining.A

[D] Of all the varieties, standard English can best convey complex ideas.

39. The description of Russians love of memorizing poetry shows the authors ________.

[A] interest in their language[B] appreciation of their efforts

[C] admiration for their memoryB

[D] contempt for their old-fashionedness

40. According to the last paragraph, paper plates is to china as ________.

[A] temporary is to permanent[B] radical is to conservative

[C] functional is to artisticC[D] humble is to noble

Part B

Directions:

In the following text, some sentences have been removed. For Questions 41-45, choose the most suitable one from the list A-G to fit into each of the numbered blanks. There are two extra choices, which do not fit in any of the gaps. Mark your answers on ANSWER SHEET 1. (10 points)

Canadas premiers (the leaders of provincial governments), if they have any breath left after complaining about Ottawa at their late July annual meeting, might spare a moment to do something, together, to reduce health-care costs.

Theyre all groaning about soaring health budgets, the fastest-growing component of which are pharmaceutical costs.

41. ____[E] According to the Canadian Institute for Health Information, prescription drug costs have risen since 1997 at twice the rate of overall health-care spending. Part of the increase comes from drugs being used to replace other kinds of treatments. Part of it arises from new drugs costing more than older kinds. Part of it is higher prices.____

What to do? Both the Romanow commission and the Kirby committee on health care -- to say nothing of reports from other experts -- recommended the creation of a national drug agency. Instead of each province having its own list of approved drugs, bureaucracy, procedures and limited bargaining power, all would pool resources, work with Ottawa, and create a national institution.

42. ____[C] What does “national” mean? Roy Romanow and Senator Michael Kirby recommended a federal-provincial body much like the recently created National Health Council.____

But national doesnt have to mean that. National could mean interprovincial -- provinces combining efforts to create one body.

Either way, one benefit of a national organization would be to negotiate better prices, if possible, with drug manufacturers. Instead of having one province -- or a series of hospitals within a province -- negotiate a price for a given drug on the provincial list, the national agency would negotiate on behalf of all provinces.

Rather than, say, Quebec, negotiating on behalf of seven million people, the national agency would negotiate on behalf of 31 million people. Basic economics suggests the greater the potential consumers, the higher the likelihood of a better price.

43. ____[G] Of course the pharmaceutical companies will scream. They like divided buyers; they can lobby better that way. They can use the threat of removing jobs from one province to another. They can hope that, if one province includes a drug on its list, the pressure will cause others to include it on theirs. They wouldn’t like a national agency, but self-interest would lead them to deal with it.____

A small step has been taken in the direction of a national agency with the creation of the Canadian Co-ordinating Office for Health Technology Assessment, funded by Ottawa and the provinces. Under it, a Common Drug Review recommends to provincial lists which new drugs should be included. Predictably, and regrettably, Quebec refused to join.

A few premiers are suspicious of any federal-provincial deal-making. They (particularly Quebec and Alberta) just want Ottawa to fork over additional billions with few, if any, strings attached. Thats one reason why the idea of a national list hasnt gone anywhere, while drug costs keep rising fast.

44. ____[F] So, if the provinces want to run the health-care show, they should prove they can run it, starting with an interprovincial health list that would end duplication, save administrative costs, prevent one province from being played off against another, and bargain for better drug prices.____

Premiers love to quote Mr. Romanows report selectively, especially the parts about more federal money. Perhaps they should read what he had to say about drugs: A national drug agency would provide governments more influence on pharmaceutical companies in order to constrain the ever-increasing cost of drugs.

45. ____[B] Or they could read Mr. Kirby’s report: “the substantial buying power of such an agency would strengthen the public prescription-drug insurance plans to negotiate the lowest possible purchase prices from drug companies.”____

So when the premiers gather in Niagara Falls to assemble their usual complaint list, they should also get cracking about something in their jurisdiction that would help their budgets and patients.

[A] Quebecs resistance to a national agency is provincialist ideology. One of the first advocates for a national list was a researcher at Laval University. Quebecs Drug Insurance Fund has seen its costs skyrocket with annual increases from 14.3 per cent to 26.8 per cent!

[B] Or they could read Mr. Kirbys report: “the substantial buying power of such an agency would strengthen the public prescription-drug insurance plans to negotiate the lowest possible purchase prices from drug companies.

[C] What does national mean? Roy Romanow and Senator Michael Kirby recommended a federal-provincial body much like the recently created National Health Council.

[D] The problem is simple and stark: health-care costs have been, are, and will continue to increase faster than government revenues.

[E] According to the Canadian Institute for Health Information, prescription drug costs have risen since 1997 at twice the rate of overall health-care spending. Part of the increase comes from drugs being used to replace other kinds of treatments. Part of it arises from new drugs costing more than older kinds. Part of it is higher prices.

[F] So, if the provinces want to run the health-care show, they should prove they can run it, starting with an interprovincial health list that would end duplication, save administrative costs, prevent one province from being played off against another, and bargain for better drug prices.

[G] Of course, the pharmaceutical companies will scream. They like divided buyers; they can lobby better that way. They can use the threat of removing jobs from one province to another. They can hope that, if one province includes a drug on its list, the pressure will cause others to include it on theirs. They wouldn’t like a national agency, but self-interest would lead them to deal with it.

Part C

Directions:

Read the following text carefully and then translate the underlined segments into Chinese. Your translation should be written clearly on ANSWER SHEET 2. (10 points)

It is not easy to talk about the role of the mass media in this overwhelmingly significant phase in European history. History and news become confused, and ones impressions tend to be a mixture of skepticism and optimism. 46) Television is one of the means by which these feelings are created and conveyed -- and perhaps never before has it served so much to connect different peoples and nations as in the recent events in Europe. The Europe that is now forming cannot be anything other than its peoples, their cultures and national identities. With this in mind we can begin to analyze the European television scene. 47) In Europe, as elsewhere, multi-media groups have been increasingly successful: groups which bring together television, radio, newspapers, magazines and publishing houses that work in relation to one another. One Italian example would be the Berlusconi group, while abroad Maxwell and Murdoch come to mind.

Clearly, only the biggest and most flexible television companies are going to be able to compete in such a rich and hotly-contested market. 48) This alone demonstrates that the television business is not an easy world to survive in, a fact underlined by statistics that show that out of eighty European television networks, no less than 50% took a loss in 1989.

Moreover, the integration of the European community will oblige television companies to cooperate more closely in terms of both production and distribution.

49) Creating a European identity that respects the different cultures and traditions which go to make up the connecting fabric of the Old Continent is no easy task and demands a strategic choice -- that of producing programs in Europe for Europe. This entails reducing our dependence on the North American market, whose programs relate to experiences and cultural traditions which are different from our own.

In order to achieve these objectives, we must concentrate more on co-productions, the exchange of news, documentary services and training. This also involves the agreements between European countries for the creation of a European bank for Television Production which, on the model of the European Investments Bank, will handle the finances necessary for production costs. 50) In dealing with a challenge on such a scale, it is no exaggeration to say “United we stand, divided we fall” -- and if I had to choose a slogan it would be “Unity in our diversity.” A unity of objectives that nonetheless respect the varied peculiarities of each country.

Section III Writing

Part A

51. Directions:

Two months ago you got a job as an editor for the magazine Designs & Fashions. But now you find that the work is not what you expected. You decide to quit. Write a letter to your boss, Mr. Wang, telling him your decision, stating your reason (s), and making an apology.

Write your letter with no less than 100 words. Write it neatly on ANSWER SHEET 2.

Do not sign your own name at the end of the letter; use Li Ming instead.

You do not need to write the address. (10 points)

Part B

52. Directions:

Write an essay of 160-200 words based on the following drawing. In your essay, you should first describe the drawing, then interpret its meaning, and give your comment on it.

You should write neatly on ANSWER SHEET 2. (20 points)

51. 参考范文

A Letter, to Quit

Jun 22, 2005

Dear Mr. Wang,

First of all, please allow me to express my deep sorry to you for my resignation. I do know that this will bring about much trouble to you so that I write to you for my explanation.

I decided to quit for some reasons as follows. To begin with, the job as an editor for the magazine Designs & Fashions is not suitable to me. What’s more, I am preparing for another degree and I prefer to further my study. Again, I apologize for my resignation to you!

I am looking forward to your early reply.

Sincerely yours,

Li Ming

Part B (20 points)

52. 参考范文

A Helpless Father

The picture ironically shows that a pitiable old man in rags is being helplessly kicked off by his three sons and a daughter, who all wear decent clothes. The father’s negligent children are all guarding their home gates lest their old father “roll into” their households. In other words, they four ignore their moral sense of assuming the responsibility for their old father even though they may be all living a satisfying life. That is a painful scene we often encounter in our daily life.

Sad to say, the moral decline of the younger generations may be a rather explosive situation in our modern society. People definitely have their living conditions improved by wider and wider margins, as evidenced by the four children’s decent dressing, but their moral sense still remains sadly unchanged or in some cases becomes dramatically downgrading. Most people might have become too much self-centered, and even worse, they discard the tradition of giving respect to the elderly. They no longer care for their elders, let alone their neighbors or the disadvantaged; instead they try every means to avoid responsibility for other citizens. When one cares for others, one might even appear stupid or may even be distrusted.

Therefore, we have to take some useful measures to avoid the scene that is mentioned above. We must launch a variety of campaigns about the return to the good tradition of giving help and love the elderly. Moreover, we must appeal to our government to establish some relevant laws to punish those who avoid their duties. The last but not the least, our respect for age is an indication of the progress of human society, as imperatives of traditions require. We sincerely wish that the old man could be welcome to any of the four households, elegantly dressed, and a smile on the face.

Section II Use of English

Directions:

Read the following text. Choose the best word (s) for each numbered blank and mark [A], [B], [C] or [D] on ANSWER SHEET 1. (10 points)

Many theories concerning the causes of juvenile delinquency (crimes committed by young people) focus either on the individual or on society as the major contributing influence. Theories on the individual suggest that children engage in criminal behavior they were not sufficiently penalized for previous misdeeds or that they have learned criminal behavior through with others. Theories focusing on the role of society suggest that children commit crimes in to their failure to rise above their socioeconomic status, as a rejection of middle-class values.

Most theories of juvenile delinquency have focused on children from disadvantaged families, the fact that children from wealthy homes also commit crimes. The latter may commit crimes lack of adequate parental control. All theories, however, are tentative and are to criticism.

Changes in the social structure may indirectly juvenile crime rates. For example, changes in the economy that to fewer job opportunities for youth and rising unemployment make gainful employment increasingly difficult to obtain. The resulting discontent may in lead more youths into criminal behavior.

Families have also changes these years. More families consist of one-parent households or two working parents; , children are likely to have less supervision at home was common in the traditional family . This lack of parental supervision is thought to be an influence on juvenile crime rates. Other causes of offensive acts include frustration or failure in school, the increased of drugs and alcohol, and the growing of child abuse and child neglect. All these conditions tend to increase the probability of a child committing a criminal act, a direct causal relationship has not yet been established.

21. [A] acting[B] relying[C] centeringC[D] commenting

22. [A] before[B] unless[C] untilD[D] because

23. [A] interactions[B] assimilation[C] cooperationA[D] consultation

24. [A] return[B] reply[C] referenceD[D] response

25. [A] or[B] but rather[C] butA[D] or else

26 [A] considering[B] ignoring[C] highlightingB[D] discarding

27. [A] on[B] in[C] forC[D] with

28. [A] immune[B] resistant[C] sensitiveD[D] subject

29. [A] affect[B] reduce[C] checkA[D] reflect

30. [A] point[B] lead[C] comeB[D] amount

31. [A] in general[B] on average[C] by contrastA[D] at length

32. [A] case[B] short[C] turnC[D] essence

33. [A] survived[B] noticed[C] undertakenD[D] experienced

34. [A] contrarily[B] consequently[C] similarlyB[D] simultaneously

35. [A] than[B] that[C] whichA[D] as

36. [A] system[B] structure[C] conceptB[D] heritage

37. [A] assessable[B] identifiable[C] negligibleB[D] incredible

38. [A] expense[B] restriction[C] allocationD[D] availability

39. [A] incidence[B] awareness[C] exposureA[D] popularity

40. [A] provided[B] since[C] althoughC[D] supposing

Section III Reading Comprehension

Part A

Directions:

Read the following four texts. Answer the questions below each text by choosing [A], [B], [C] or [D] Mark your answers on ANSWER SHEET 1. (40 points)

Text 1

Hunting for a job late last year, lawyer Gant Redmon stumbled across CareerBuilder, a job database on the Internet. He searched it with no success but was attracted by the site’s “personal search agent.” It’s an interactive feature that lets visitors key in job criteria such as location, title, and salary, then E-mails them when a matching position is posted in the database. Redmon chose the keywords legal, intellectual property, and Washington, D.C. Three weeks later, he got his first notification of an opening. “I struck gold,” says Redmon, who E-mailed his resume to the employer and won a position as in-house counsel for a company.

With thousands of career-related sites on the Internet, finding promising openings can be time-consuming and inefficient. Search agents reduce the need for repeated visits to the databases. But although a search agent worked for Redmon, career experts see drawbacks. Narrowing your criteria, for example, may work against you: “Every time you answer a question you eliminate a possibility.” says one expert.

For any job search, you should start with a narrow concept -- what you think you want to do -- then broaden it. “None of these programs do that,” says another expert. “There’s no career counseling implicit in all of this.” Instead, the best strategy is to use the agent as a kind of tip service to keep abreast of jobs in a particular database; when you get E-mail, consider it a reminder to check the database again. “I would not rely on agents for finding everything that is added to a database that might interest me,” says the author of a job-searching guide.

Some sites design their agents to tempt job hunters to return. When CareerSite’s agent sends out messages to those who have signed up for its service, for example, it includes only three potential jobs -- those it considers the best matches. There may be more matches in the database; job hunters will have to visit the site again to find them -- and they do. “On the day after we send our messages, we see a sharp increase in our traffic,” says Seth Peets, vice president of marketing for CareerSite.

Even those who aren’t hunting for jobs may find search agents worthwhile. Some use them to keep a close watch on the demand for their line of work or gather information on compensation to arm themselves when negotiating for a raise. Although happily employed, Redmon maintains his agent at CareerBuilder. “You always keep your eyes open,” he says. Working with a personal search agent means having another set of eyes looking out for you.

41. How did Redmon find his job?

[A] By searching openings in a job database.[B] By posting a matching position in a database.

[C] By using a special service of a database.C[D] By E-mailing his resume to a database.

42. Which of the following can be a disadvantage of search agents?

[A] Lack of counseling. [B] Limited number of visits.

[C] Lower efficiency.A [D] Fewer successful matches.

43. The expression “tip service” (Line 4, Paragraph 3) most probably means ________.

[A] advisory [B] compensation [C] interactionD [D] reminder

44. Why does CareerSite’s agent offer each job hunter only three job options?

[A] To focus on better job matches.[B] To attract more returning visits.

[C] To reserve space for more messages.B[D] To increase the rate of success.

45. Which of the following is true according to the text?

[A] Personal search agents are indispensable to job-hunters.

[B] Some sites keep E-mailing job seekers to trace their demands.

[C] Personal search agents are also helpful to those already employed.C

[D] Some agents stop sending information to people once they are employed.

Text 2

Over the past century, all kinds of unfairness and discrimination have been condemned or made illegal. But one insidious form continues to thrive: alphabetism. This, for those as yet unaware of such a disadvantage, refers to discrimination against those whose surnames begin with a letter in the lower half of the alphabet.

It has long been known that a taxi firm called AAAA cars has a big advantage over Zodiac cars when customers thumb through their phone directories. Less well known is the advantage that Adam Abbott has in life over Zoë Zysman. English names are fairly evenly spread between the halves of the alphabet. Yet a suspiciously large number of top people have surnames beginning with letters between A and K.

Thus the American president and vice-president have surnames starting with B and C respectively; and 26 of George Bush’s predecessors (including his father) had surnames in the first half of the alphabet against just 16 in the second half. Even more striking, six of the seven heads of government of the G7 rich countries are alphabetically advantaged (Berlusconi, Blair, Bush, Chirac, Chrétien and Koizumi). The world’s three top central bankers (Greenspan, Duisenberg and Hayami) are all close to the top of the alphabet, even if one of them really uses Japanese characters. As are the world’s five richest men (Gates, Buffett, Allen, Ellison and Albrecht).

Can this merely be coincidence? One theory, dreamt up in all the spare time enjoyed by the alphabetically disadvantaged, is that the rot sets in early. At the start of the first year in infant school, teachers seat pupils alphabetically from the front, to make it easier to remember their names. So short-sighted Zysman junior gets stuck in the back row, and is rarely asked the improving questions posed by those insensitive teachers. At the time the alphabetically disadvantaged may think they have had a lucky escape. Yet the result may be worse qualifications, because they get less individual attention, as well as less confidence in speaking publicly.

The humiliation continues. At university graduation ceremonies, the ABCs proudly get their awards first; by the time they reach the Zysmans most people are literally having a ZZZ. Shortlists for job interviews, election ballot papers, lists of conference speakers and attendees: all tend to be drawn up alphabetically, and their recipients lose interest as they plough through them.

46. What does the author intend to illustrate with AAA A cars and Zodiac cars?

[A] A kind of overlooked inequality.[B] A type of conspicuous bias.

[C] A type of personal prejudice.A[D] A kind of brand discrimination.

47. What can we infer from the first three paragraphs?

[A] In both East and West, names are essential to success.

[B] The alphabet is to blame for the failure of Zoë Zysman.

[C] Customers often pay a lot of attention to companies’ names.D

[D] Some form of discrimination is too subtle to recognize.

48. The 4th paragraph suggests that ________.

[A] questions are often put to the more intelligent students

[B] alphabetically disadvantaged students often escape from class

[C] teachers should pay attention to all of their studentsC

[D] students should be seated according to their eyesight

49. What does the author mean by “most people are literally having a ZZZ” (Lines 2-3, Paragraph 5)?

[A] They are getting impatient. [B] They are noisily dozing off.

[C] They are feeling humiliated.B [D] They are busy with word puzzles.

50. Which of the following is true according to the text?

[A] People with surnames beginning with N to Z are often ill-treated.

[B] VIPs in the Western world gain a great deal from alphabetism.

[C] The campaign to eliminate alphabetism still has a long way to go.D

[D] Putting things alphabetically may lead to unintentional bias.

Text 3

When it comes to the slowing economy, Ellen Spero isn’t biting her nails just yet. But the 47-year-old manicurist isn’t cutting, filing or polishing as many nails as she’d like to, either. Most of her clients spend $12 to $50 weekly, but last month two longtime customers suddenly stopped showing up. Spero blames the softening economy. “I’m a good economic indicator,” she says. “I provide a service that people can do without when they’re concerned about saving some dollars.” So Spero is downscaling, shopping at middle-brow Dillard’s department store near her suburban Cleveland home, instead of Neiman Marcus. “I don’t know if other clients are going to abandon me, too.” she says.

Even before Alan Greenspan’s admission that America’s red-hot economy is cooling, lots of working folks had already seen signs of the slowdown themselves. From car dealerships to Gap outlets, sales have been lagging for months as shoppers temper their spending. For retailers, who last year took in 24 percent of their revenue between Thanksgiving and Christmas, the cautious approach is coming at a crucial time. Already, experts say, holiday sales are off 7 percent from last year’s pace. But don’t sound any alarms just yet. Consumers seem only mildly concerned, not panicked, and many say they remain optimistic about the economy’s long-term prospects, even as they do some modest belt-tightening.

Consumers say they’re not in despair because, despite the dreadful headlines, their own fortunes still feel pretty good. Home prices are holding steady in most regions. In Manhattan, “there’s a new gold rush happening in the $4 million to $10 million range, predominantly fed by Wall Street bonuses,” says broker Barbara Corcoran. In San Francisco, prices are still rising even as frenzied overbidding quiets. “Instead of 20 to 30 offers, now maybe you only get two or three,” says John Tealdi, a Bay Area real-estate broker. And most folks still feel pretty comfortable about their ability to find and keep a job.

Many folks see silver linings to this slowdown. Potential home buyers would cheer for lower interest rates. Employers wouldn’t mind a little fewer bubbles in the job market. Many consumers seem to have been influenced by stock-market swings, which investors now view as a necessary ingredient to a sustained boom. Diners might see an upside, too. Getting a table at Manhattan’s hot new Alain Ducasse restaurant used to be impossible. Not anymore. For that, Greenspan & Co. may still be worth toasting.

51. By “Ellen Spero isn’t biting her nails just yet” (Lines 1-2, Paragraph 1), the author means ________.

[A] Spero can hardly maintain her business [B] Spero is too much engaged in her work

[C] Spero has grown out of her bad habitD [D] Spero is not in a desperate situation

52. How do the public feel about the current economic situation?

[A] Optimistic. [B] Confused. [C] Carefree.A [D] Panicked.

53. When mentioning “the $4 million to $10 million range” (Lines 3-4, Paragraph 3) the author is talking about ________.

[A] gold market [B] real estate [C] stock exchangeB [D] venture investment

54. Why can many people see “silver linings” to the economic slowdown?

[A] They would benefit in certain ways.[B] The stock market shows signs of recovery.

[C] Such a slowdown usually precedes a boom.A[D] The purchasing power would be enhanced.

55. To which of the following is the author likely to agree?

[A] A new boom, on the horizon.[B] Tighten the belt, the single remedy.

[C] Caution all right, panic not.C[D] The more ventures, the more chances.

Text 4

Americans today don’t place a very high value on intellect. Our heroes are athletes, entertainers, and entrepreneurs, not scholars. Even our schools are where we send our children to get a practical education -- not to pursue knowledge for the sake of knowledge. Symptoms of pervasive anti-intellectualism in our schools aren’t difficult to find.

“Schools have always been in a society where practical is more important than intellectual,” says education writer Diane Ravitch. “Schools could be a counterbalance.” Ravitch’s latest book, Left Back: A Century of Failed School Reforms, traces the roots of anti-intellectualism in our schools, concluding they are anything but a counterbalance to the American distaste for intellectual pursuits.

But they could and should be. Encouraging kids to reject the life of the mind leaves them vulnerable to exploitation and control. Without the ability to think critically, to defend their ideas and understand the ideas of others, they cannot fully participate in our democracy. Continuing along this path, says writer Earl Shorris, “We will become a second-rate country. We will have a less civil society.”

“Intellect is resented as a form of power or privilege,” writes historian and professor Richard Hofstadter in Anti-Intellectualism in American Life, a Pulitzer-Prize winning book on the roots of anti-intellectualism in US politics, religion, and education. From the beginning of our history, says Hofstadter, our democratic and populist urges have driven us to reject anything that smells of elitism. Practicality, common sense, and native intelligence have been considered more noble qualities than anything you could learn from a book.

Ralph Waldo Emerson and other Transcendentalist philosophers thought schooling and rigorous book learning put unnatural restraints on children: “We are shut up in schools and college recitation rooms for 10 or 15 years and come out at last with a bellyful of words and do not know a thing.” Mark Twain’s Huckleberry Finn exemplified American anti-intellectualism. Its hero avoids being civilized -- going to school and learning to read -- so he can preserve his innate goodness.

Intellect, according to Hofstadter, is different from native intelligence, a quality we reluctantly admire. Intellect is the critical, creative, and contemplative side of the mind. Intelligence seeks to grasp, manipulate, re-order, and adjust, while intellect examines, ponders, wonders, theorizes, criticizes and imagines.

School remains a place where intellect is mistrusted. Hofstadter says our country’s educational system is in the grips of people who “joyfully and militantly proclaim their hostility to intellect and their eagerness to identify with children who show the least intellectual promise.”

56. What do American parents expect their children to acquire in school?

[A] The habit of thinking independently.[B] Profound knowledge of the world.

[C] Practical abilities for future career.C[D] The confidence in intellectual pursuits.

57. We can learn from the text that Americans have a history of ________.

[A] undervaluing intellect [B] favoring intellectualism

[C] supporting school reformA [D] suppressing native intelligence

58. The views of Ravitch and Emerson on schooling are ________.

[A] identical [B] similar [C] complementaryD [D] opposite

59. Emerson, according to the text, is probably ________.

[A] a pioneer of education reform [B] an opponent of intellectualism

[C] a scholar in favor of intellectB [D] an advocate of regular schooling

60. What does the author think of intellect?

[A] It is second to intelligence. [B] It evolves from common sense.

[C] It is to be pursued.C [D] It underlies power.

Part B

Directions:

Read the following text carefully and then translate the underlined segments into Chinese. Your translation should be written clearly on ANSWER SHEET 2. (10 points)

The relation of language and mind has interested philosophers for many centuries. 61) The Greeks assumed that the structure of language had some connection with the process of thought, which took root in Europe long before people realized how diverse languages could be.

Only recently did linguists begin the serious study of languages that were very different from their own. Two anthropologist-linguists, Franz Boas and Edward Sapir, were pioneers in describing many native languages of North and South America during the first half of the twentieth century. 62) We are obliged to them because some of these languages have since vanished, as the peoples who spoke them died out or became assimilated and lost their native languages. Other linguists in the earlier part of this century, however, who were less eager to deal with bizarre data from “exotic” language, were not always so grateful. 63) The newly described languages were often so strikingly different from the well studied languages of Europe and Southeast Asia that some scholars even accused Boas and Sapir of fabricating their data. Native American languages are indeed different, so much so in fact that Navajo could be used by the US military as a code during World War II to send secret messages.

Sapir’s pupil, Benjamin Lee Whorf, continued the study of American Indian languages. 64) Being interested in the relationship of language and thought, Whorf developed the idea that the structure of language determines the structure of habitual thought in a society. He reasoned that because it is easier to formulate certain concepts and not others in a given language, the speakers of that language think along one track and not along another. 65) Whorf came to believe in a sort of linguistic determinism which, in its strongest form, states that language imprisons the mind, and that the grammatical patterns in a language can produce far-reaching consequences for the culture of a society. Later, this idea became to be known as the Sapir-Whorf hypothesis, but this term is somewhat inappropriate. Although both Sapir and Whorf emphasized the diversity of languages, Sapir himself never explicitly supported the notion of linguistic determinism.

61. ________ 62. ________ 63. ________64. ________65. ________

Section IV Writing

66. Directions:

Study the following drawing carefully and write an essay in which you should

1) describe the drawing, 2) interpret its meaning, and 3) support your view with examples.

You should write about 200 words neatly on ANSWER SHEET 2. (20 points)

Section IV: Writing (20 points)

66. 参考范文

Look at this picture. A man is running toward the end of a race, sweating all over. Perhaps there is nothing special about the man and the race, yet the end line leaves a deep impression on us for it is both finish line and starting line if we look at it from a different angle.

The picture tells us a lot about life. An ordinary runner may think that his achievement calls for celebration for he has reached his goal. But an ambitious runner will well realize that his success is the starting point for a new race. Obviously, the way we look at things determines how far we will go. If we feel satisfied with the ability to read and write, perhaps we will not strive to get a college education. Again, if we think a Bachelor degree is quite enough, we will not take pains to pass the examination for postgraduate. Life is just like an endless race. If we dont prepare for new races, we are either disqualified from the race or surpassed by others. Thats why the finish line is also a starting live.

I like this picture. I may have been running this endless race, but I have been pushed forward by my parents or people around me. From now on, I will become more active and take the initiative, for the picture has really enlightened me.(233 words)

Section II Use of English

Directions:

Read the following text. Choose the best word (s) for each numbered blank and mark [A], [B], [C] or [D] on ANSWER SHEET 1. (10 points)

Teachers need to be aware of the emotional, intellectual, and physical changes that young adults experience. And they also need to give serious to how they can best such changes. Growing bodies need movement and , but not just in ways that emphasize competition. they are adjusting to their new bodies and a whole host of new intellectual and emotional challenges, teenagers are especially self-conscious and need the that comes from achieving success and knowing that their accomplishments are by others. However, the typical teenage lifestyle is already filled with so much competition that it would be to plan activities in which there are more winners than losers, , publishing newsletters with many student-written book reviews, student artwork, and sponsoring book discussion clubs. A variety of small clubs can provide opportunities for leadership, as well as for practice in successful dynamics. Making friends is extremely important to teenagers, and many shy students need the of some kind of organization with a supportive adult visible in the background.

In these activities, it is important to remember that the young teens have attention spans. A variety of activities should be organized participants can remain active as long as they want and then go on to else without feeling guilty and without letting the other participants . This does not mean that adults must accept irresponsibility. , they can help students acquire a sense of commitment by for roles that are within their and their attention spans and by having clearly stated rules.

21. [A] thought[B] idea[C] opinionA[D] advice

22. [A] strengthen[B] accommodate[C] stimulateB[D] enhance

23. [A] care[B] nutrition[C] exerciseC[D] leisure

24. [A] If[B] Although[C] WhereasD[D] Because

25. [A] assistance[B] guidance[C] confidenceC[D] tolerance

26. [A] claimed[B] admired[C] ignoredB[D] surpassed

27. [A] improper[B] risky[C] fairD[D] wise

28. [A] in effect[B] as a result[C] for exampleC[D] in a sense

29. [A] displaying[B] describing[C] creatingA[D] exchanging

30. [A] durable[B] excessive[C] surplusD[D] multiple

31. [A] groups[B] individual[C] personnelA[D] corporation

32. [A] consent[B] insurance[C] admissionD[D] security

33. [A] particularly[B] barely[C] definitelyB[D] rarely

34. [A] similar[B] long[C] differentD[D] short

35. [A] if only[B] now that[C] so thatC[D] even if

36. [A] everything[B] anything[C] nothingD[D] something

37. [A] off[B] down[C] outB[D] alone

38. [A] On the contrary[B] On the average[C] On the wholeA[D] On the other hand

39. [A] making[B] standing[C] planningC[D] taking

40. [A] capability[B] responsibility[C] proficiencyA[D] efficiency

Section III Reading Comprehension

Part A

Directions:

Read the following four texts. Answer the questions below each text by choosing [A], [B], [C] or [D]. Mark your answers on ANSWER SHEET 1 (40 points)

Text 1

Wild Bill Donovan would have loved the Internet. The American spymaster who built the Office of Strategic Services in the World War II and later laid the roots for the CIA was fascinated with information. Donovan believed in using whatever tools came to hand in the “great game” of espionage -- spying as a “profession.” These days the Net, which has already re-made such everyday pastimes as buying books and sending mail, is reshaping Donovan’s vocation as well.

The latest revolution isnt simply a matter of gentlemen reading other gentlemens e-mail. That kind of electronic spying has been going on for decades. In the past three or four years, the World Wide Web has given birth to a whole industry of point-and-click spying. The spooks call it “open-source intelligence,” and as the Net grows, it is becoming increasingly influential. In 1995 the CIA held a contest to see who could compile the most data about Burundi. The winner, by a large margin, was a tiny Virginia company called Open Source Solutions, whose clear advantage was its mastery of the electronic world.

Among the firms making the biggest splash in the new world is Straitford, Inc., a private intelligence-analysis firm based in Austin, Texas. Straitford makes money by selling the results of spying (covering nations from Chile to Russia) to corporations like energy-services firm McDermott International. Many of its predictions are available online at .

Straitford president George Friedman says he sees the online world as a kind of mutually reinforcing tool for both information collection and distribution, a spymasters dream. Last week his firm was busy vacuuming up data bits from the far corners of the world and predicting a crisis in Ukraine. “As soon as that report runs, we’ll suddenly get 500 new Internet sign-ups from Ukraine,” says Friedman, a former political science professor. “And we’ll hear back from some of them.” Open-source spying does have its risks, of course, since it can be difficult to tell good information from bad. That’s where Straitford earns its keep.

Friedman relies on a lean staff in Austin. Several of his staff members have military-intelligence backgrounds. He sees the firms outsider status as the key to its success. Straitfords briefs dont sound like the usual Washington back-and-forthing, whereby agencies avoid dramatic declarations on the chance they might be wrong. Straitford, says Friedman, takes pride in its independent voice.

41. The emergence of the Net has ________.

[A] received support from fans like Donovan [B] remolded the intelligence services

[C] restored many common pastimesB [D] revived spying as a profession

42. Donovans story is mentioned in the text to ________.

[A] introduce the topic of online spying [B] show how he fought for the U.S.

[C] give an episode of the information warA [D] honor his unique services to the CIA

43. The phrase making the biggest splash (Line 1, Paragraph 3) most probably means ________.

[A] causing the biggest trouble [B] exerting the greatest effort

[C] achieving the greatest successC [D] enjoying the widest popularity

44. It can be learned from Paragraph 4 that ________.

[A] Straitfords prediction about Ukraine has proved true

[B] Straitford guarantees the truthfulness of its information

[C] Straitfords business is characterized by unpredictabilityD

[D] Straitford is able to provide fairly reliable information

45. Straitford is most proud of its ________.

[A] official status [B] nonconformist image [C] efficient staffB [D] military background

Text 2

To paraphrase 18th-century statesman Edmund Burke, all that is needed for the triumph of a misguided cause is that good people do nothing. One such cause now seeks to end biomedical research because of the theory that animals have rights ruling out their use in research. Scientists need to respond forcefully to animal rights advocates, whose arguments are confusing the public and thereby threatening advances in health knowledge and care. Leaders of the animal rights movement target biomedical research because it depends on public funding, and few people understand the process of health care research. Hearing allegations of cruelty to animals in research settings, many are perplexed that anyone would deliberately harm an animal.

For example, a grandmotherly woman staffing an animal rights booth at a recent street fair was distributing a brochure that encouraged readers not to use anything that comes from or is tested in animalsno meat, no fur, no medicines. Asked if she opposed immunizations, she wanted to know if vaccines come from animal research. When assured that they do, she replied, “Then I would have to say yes.” Asked what will happen when epidemics return, she said, “Don’t worry, scientists will find some way of using computers.” Such well-meaning people just don’t understand.

Scientists must communicate their message to the public in a compassionate, understandable way -- in human terms, not in the language of molecular biology. We need to make clear the connection between animal research and a grandmothers hip replacement, a father’s bypass operation, a baby’s vaccinations, and even a pet’s shots. To those who are unaware that animal research was needed to produce these treatments, as well as new treatments and vaccines, animal research seems wasteful at best and cruel at worst.

Much can be done. Scientists could adopt middle school classes and present their own research. They should be quick to respond to letters to the editor, lest animal rights misinformation go unchallenged and acquire a deceptive appearance of truth. Research institutions could be opened to tours, to show that laboratory animals receive humane care. Finally, because the ultimate stakeholders are patients, the health research community should actively recruit to its cause not only well-known personalities such as Stephen Cooper, who has made courageous statements about the value of animal research, but all who receive medical treatment. If good people do nothing there is a real possibility that an uninformed citizenry will extinguish the precious embers of medical progress.

46. The author begins his article with Edmund Burkes words to ________.

[A] call on scientists to take some actions[B] criticize the misguided cause of animal rights

[C] warn of the doom of biomedical researchA[D] show the triumph of the animal rights movement

47. Misled people tend to think that using an animal in research is ________.

[A] cruel but natural [B] inhuman and unacceptable

[C] inevitable but viciousB [D] pointless and wasteful

48. The example of the grandmotherly woman is used to show the publics ________.

[A] discontent with animal research [B] ignorance about medical science

[C] indifference to epidemicsB [D] anxiety about animal rights

49. The author believes that, in face of the challenge from animal rights advocates, scientists should ________.

[A] communicate more with the public [B] employ hi-tech means in research

[C] feel no shame for their causeA [D] strive to develop new cures

50. From the text we learn that Stephen Cooper is ________.

[A] a well-known humanist [B] a medical practitioner

[C] an enthusiast in animal rightsD [D] a supporter of animal research

Text 3

In recent years, railroads have been combining with each other, merging into super systems, causing heightened concerns about monopoly. As recently as 1995, the top four railroads accounted for under 70 percent of the total ton-miles moved by rails. Next year, after a series of mergers is completed, just four railroads will control well over 90 percent of all the freight moved by major rail carriers.

Supporters of the new super systems argue that these mergers will allow for substantial cost reductions and better coordinated service. Any threat of monopoly, they argue, is removed by fierce competition from trucks. But many shippers complain that for heavy bulk commodities traveling long distances, such as coal, chemicals, and grain, trucking is too costly and the railroads therefore have them by the throat.

The vast consolidation within the rail industry means that most shippers are served by only one rail company. Railroads typically charge such captive shippers 20 to 30 percent more than they do when another railroad is competing for the business. Shippers who feel they are being overcharged have the right to appeal to the federal government’s Surface Transportation Board for rate relief, but the process is expensive, time-consuming, and will work only in truly extreme cases.

Railroads justify rate discrimination against captive shippers on the grounds that in the long run it reduces everyones cost. If railroads charged all customers the same average rate, they argue, shippers who have the option of switching to trucks or other forms of transportation would do so, leaving remaining customers to shoulder the cost of keeping up the line. It’s a theory to which many economists subscribe, but in practice it often leaves railroads in the position of determining which companies will flourish and which will fail. “Do we really want railroads to be the arbiters of who wins and who loses in the marketplace?” asks Martin Bercovici, a Washington lawyer who frequently represents shipper.

Many captive shippers also worry they will soon be hit with a round of huge rate increases. The railroad industry as a whole, despite its brightening fortunes, still does not earn enough to cover the cost of the capital it must invest to keep up with its surging traffic. Yet railroads continue to borrow billions to acquire one another, with Wall Street cheering them on. Consider the $10.2 billion bid by Norfolk Southern and CSX to acquire Conrail this year. Conrail’s net railway operating income in 1996 was just $427 million, less than half of the carrying costs of the transaction. Who’s going to pay for the rest of the bill? Many captive shippers fear that they will, as Norfolk Southern and CSX increase their grip on the market.

51. According to those who support mergers, railway monopoly is unlikely because ________.

[A] cost reduction is based on competition[B] services call for cross-trade coordination

[C] outside competitors will continue to existC[D] shippers will have the railway by the throat

52. What is many captive shippers attitude towards the consolidation in the rail industry?

[A] Indifferent. [B] Supportive. [C] Indignant.D[D] Apprehensive.

53. It can be inferred from Paragraph 3 that ________.

[A] shippers will be charged less without a rival railroad

[B] there will soon be only one railroad company nationwide

[C] overcharged shippers are unlikely to appeal for rate reliefC

[D] a government board ensures fair play in railway business

54. The word arbiters (Line 7, Paragraph 4) most probably refers to those ________.

[A] who work as coordinators [B] who function as judges

[C] who supervise transactionsB[D] who determine the price

55. According to the text, the cost increase in the rail industry is mainly caused by ________.

[A] the continuing acquisition [B] the growing traffic

[C] the cheering Wall StreetA[D] the shrinking market

Text 4

It is said that in England death is pressing, in Canada inevitable and in California optional. Small wonder. Americans life expectancy has nearly doubled over the past century. Failing hips can be replaced, clinical depression controlled, cataracts removed in a 30-minutes surgical procedure. Such advances offer the aging population a quality of life that was unimaginable when I entered medicine 50 years ago. But not even a great health-care system can cure death -- and our failure to confront that reality now threatens this greatness of ours.

Death is normal; we are genetically programmed to disintegrate and perish, even under ideal conditions. We all understand that at some level, yet as medical consumers we treat death as a problem to be solved. Shielded by third-party payers from the cost of our care, we demand everything that can possibly be done for us, even if it’s useless. The most obvious example is late-stage cancer care. Physicians -- frustrated by their inability to cure the disease and fearing loss of hope in the patient -- too often offer aggressive treatment far beyond what is scientifically justified.

In 1950, the U.S. spent $12.7 billion on health care. In 2002, the cost will be $1,540 billion. Anyone can see this trend is unsustainable. Yet few seem willing to try to reverse it. Some scholars conclude that a government with finite resources should simply stop paying for medical care that sustains life beyond a certain age -- say 83 or so. Former Colorado governor Richard Lamm has been quoted as saying that the old and infirm “have a duty to die and get out of the way,” so that younger, healthier people can realize their potential.

I would not go that far. Energetic people now routinely work through their 60s and beyond, and remain dazzlingly productive. At 78, Viacom chairman Sumner Redstone jokingly claims to be 53. Supreme Court Justice Sandra Day OConnor is in her 70s, and former surgeon general C. Everett Koop chairs an Internet start-up in his 80s. These leaders are living proof that prevention works and that we can manage the health problems that come naturally with age. As a mere 68-year-old, I wish to age as productively as they have.

Yet there are limits to what a society can spend in this pursuit. Ask a physician, I know the most costly and dramatic measures may be ineffective and painful. I also know that people in Japan and Sweden, countries that spend far less on medical care, have achieved longer, healthier lives than we have. As a nation, we may be overfunding the quest for unlikely cures while underfunding research on humbler therapies that could improve people’s lives.

56. What is implied in the first sentence?

[A] Americans are better prepared for death than other people.

[B] Americans enjoy a higher life quality than ever before.

[C] Americans are over-confident of their medical technology.C

[D] Americans take a vain pride in their long life expectancy.

57. The author uses the example of cancer patients to show that ________.

[A] medical resources are often wasted [B] doctors are helpless against fatal diseases

[C] some treatments are too aggressiveA[D] medical costs are becoming unaffordable

58. The authors attitude toward Richard Lamms remark is one of ________.

[A] strong disapproval[B] reserved consent[C] slight contemptB[D] enthusiastic support

59. In contrast to the U.S., Japan and Sweden are funding their medical care ________.

[A] more flexibly[B] more extravagantly[C] more cautiouslyD[D] more reasonably

60. The text intends to express the idea that ________.

[A] medicine will further prolong peoples lives[B] life beyond a certain limit is not worth living

[C] death should be accepted as a fact of lifeC[D] excessive demands increase the cost of health care

Part B

Directions:

Read the following text carefully and then translate the underlined segments into Chinese. Your translation should be written clearly on ANSWER SHEET 2. (10 points)

Human beings in all times and places think about their world and wonder at their place in it. Humans are thoughtful and creative, possessed of insatiable curiosity. 61) Furthermore, humans have the ability to modify the environment in which they live, thus subjecting all other life forms to their own peculiar ideas and fancies. Therefore, it is important to study humans in all their richness and diversity in a calm and systematic manner, with the hope that the knowledge resulting from such studies can lead humans to a more harmonious way of living with themselves and with all other life forms on this planet Earth.

Anthropology derives from the Greek words anthropos human and logos the study of. By its very name, anthropology encompasses the study of all humankind.

Anthropology is one of the social sciences. 62) Social science is that branch of intellectual enquiry which seeks to study humans and their endeavors in the same reasoned, orderly, systematic, and dispassioned manner that natural scientists use for the study of natural phenomena.

Social science disciplines include geography, economics, political science, psychology, and sociology. Each of these social sciences has a subfield or specialization which lies particularly close to anthropology.

All the social sciences focus upon the study of humanity. Anthropology is a field-study oriented discipline which makes extensive use of the comparative method in analysis. 63) The emphasis on data gathered first-hand, combined with a cross-cultural perspective brought to the analysis of cultures past and present, makes this study a unique and distinctly important social science.

Anthropological analyses rest heavily upon the concept of culture. Sir Edward Tylors formulation of the concept of culture was one of the great intellectual achievements of 19th century science. 64) Tylor defined culture as “… that complex whole which includes belief, art, morals, law, custom, and any other capabilities and habits acquired by man as a member of society.” This insight, so profound in its simplicity, opened up an entirely new way of perceiving and understanding human life. Implicit within Tylor’s definition is the concept that culture is learned, shared, and patterned behavior.

65) Thus, the anthropological concept of culture, like the concept of set in mathematics, is an abstract concept which makes possible immense amounts of concrete research and understanding.

Section IV Writing

66. Directions:

Study the following set of drawings carefully and write an essay in which you should

1) describe the set of drawings, interpret its meaning, and

2) point out its implications in our life.

You should write about 200 words neatly on ANSWER SHEET 2. (20 points)

66. 参考范文

As is shown in the two pictures, the same flower has different fate when put in different situations. When there is protection, the flower blooms beautifully. However, when exposed directly to rain and storm, the flower quickly withers. We can safely come to the conclusion that greenhouse flowers cannot withstand rain and storm.

The same is true with our children. As the one-child policy is adopted in China, more and more parents overprotect their children. With everything done for them and every need satisfied by their parents or grandparents, these only children seem to be healthy, happy and high-spirited. But when they leave home, they become disappointed and frustrated. Facing harsh things they have never experienced before, such as competition from peers, criticism from supervisors, pressure for deadlines and restriction for discipline, they will be at a loss, miserable or even desperate. In fact, the more protection they get from their parents, the less ability they acquire. Parents love for their children turns out to be a disaster.

Children, like flowers, should be treated with care, but reasonable and sensible ways of doing things will help to bring about more fruitful results, if we not only love our children but also teach them how to love, our love will be passed down from generation to generation; if we not only solve problems for our children but also encourage them to solve problems, our diligence and ingenuity will make our life more meaningful and worthwhile.

本文来源:https://www.2haoxitong.net/k/doc/e89889d269ec0975f46527d3240c844768eaa073.html

《2001--2012考研英语一真题及答案.doc》
将本文的Word文档下载到电脑,方便收藏和打印
推荐度:
点击下载文档

文档为doc格式